Jump to content

Wikipedia:Reference desk/Science

From Wikipedia, the free encyclopedia

This is an old revision of this page, as edited by 82.43.88.151 (talk) at 13:10, 2 August 2010 (→‎documentary: new section). The present address (URL) is a permanent link to this revision, which may differ significantly from the current revision.

Welcome to the science section
of the Wikipedia reference desk.
Select a section:
Want a faster answer?

Main page: Help searching Wikipedia

   

How can I get my question answered?

  • Select the section of the desk that best fits the general topic of your question (see the navigation column to the right).
  • Post your question to only one section, providing a short header that gives the topic of your question.
  • Type '~~~~' (that is, four tilde characters) at the end – this signs and dates your contribution so we know who wrote what and when.
  • Don't post personal contact information – it will be removed. Any answers will be provided here.
  • Please be as specific as possible, and include all relevant context – the usefulness of answers may depend on the context.
  • Note:
    • We don't answer (and may remove) questions that require medical diagnosis or legal advice.
    • We don't answer requests for opinions, predictions or debate.
    • We don't do your homework for you, though we'll help you past the stuck point.
    • We don't conduct original research or provide a free source of ideas, but we'll help you find information you need.



How do I answer a question?

Main page: Wikipedia:Reference desk/Guidelines

  • The best answers address the question directly, and back up facts with wikilinks and links to sources. Do not edit others' comments and do not give any medical or legal advice.
See also:



July 26

Smardale RIGS

Smardale bridge is listed on List of Regionally Important Geological / Geomorphicological Sites (RIGS) in Cumbria - does anyone know where or how I can find out why it's listed, or more info? Thanks. Sf5xeplus (talk) 00:00, 26 July 2010 (UTC)[reply]

Grid reference NY725055. Physchim62 (talk) 00:30, 26 July 2010 (UTC)[reply]
The very first Google hit for "Smardale bridge" shows a photo of the bridge, with the caption "Smardale Bridge crosses Smardale Gill.... Behind the bridge are 'Giants' Graves,' archaeological features of uncertain origin that date to prehistoric times." There are lots more links beside. Looie496 (talk) 05:45, 26 July 2010 (UTC)[reply]
No.No. You misunderstand, I know where it is. My question is: why is/what is it geologically important and how can I find more about the geological features that are important. Sf5xeplus (talk) 11:23, 26 July 2010 (UTC)[reply]
Don't forget that someone could have added it when it wasn't regionally important. --Chemicalinterest (talk) 13:14, 26 July 2010 (UTC)[reply]
I think your best bet is to simply ask the Cumbria RIGS Group what the criteria for the listing were (there's a "Contact us" link at the bottom of the page). Alternatively, since the group says that "site details are logged with local authorities and the County Council", you might try the county council. Deor (talk) 15:58, 26 July 2010 (UTC)[reply]

Effeminacy

What causes effeminacy in gay males? —Preceding unsigned comment added by 88.104.91.111 (talk) 06:29, 26 July 2010 (UTC)[reply]

Trying to answer the question inevitably opens the Nature versus nurture debate about causes. Cuddlyable3 (talk) 10:29, 26 July 2010 (UTC)[reply]
Probably the same thing that causes effeminacy in straight males. --LarryMac | Talk 12:58, 26 July 2010 (UTC)[reply]
Many (most?) gay males don't exhibit any more effeminacy than straight males do. Many gay males gravitate towards the hyper-masculine, others might be 'bears', others don't behave any different from anybody else. It's largely a social thing, mixed with whatever makes the person comfortable. So if we narrow your question down to "what causes effeminacy in the small percentage of gay males who exhibit a marked tendency towards effeminacy?", then Cuddlyable3 and LarryMac's answers have it covered. 76.105.238.158 (talk) 00:32, 27 July 2010 (UTC)[reply]

Non-scientific answer: because the effeminate ones are cool, sociable, expressive and not shy. So they tend to be popular and more visible. John Riemann Soong (talk) 03:22, 27 July 2010 (UTC)[reply]

They purposely do that so people know their gay. there is no other explanation.--Horseluv10 11:55, 27 July 2010 (UTC) —Preceding unsigned comment added by Horseluv10 (talkcontribs)

Why do straight effiminate males do it then? And why don't the other gay males bother? And why do some gay effiminate males stay in the closet for so long? And what's wrong with the plenty of other explainations like the ones above? Nil Einne (talk) 21:34, 27 July 2010 (UTC)[reply]
What possible meaningful scientific answer do you imagine you could get for the above questions? Ask your self why YOU behave the way you behave? Why you are attracted to (presumably) girls instead of boys? If you can come up with a more meaningful answer then "because they're hot", and you aren't allowed to cheat by saying "millions of years of evolution" because that's something you are told by others, I'm sure you aren't thinking "I'm sure glad evolution has made me attracted to girls so we can have lots of offspring" when you admire someone you fancy. Human behavior is probably one of the most complex subjects you can study and people spend lifetimes trying to understand and explain it, often getting it totally wrong. It doesn't help that there are a LOT of errors and misconceptions in "common wisdom" about psychology. Vespine (talk) 23:30, 27 July 2010 (UTC)[reply]
I'm pretty sure Nil was directly and exclusively responding to Horseluv's simplistic and dogmatic statement. 86.164.66.83 (talk) 02:39, 28 July 2010 (UTC)[reply]
Yeah that makes more sense reading it again, sorry my rhetorical detector wasn't functioning correctly.. It's precisely that dogma around this subject that frustrates me, perhaps a little too quickly. Part of the reason for that is Nil's questions would NOT be the most ignorant questions I've ever heard about sexual preference if he had been serious.Vespine (talk) 05:00, 28 July 2010 (UTC)[reply]
Yeah 86 is correct. No problem about the confusion since as you may have guessed I too dislike silly statements about sexuality. You are correct that I'm heterosexual in any case. Nil Einne (talk) 09:53, 28 July 2010 (UTC)[reply]
This isn't a comment on your statement, just an interesting thought I've had, that when I answer people who "question" homosexuality I honestly have to fight the urge to say that I'm hetrosexual, because I can almost feel the people on the other end thinking "oh he's probably gay". But I know pointing out I'm hetro won't change their opinion anyway so unless they ask I make it a point not to mention it. (I know that's not what you were doing).. The funny part is that I think there is a real similarity to racism, now there is such a stigma around racism that I don't even feel comfortable drawing that analogy but maybe that's more reason to do so. I NEVER feel the need to point out that I'm white, I believe one day we'll live in a world when no one feels the need to point out they are hetro. Imagine these days someone asking what it "feels like" to be black, as if it is "different" from being a normal human. Unfortunately there are stsill many people who feel this way about homosexuals, but I also believe they are a dying breed. Vespine (talk) 00:03, 29 July 2010 (UTC)[reply]
One scientific paper which I read suggested the idea that there are many (epi)genetic factors which contribute to the "feminine" (gentle?) traits of the male and that the ownership of some of these factors make their benefactor a better mate by ensuring an appropriate response to their fragile partner and offspring. Since our reproductive systems rely on random homologous recombination, it possible that in some cases, an individual inherits more or less of these characteristics than average and as a result become overly masculine or overly feminine for optimum reproductive value. On top of this however, you have the confounding environmental (including cultural) aspects already alluded to above. --Seans Potato Business 23:00, 28 July 2010 (UTC)[reply]

A 100 W light bulb emits light with a wavelength of 575 nm

How many photons leave the filament of the globe in 1 second? Basically I have no idea how to do the question. I can convert the wavelength to frequency and then I can convert that to energy if that helps --220.253.172.214 (talk) 10:10, 26 July 2010 (UTC)[reply]

Since it is a filament lamp the filament emits a range of wavelengths like a black body i.e. not only at 575 nm, though that may be the strongest radiation. Have a look at the referenced article. Cuddlyable3 (talk) 10:25, 26 July 2010 (UTC)[reply]
I think this is just a homework question ;) You're right that the situation is not physically realistic, but the same goes for many such problems. Physchim62 (talk) 10:35, 26 July 2010 (UTC)[reply]
Hint: How many Joules does a 100 W light emit in 1 second? Dragons flight (talk) 10:29, 26 July 2010 (UTC)[reply]
100 Joules. Are you suggesting that I need to make the calculation ?--220.253.172.214 (talk) 10:52, 26 July 2010 (UTC)[reply]
Don't leave out the units! Besides being a requirement for physics calculations, making sure the units match is an excellent way of checking that your calculations are right. (In many cases if you don't know the formula, simply making the units correct will automatically give you the correct formula. This doesn't always work, but it works often enough to be a good first try for an unknown calculation.) Ariel. (talk) 13:47, 26 July 2010 (UTC)[reply]
Even bigger hint: what is the definition of power (which is measured in watts)? You calculation of the energy per photon is correct. Physchim62 (talk) 10:35, 26 July 2010 (UTC)[reply]

Bone Weight

What is the collective weight of all the bones (bones only) in an average human adult.  Jon Ascton  (talk) 13:19, 26 July 2010 (UTC)[reply]

There's some basic figures at the bottom of this page [1] , other sources quote a figure of 10-20% of total weight (presumably for a non obese person).This [2] gives 13% of body weight. 87.102.43.171 (talk) 13:39, 26 July 2010 (UTC)[reply]
Living human, or dry bones? (Not that I know the answer, but there is a large weight difference between the two.) Ariel. (talk) 13:49, 26 July 2010 (UTC)[reply]
Actually, I read somewhere today that it is about 12 pounds. (10 for females). I doubt it.  Jon Ascton  (talk) 14:09, 26 July 2010 (UTC)[reply]

Are strings matter?

I'm talking about the objects that make up quarks, not the things you use to tie your shoe with. Since they are 1 dimensional structures, it made me think if they are really matter or energy. Of course I'm aware that matter and energy are related (E=mc2) but, is it fair to say that the line is kind of blurred at that level? 148.168.127.10 (talk) 14:04, 26 July 2010 (UTC)[reply]

First of all, it is important to note that string theory doesn't have any significant evidence in favour of it (so it's a bit of a stretch to even call it a theory, it's just an idea). Assuming string theory is correct, then it doesn't really make sense to distinguish between matter and energy at the scale of individual strings. Photons are made out of strings in the same way quarks are and there isn't really any difference between those strings other than the way they vibrate. --Tango (talk) 14:35, 26 July 2010 (UTC)[reply]
What determines wether the particle has mass is the vibrational pattern of the string, not the string having mass itself, for example a photon is massless because it has a different vibration than quarks do even though they are both 1 dimensional vibrating strings according to string theory.--74.67.89.61 (talk) 14:50, 26 July 2010 (UTC)[reply]
Just to comment on the distinction between matter and energy: One shouldn't say that "something is energy" as if energy were something material or substantial; it is not. The correct way to put it is "Matter has energy", and the (misnomed) equivalence between matter and energy means that matter has energy even when it is at rest. So photons and strings will have energy, but they are not energy. Whether you want to call photons and strings "matter" depends on whether you want matter to be made of particles with mass or whether you accept massless particles, too. --Wrongfilter (talk) 10:02, 27 July 2010 (UTC)[reply]
So then what is energy? 148.168.127.10 (talk) 12:15, 27 July 2010 (UTC)[reply]
See energy for a long discussion of that very question. --Mr.98 (talk) 12:37, 27 July 2010 (UTC)[reply]
Oh sure, I know about that. I just meant within the context of what he was saying. I suppose we can't call any object energy then. Photon is not matter or energy, but it has energy. What about the four fundamental forces? Can we call any of those energy? 148.168.127.10 (talk) 15:53, 27 July 2010 (UTC)[reply]
Actually photons ARE energy. So is mass - mass does not contain energy - it is energy (and energy is mass, they are not just interconvertable they are two words for the same thing, but each has a different emphasis). The forces are not energy, but they do transmit energy, so while in transit they could contain energy. Ariel. (talk) 01:25, 28 July 2010 (UTC)[reply]
If they were the same thing, then we could simply throw out one of the terms and only use the other; much more economical. No, conceptually, mass and energy are very different things. Mass (that is rest mass) is an intrinsic, unchangeable property of a material particle; as such, mass is independent of the frame of reference. Energy is a quantity that describes the state (of motion, for instance) of a particle or physical system. It is a mathematical function that has properties which make it extremely useful to describe the state and behaviour of real physical systems; and energy depends on which frame of reference you choose. --Wrongfilter (talk) 09:41, 29 July 2010 (UTC)[reply]
That is incorrect. Mass also depends on the frame of reference, and energy might be invariant. For example chemical energy, or potential nuclear binding energy (the energy that the sun releases) are both invariant, and they show up as mass of course, even though they are "energy". Temperature and electric charge are also invariant (yes electric charge is energy - potential energy). Actually the only that is not invariable is kinetic energy. As you drill down you will find there is no intrinsic mass - all the mass is actually the result of potential energy of one kind or another. There is a reason that particles are weighed in electron volts (supposedly a measure of energy) instead of grams. Mass and energy are the exact same thing, and the only reason we use two words is historical. And now that we have two words we give them different emphasis. Ariel. (talk) 16:04, 29 July 2010 (UTC)[reply]
We could probably go on for a long time, so this'll be my last comment. Mass: Note that I referred to rest mass as the intrinsic property of a particle. Rest mass is the invariant length of four-momentum (the time component of which is energy - making it very clear from a mathematical point of view that energy and mass are different things). Temperature: When measuring temperature one doesn't take into account centre-of-mass motion. This is of course irrelevant because and entropy doesn't depend on bulk motion (and vice versa). Chemical/binding energy: You seem to suggest that I dispute the validity of E=mc2 - of course I am not. But that equation doesn't imply that the two are identical concepts. I would like to see a reference for your claim that "electric charge is energy". I would also like to see a reference for the claim that "mass is ... the result of potential energy" - have you solved the problem of the origin of mass? --Wrongfilter (talk) 17:53, 29 July 2010 (UTC)[reply]

soundwaves

Is it possible to view a soundwave in 3D? So for example would it be possible to develop a software that shows a soundwave from front on? —Preceding unsigned comment added by 91.103.185.230 (talk) 15:51, 26 July 2010 (UTC)[reply]

You could have software that shows 2d sections through a soundwave. More complicated solutions could include using transparent colour or exaggerated refractive index to visualise the soundwave. However note that high frequency sounds have short wave lengths eg 3.3kHz has only a 10cm wavelength. The problem with 3d visualisation of room full of soundwaves is that it would be looking through fog - also from front on you would be looking through pressure wave peaks and troughs - as the sound wave propagates itself there would be very little or no difference over time in the appearance (from front on). 87.102.43.171 (talk) 16:28, 26 July 2010 (UTC)[reply]
Showing 'isobars' of soundpressure level would be a lot easier - again using transparent (coloured) surfaces to visualise the sound level at different positions in space.87.102.43.171 (talk) 16:31, 26 July 2010 (UTC)[reply]
We regularly encounter this problem during full wave equation modeling for seismic imaging. (I'm sure RADARs and SONARs could have similar visualizations, but their data is less commonly visualized in a full 3D sense - in seismic imaging, the final desired result is a visualization of the entirety of a 3D volume, including its interior). Because the 3D wavefront is, well, three-dimensional, trying to render it on a 2D screen means you have to makes some tradeoffs. Many options exist for displaying the actual wavefront - you can pick an isosurface (i.e., interpolate one surface such that each point (X,Y,Z) has the same acoustic amplitude or phase) and render it in 3D; you can render a cube, and use a unique user-interface to select planar cross-section views; you can render an animated movie that traverses through one or more axes. Or, you can process the results of the acoustic wave, and instead of visualizing the wavefront, you can image some derived parameter - e.g. reverse time migration, where the results are correlations of the acoustic wavefront with a time-reversed version of the recorded echos. You can visualize the 3D acoustic wavefront in the fourier domain, or some other transform domain, where its 3D characteristics collapse along one or more dimensions. Every one of these approaches comes with difficult tradeoffs that boil down to, ultimately, you can't completely represent a complicated 3D structure on a 2D viewing screen. We have some peripherally related articles, volume rendering, tomography, and so on, that explain the difficulty of imaging a 3D structure. Nimur (talk) 20:15, 26 July 2010 (UTC)[reply]

Thermodynamics what-if

You have a box made out of a material which is very rigid and has a low thermal expansion coefficient. It is filled to the brim with water at room temperature. Suddenly you thrust it out of the airlock of your spaceship into the vacuum of space which is a couple of degrees above absolute zero. Because the box material conducts heat, the heat energy of the water leaves through the box and the water starts to cool down. When the temperature of the water gets to the freezing point, since water is one of the different materials that is less dense as a solid than it is as a liquid, the water begins to be under pressure as it pushes on the box. Say the box held rigid. It seems to me this pressure would have an effect of increasing the water temperature so it would go back to liquid. So would the water in the box in that frigid environment stay liquid or form some denser-than-normal ice? 20.137.18.50 (talk) 16:35, 26 July 2010 (UTC)[reply]

It would stay liquid for longer, i.e. at a lower temperature, but eventually it would still freeze. Keep in mind that the lower density of ice is only around the freezing point. Eventually it starts contracting like other materials, this article suggests that that temperature is 200K (about -70c) - see the graph. But keep in mind this graph was made at ambient pressure. Inside your box the pressure will be much higher which will influence things. Ariel. (talk) 16:48, 26 July 2010 (UTC)[reply]
Freezing point doesn't vary much with pressure (unlike boiling point), so it wouldn't stay liquid at a significantly lower temperature. --Tango (talk) 17:02, 26 July 2010 (UTC)[reply]
If you increase the pressure on a gas, it warms up, but I'm not sure the same is true of a liquid. Liquids are pretty much incompressible, so I don't think they would change temperature much. Regardless, as Ariel says, the water would eventually cool to the point where ice is denser than liquid water, at which point it would definitely freeze. --Tango (talk) 17:02, 26 July 2010 (UTC)[reply]
At equilibrium, the pressure inside the box will be lower than when you started, because the density of (normal) ice is higher than that of liquid water at low temperatures. While the water is cooling down, the pressure will be very much higher (but I'm assuming you want us to consider an infinitely rigid box). Physchim62 (talk) 17:15, 26 July 2010 (UTC)[reply]
I think I need to slightly correct what Tango said above - freezing point does change with pressure for materials like water that change volume on freezing. Since water initially expands on freezing increased pressure lowers the freezing point (described in general by Le Chatelier's principle) .. Since both water and ice are very incompressible even a small amount of (normal) ice formation would cause a very massive increase in pressure.
There are denser forms of ice - I don't know if the conditions here would be sufficient to cause its formation over normal ice - though I would guess it's likely.87.102.43.171 (talk) 17:34, 26 July 2010 (UTC)[reply]
Generic phase diagram - the dashed green line shows water, the solid green line shows typical substances.
This phase diagram seems to show that the freezing point of water changes less with changes of pressure than typical substances (although water does at least change in the right direction). Both the solid and dashed green lines are pretty close to vertical, so the change is pretty small. --Tango (talk) 18:08, 26 July 2010 (UTC)[reply]
water expands about 10% on freezing, the bulk modulus of ice is ~1010Pa - so to compress ice to the same volume of water would require about 1000MPa of pressure. In fact this is beyond the pressure for which normal ice exists - so one of the more dense forms of ice is formed. There's a similar discussion (from which I got the compressability data from) here [3]
The same link references this page [4] which has a phase diagram which extends beyond that given right - It looks like ice V or VI would be formed at 0C at these pressures. However if the cooling was rapidly to 0K and not 0C all this is not relavent.87.102.43.171 (talk) 19:34, 26 July 2010 (UTC)[reply]
I got a very rough figure of 1degree change in freezing point per MPa (ie 0.1 degree C per bar/atmosphere) - no idea how accurate that is.87.102.43.171 (talk) 20:44, 26 July 2010 (UTC)[reply]
That will only be accurate near a specific temperature (probably 0 deg C). It can't be accurate everywhere, otherwise the freezing point at 300MPa would be below absolute zero, which is obviously nonsense. --Tango (talk) 21:39, 26 July 2010 (UTC)[reply]
You are essentially asking about freezing water at constant density (i.e. by not allowing the water expand as it freezes). Assuming an sufficiently rigid container to counter the large pressures involved, I would expect to see a combination of normal ice and one of the high density ices such that the density of the mixture stayed constant. Dragons flight (talk) 18:28, 26 July 2010 (UTC)[reply]
Right. Note that below about 250K there is no stable liquid form at any pressure, so the result is bound to be ice of some type. Looie496 (talk) 18:34, 26 July 2010 (UTC)[reply]
Looking into this further, what would happen, I believe, is that the water would remain liquid as it cooled down to 250K, with the pressure rising steadily to over 200 million atmospheres. Then it would freeze into a mixture of ordinary ice and altered phases of ice. As it continued to cool, it would evolve through several combinations of phases, with the pressure slowly dropping but remaining in the hundreds of millions of atmospheres. As it approached absolute zero, you would end with a mixture of about two parts ice XI to one part ice IX -- assuming that the container was totally filled with water at the start and totally nondeformable. Looie496 (talk) 19:55, 26 July 2010 (UTC)[reply]
I think that is almost but not quite right - ice can form at -1C and the pressure increase. At 20% normal ice (at -1C) the pressure would be 200MPa and then other forms of ice could form.the figures in the last sentence may be quite far out Nevertheless there would definately be normal ice in the box below 0C below 200MPa (above 250K) 87.102.43.171 (talk) 20:07, 26 July 2010 (UTC)[reply]
I think you meant 200million pascal, not atmospheres87.102.43.171 (talk) 22:15, 26 July 2010 (UTC)[reply]

Human magnetoception

What is the current status of human magnetoception research? The magnetoception article lists research from 2007.[5] Is it bunk or is there something to it? Viriditas (talk) 16:37, 26 July 2010 (UTC)[reply]

The article Magnetic resonance imaging contains an uncited note that Volunteers report a twitching sensation when exposed to rapidly switched fields, particularly in their extremities. Cuddlyable3 (talk) 17:00, 26 July 2010 (UTC)[reply]
The scholarly consensus (e. g., PMID 20504748) is still that humans can't detect the Earth's magnetic field, the cited article notwithstanding. MRI involves fields stronger by many orders of magnitude, so a moderate sensitivity to them wouldn't mean all that much. Looie496 (talk) 18:17, 26 July 2010 (UTC)[reply]
A human would be able to magnetically detect a magnetar from a good long distance off, since at 1000km away, the magnetic field is lethal. It's not strictly relevant; I just mention this fact whenever I can. Paul (Stansifer) 03:00, 27 July 2010 (UTC)[reply]

Westons Mill Pond is a dammed section of the Lawrence Brook in New Jersey. Would it be included in WP:RIVERS? --Chemicalinterest (talk) 17:28, 26 July 2010 (UTC)[reply]

There is also Wikipedia:WikiProject Lakes. Rmhermen (talk) 19:17, 26 July 2010 (UTC)[reply]
You could ask at Wikipedia_talk:WikiProject_Rivers. Falconusp t c 04:21, 27 July 2010 (UTC)[reply]

culturing cells on conductive cover slips

I think my group is using conductive indium tin oxide cover slips for electric field experiments. However, my group doesn't know what happens if you culture epithelial cells on it -- any predictions? Would poly-L-lysine coating -- used for helping cells adhere -- ruin the conductive properties, or no? John Riemann Soong (talk) 18:53, 26 July 2010 (UTC)[reply]

Google suggests that others have managed it http://www.google.co.uk/search?sourceid=chrome&ie=UTF-8&q=polylysine+indium+tin+oxide eg [6] (at least compatable) , here google books (must be still conductive) seems to suggest that a polylysine coated indium tin oxide film was successfully used in a reduction (hence still conductive)
The issue I can see would be if polylysine (basic) reacted with the indium tin oxide - if the indium tin oxide film is very thin (ie 1 atom) you could end up stripping that film by complexation with the polyamine... However I imagine the film will be much thicker.
I have no idea about culturing cells on indium tin oxide - if indium or tin is toxic to your cells then that could be a problem - beyond that no idea. It's not clear why a conductive glass slide is needed - wouldn't anodised aluminium be just as good for culturing cells and doing field experiments?87.102.43.171 (talk) 21:28, 26 July 2010 (UTC)[reply]
Well it's what my group has. Is anodised aluminum see-through? :S I'll probably try using these cover slips tomorrow. John Riemann Soong (talk) 22:33, 26 July 2010 (UTC)[reply]

Plant ID request

We saw this plant at Disneyland and was wondering what it is. Any help from an editor out there? Thanks, Alanraywiki (talk) 19:50, 26 July 2010 (UTC)[reply]

Impatiens, possibly. Maybe one of the New Guinea impatiens cultivars, see Impatiens hawkeri --Dr Dima (talk) 20:38, 26 July 2010 (UTC)[reply]
Thank you so much. I feel a little dumb because we have some impatiens in our yard, but this looked a little different to us. I appreciate your time in answering. Alanraywiki (talk) 22:00, 26 July 2010 (UTC)[reply]
Please place all further responses to this question on Wikipedia:Reference_Desk/Miscellaneous#Plant_ID_request, where this question was also posted. Falconusp t c 04:30, 27 July 2010 (UTC)[reply]

Help me identify these intercellular bridges, follow-up?

I've put up some of my images at User:John Riemann Soong/intercellular bridges. I'm trying to puzzle out what kind of intercellular bridges these are. These bridges are between A549 lung cancer cells. Help is appreciated!

for background, most of the white things that don't appear to change appearance with rotations are vesicles, not gold nanorods. Particles that go black-white-black or "wobble" are gold nanorods. I excluded most of the video of course as they are much too big, but I can upload huge (~30 MB) animated gifs if necessary. A brief description of what happened on each day is on the page I linked to.

I've been reading this paper to try to help me identify these bridges. Do any of them consist of membrane nanotubes at least internally? I don't think I have any "tunneling nanotubes" right? Am I observing type I epithelial bridges, or just mere filopodial bridges? John Riemann Soong (talk) 23:52, 26 July 2010 (UTC)[reply]


July 27

Can a gas shield the Earth?

Can we release a gas which will shield the Earth from sun rays and reduce global warming? (That would be the equivalent of CO2 but reflecting heat instead of absorbing.)--Mr.K. (talk) 11:17, 27 July 2010 (UTC)[reply]

Solar radiation management discusses several schemes. -- Finlay McWalterTalk 11:20, 27 July 2010 (UTC)[reply]
The sun emits most of its energy in the visible range of the spectrum. I don't know of too many gases that significantly reflect in the visible, are stable in the atmosphere, and are mostly harmless to the ecosystem. Also, of course, it would seem to be more rational to do less meddling with a large, complex, critical system, rather than more. --Stephan Schulz (talk) 11:34, 27 July 2010 (UTC)[reply]
(Actually, you have it wrong - the whole problem with CO2 is that it DOES "reflect heat" - thereby trapping the heat in our atmosphere instead of letting it radiate harmlessly out into space!)
The tricky part is that you need to find something that's able to reflect visible and UV light - but be transparent to infrared. That's because it would have to bounce away the frequencies of light where the sun is injecting energy into the planet - but not prevent the planet from radiating away waste heat as infrared light. The Greenhouse gasses like CO2 that are causing all of the problems are quite the opposite - they are transparent to visible light (letting sunlight into the planetary atmosphere) and reflect infrared (preventing waste heat from escaping).
So the material you're looking for wouldn't be a transparent substance like most gasses - because those aren't reflecting visible light - they are transmitting it. The material you're imagining would have to be bright white to look at - yet transparent to IR light. Clouds are somewhat like that - but they are tricky to manage. Water vapor is a greenhouse gas, like CO2 - but when the water droplets are just the right size and temperature to form clouds, they become opaque and reflect away sunlight. But it's tough to control water vapor - to have it form clouds and reflect light without becoming a greenhouse gas.
You'd also need something that was very cheap and non-damaging to manufacture (if, for example, it took a lot of energy to make - it might cause more problems than it solved). You need an ungodly amount of 'stuff' to fill all of that atmosphere densely enough. It would also have to not damage ozone, not be poisonous, etc. Ideally, you'd want something that would break down naturally - at just the right rate so that we could control it's effects - you wouldn't want something where some small misunderstanding of the Earth's environment caused us to use too much of the stuff and plunge the planet into a massive cooling spell. Overall, this kind of approach is very dangerous...it's hard to imagine that doing this would be sufficiently non-risky to be acceptable. The politics of doing this would be very tough to negotiate too...suppose one country decided to do this kind of risky planetary engineering without the agreement of all of the other countries on Earth?
SteveBaker (talk) 11:56, 27 July 2010 (UTC)[reply]
The earth has natural cycles of heat and cold, and our industry only makes a small difference to the temperature of the earth. Yes, the risks and expenses of making such a system would make it completely impractical. --Chemicalinterest (talk) 12:02, 27 July 2010 (UTC)[reply]
Let's not derail this into a discussion about whether climate change is anthropogenic, please. --Mr.98 (talk) 12:35, 27 July 2010 (UTC)[reply]
In any case, if the earth is only 7000? years old it's questionable how we can know whether the earth has natural cycles or not and these natural cycles must be so short that we better be bloody scared. Nil Einne (talk) 21:31, 27 July 2010 (UTC)[reply]
No. There was a warmth period (global warming) in the time of the Vikings, which is why they moved to North America. They left when it started cooling, making it too cold for them to survive profitably. It was still cold throughout history until recently, when it started warming again. During that cold spell, the early immigrants to the US (the Pilgrims) had a hard time being established here because of the cold. --Chemicalinterest (talk) 13:18, 28 July 2010 (UTC)[reply]
See Medieval warm period and little ice age, both of which are fairly insignificant compared to the observed and projected warming of the 20th and 21st century, especially the MWP which was cooler than year 1900 depending on the proxy source. ~AH1(TCU) 01:52, 2 August 2010 (UTC)[reply]
Clouds reflect a lot of visible light, but don't they also reflect IR? That's why it's generally warmer at night if there is cloud cover. --Tango (talk) 14:19, 27 July 2010 (UTC)[reply]
If you have an infrared thermometer you can actually measure this. If on a day with some clouds in the sky you point your thermometer in the direction of clear sky you get a very low reading (like -50 °C), if you point it toward a cloud you get a much higher reading (e.g. 0 °C). Count Iblis (talk) 15:38, 27 July 2010 (UTC)[reply]
Isn't that detecting the thermal radiation from the cloud, rather than reflected thermal radiation from the ground? --Tango (talk) 21:59, 27 July 2010 (UTC)[reply]
Why not paint large areas of the Earth's surface white? Count Iblis (talk) 15:16, 27 July 2010 (UTC)[reply]
Direct modification of the Earth's albedo is discussed in the article on solar radiation management, linked in the first answer above. TenOfAllTrades(talk) 15:19, 27 July 2010 (UTC)[reply]
Thanks! Count Iblis (talk) 15:38, 27 July 2010 (UTC)[reply]
There is a big misconception circling around here - that gases "reflect" visible light or IR radiation. They do not - gases can scatter and absorb, but there is not mirror-like reflection unless there is a flat surface of some sort (which gases don't have). Gas molecules with polar bonds (e.g. C=O, C-H, O-H, C-F, etc.) tend to be good at absorption of long-wave IR radiation (The same gases also emit a lot when warm) which makes them greenhouse gases. An "anti-greenhouse" gas would need to be poor at absorbing long-wave IR (so it must not contain such polar bonds) and good at scattering or absorbing visible light. However, the common gases with nonpolar bonds such as N2 and O2 are transparent. Gases without polar bonds can be colored/non-transparent such as Cl2, but it is toxic, harmful and unstable so clearly we shouldn't be pumping it in massive quantities into our atmosphere. I am not aware of any gases which i) have no polar bonds, ii) are not transparent to visible or short-wave-IR, and iii) Could be released in massive quantities necessary to achieve a substantial cooling effect without disastrous side effects. However, if you remove the restriction to gases, sulfate aerosols might work (see Geoengineering.) 129.2.46.178 (talk) 01:02, 28 July 2010 (UTC)Nightvid[reply]
Some people think that there may be a way to protect the earth via a gas (See this article). Basically, a relatively small amountof SO2 in the stratosphere could 'reflect' some of the light that would have entered the atmosphere. Not sure if it really reflects (see above regarding gases absorbing or scattering light), but it is possible that it would exist as a fine particulate, maybe sulfates as mentioned above, that would reflect. As for cheap sources, there is quite literally tons of sulfur sitting in the Alberta tar sands that some people would love to find a use for...24.150.18.30 (talk) 02:50, 28 July 2010 (UTC)[reply]
A recent article in New Scientist argued that reduction of the UK's power generation and industrial emissions of Sulpher dioxide (by using 'cleaner' fuels) to lessen the acid rain impact on Scandinavia had, ironically, had little impact on the acid rain (because the main sources of Scandinavian SO2 were actually Continental), but had increased the UK's local warming by decreasing SO2 reflection of sunlight. 87.81.230.195 (talk) 10:16, 28 July 2010 (UTC)[reply]
Direct injection of sulphur dioxide into the stratosphere is one of the possibilities discussed in geoengineering. However it would be a problem to remove all those trillions of microscopic particles should something go wrong, and when there are removed the temperature on Earth could skyrocket by 7°C. Also take a look at carbon dioxide air capture, water vapor feedback and Iris hypothesis. ~AH1(TCU) 01:52, 2 August 2010 (UTC)[reply]

There is not enough darkness in all the world to put out the light of even one small candle

Is this true?

Namely, if you had a small unobtrusive clear container of STP air in a remote and (perfectly?) dark region of space with a small wax candle burning inside, would black-body radiation drop the temperature low enough to extinguish the flame? -Craig Pemberton 18:16, 27 July 2010 (UTC)[reply]

I assume you're thinking of something like the fact that water left outside on a clear night can freeze even if the air temperature is above freezing, because it loses heat to space, and air (not being very efficient at radiating heat) does not radiate enough heat back to the water to compensate. Whereas if the night is cloudy, it won't happen, because the clouds will radiate enough heat to keep the water liquid.
I am quite sure that effect cannot put out a candle flame. The limit of the effect is the heat that would otherwise be transferred by radiation from the air to the candle wick, if air followed the black-body law. But that's a pretty trivial amount of heat compared to what a candle generates. --Trovatore (talk) 18:23, 27 July 2010 (UTC)[reply]
Let's try some back-of-the-envelope calculations and see what they reveal. Let's assume that the container is a cube 10cm on each side, that the space is at absolute zero (actually, cosmic microwave background radiation makes it about 3K, but that's close enough to zero), that the container is a perfect black body and that the candle continues to have enough oxygen to burn by magic. The relevant law is the Stefan–Boltzmann law, which states that the power lost is , where A is surface area, T is temperature and . According to candle, a typical candle emits about 40W. We can now rearrange the Stefan-Boltzmann law and find the equilibrium temperature. . The candle will have no problem burning with the container at 56C (the air right next to the candle will be hotter). If we increase the size of the container, then the equilibrium temperature will be lower, for example a cube 1m on each side would have a temperature of -170C, which is probably too low for the candle to keep going (the air in the centre would be much hotter, but the outside would be very close to the point where oxygen condenses, which would cause problems). --Tango (talk) 18:47, 27 July 2010 (UTC)[reply]
As a minor aside, I will note that bringing liquid oxygen together with a combustible material in the presence of a spark or other ignition source is actually a very effective way of making a very impressive fire. While I have not tested this personally – and I would discourage any but the most qualified from making the attempt – I strongly suspect that dropping a lit candle into liquid oxygen would be very...exothermic...indeed. Making interesting reading (and viewing) are the numerous accounts of people lighting charcoal barbecues using a lit cigarette, a pile of charcoal, and a bucket of liquid oxygen — attached to a very long pole. TenOfAllTrades(talk) 22:03, 27 July 2010 (UTC)[reply]
(ec)A candle can burn as hot as 1930K - i.e. a temperature rise of 1637K, even if you froze the whole thing to absolute zero the candle would still release enough energy to burn (at least once you managed to ignite it). Ariel. (talk) 18:50, 27 July 2010 (UTC)[reply]
I quite like Tango's analysis above, but I fear that once the box gets large (and the corresponding mass and volume of air around the candle become significant) we will no longer be able to approximate the system as reaching a uniform air temperature. Instead, we will have a gradient of air temperatures ranging from 'quite hot' adjacent to the candle flame down to 'very cold' at the walls of the box. Assuming the box is under gravity then convection currents will also play a role. (If the box is not under gravitational or other acceleration, then we might have problems with the candle depleting the oxygen around itself and going out for that reason.) Conductive and convective heat transfer will become increasingly important in larger boxes. TenOfAllTrades(talk) 22:03, 27 July 2010 (UTC)[reply]
Indeed, my calculations are for the container, rather than the air. The air will have a temperature gradient, from the temperatures I gave at the edge to thousands of degrees at the candle itself. The exact details of that gradient are beyond my ability to calculate. --Tango (talk) 22:13, 27 July 2010 (UTC)[reply]
Per Fourier's law, the gradient will tend towards a linear isotropic temperature falloff after a long period of time. In other words, dT/dr = constant, where r is the vector from a test point to the candle heat source. In reality, two factors confound this calculation; there is not spherical symmetry (if the box is a "cube"), so depending on whether we have "imaginary nonconductive walls" or some walls with real thermal characteristics, that will break the linear isotropic assumption. We could call the walls "very conductive" and therefore at constant temperature at all locations along the wall, in which case they would serve as a boundary condition for the heat flux equation. Or, we could call them "very non-conductive" (much less conductive than air), in which case they would simply truncate an otherwise spherically symmetric solution. The realistic case, where the conductivity is "comparable" but non-equal to that of air, would be a complicated boundary value problem. The second problem is that air will convect and turbulence will exist; if the candle burns for an "indefinite period of time", we can assume a steady-state will be reached eventually, but it may be a very long time before that is the case. Tiny fluctuations in initial conditions of the momentum and angular momentum of each air particle will persist in a very unpredictable way. I agree with Tango, any realistic solution for the air temperature that does not rely on trivial-solution assumptions are extremely difficult. (Not surprisingly, these are the same calculations used to determine planetary energy-balance and surface-temperature equations in planetary science. The results can provide bounds on material composition, internal seismicity or radiogenic heat, and so on. There is good coverage in de Pater and Lissauer's Planetary Science text). The scenario described above is a lot like a miniature "hot gas planet" with an internal heat source. Nimur (talk) 02:34, 28 July 2010 (UTC)[reply]
Point of correction: Fourier's Law implies that and is constant in a spherical geometry. Dragons flight (talk) 03:01, 28 July 2010 (UTC)[reply]
Ah, yes, I forgot about the conductivity of the container. I made an implicit assumption that it was highly conductive, which I should have included in my list of assumptions at the beginning. Thanks! --Tango (talk) 13:05, 28 July 2010 (UTC)[reply]

Volume of blood in the human body

On average, how many cm3 of blood are there in the human body? --138.110.206.99 (talk) 19:28, 27 July 2010 (UTC)[reply]

Humans typically have between 5 and 7 liters of blood, so 6,000cc would be a good estimate. Googlemeister (talk) 19:36, 27 July 2010 (UTC)[reply]
Blood gives a typical volume of 5 liters, which is in sync with the 4.7–5.7 L range given in circulatory system. Of course none of this is specifically cited. Anyone with a medical text or similar ref handy? DMacks (talk) 19:41, 27 July 2010 (UTC)[reply]
10-12 pints. DRosenbach (Talk | Contribs) 22:46, 27 July 2010 (UTC)[reply]
(Which is, roughly speaking, the same as 5-6 liters). TenOfAllTrades(talk) 23:07, 27 July 2010 (UTC)[reply]
Yes, that would be correct, Ten. DRosenbach (Talk | Contribs) 23:36, 27 July 2010 (UTC)[reply]
"Ten" in this case does not refer to the question, but the username of the user whom the above user responded to, TenOfAllTrades. ~AH1(TCU) 01:44, 2 August 2010 (UTC)[reply]

Bermuda Triangle

Did we find out what really happened in this triangle?75.73.152.238 (talk) 22:40, 27 July 2010 (UTC)[reply]

Yes. The answer is: Nothing special. Our Bermuda Triangle article has details. Comet Tuttle (talk) 22:41, 27 July 2010 (UTC)[reply]

How much paneer cheese would I get per litre of milk?

Having read the paneer article I'm intrigued by the idea of making some myself, rather than buying salt-laden fetta cheese from the supermarket. What weight of paneer cheese would I get from a litre of milk please? Is it possible to make low-fat paneer cheese by using skimmed milk? Thanks 92.29.116.34 (talk) 23:19, 27 July 2010 (UTC)[reply]

It is not specific to paneer, but this website [7] describes turning 5 gallons (roughly 40 pounds) of milk into 6 pounds of cheese. So, using that ratio, 1 liter of milk should make about 150 grams of cheese. Dragons flight (talk) 23:42, 27 July 2010 (UTC)[reply]
I think it can vary a lot though by type of cheese, softer cheeses requiring less milk per final product. Rckrone (talk) 04:31, 28 July 2010 (UTC)[reply]
Softer cheeses contain more whey, so they have greater yields. Cheddared cheeses are the opposite. Regardless, the total cheese and whey combined that is produced should be roughly invariant. Whey can be used in the place of water in a lot of cooking and is nutritious. -Craig Pemberton 07:13, 28 July 2010 (UTC)[reply]
I've looked into and made paneer before. Skimmed milk usually isn't recommended because the paneer is said to be grainy and rubbery (haven't tried it myself) but I've seen it suggested half skim and half full works okay. I've used reconsituted (full cream) milk powder myself because it's cheaper and I don't think it will make that much of a difference to the taste or texture but I haven't actually tested it to see. I can't remember the yields but the above sounds roughly write, in any case if you use milkpowder the amount isn't going to be that different from the solids you put in, 1 kg of full cream milk powder yields 8 litre IIRC which means 125g for 1 litre which is about what Craig Pemberton found in the ref above. Nil Einne (talk) 09:43, 28 July 2010 (UTC)[reply]
Do you need non-homogenized milk to make paneer? Googlemeister (talk) 15:23, 28 July 2010 (UTC)[reply]

Thanks. What about filtering it - where could I get some muslin from in the High Street? Or is something else suitable? The cost of full-fat milk means it would be more expensive than shop-bought cheese - is there anything I could add to it to stretch or bulk it? I do not want to use dried milk. 92.29.121.86 (talk) 12:29, 29 July 2010 (UTC)[reply]

July 28

Phlebotomy Laws

Where would I look things up about Canadian Phlebotomy Laws. What about patient consent ?. Can the Dr. make me take blood fr om a patient, consenting? , or non-consenting? —Preceding unsigned comment added by Dean J Taylor (talkcontribs) 00:10, 28 July 2010 (UTC)[reply]

I don't know about Canadian law specifically, but I'm pretty sure it's the same as other similar legal systems in that sticking a needle in someone without their consent is an assault (or, maybe, a battery, depending on the terminology used in the jurisdiction in question). There are exceptions made for people trying to help someone who cannot consent (eg. they are unconscious, insane, a child, etc.). The exact details of those exceptions will depend on the jurisdiction and, as I say, I'm not familiar with Canadian law. --Tango (talk) 00:15, 28 July 2010 (UTC)[reply]
This document specific to Canada appears to confirm that adults generally have the right to refuse any medical treatment, even if the anticipated consequence of that refusal is death. Dragons flight (talk) 01:10, 28 July 2010 (UTC)[reply]
Of course taking blood from a a non-consenting person (in normal circumstances) is unlawful but I'm not sure the OP is asking about that. The question seems to imply that 'the doctor' is the employer and that the OP may be coerced into doing something he/she doesn't want to do. If taking blood is part of your job description then you may be obliged to carry out that duty. If this is a new task that the doctor is imposing on you then negotiation is in order. Do you have access to a local union? I am not an expert and this is not legal advice. Richard Avery (talk) 07:21, 28 July 2010 (UTC)[reply]
There would probably be a few other cases where medical treatment isn't really involved but consent isn't required to take blood, like a blood alcohol test compulsory under law (in many countries a police may be able to order a driver to undergo some test to determine their blood alcohol content) or a court ordered blood test for some other reason. Also someone who 'can't' consent may also include a suicidal person in quite a few countries where someone who is attempting/has attempted to commit suicide may be subject to suicide intervention and considered unable to refuse medical treatment. As per Richard Avery, if you are uncertain about what circumstances you may be required to take blood without the patient having given consent, you should consult a union or some other such authority who can hopefully advise you. Nil Einne (talk) 10:14, 28 July 2010 (UTC)[reply]
Your job description cannot oblige you to carry out an illegal act, though. If you think what the doctor is telling you to do is illegal, then you are legally obliged to disobey. --Tango (talk) 13:09, 28 July 2010 (UTC)[reply]
As for the needle, what about cases where intravenous injection is required for an unconscious patient? ~AH1(TCU) 01:42, 2 August 2010 (UTC)[reply]

A question about Desmoplastic small round cell tumor question

is desmoplastic small round cell tumor in ewing sarcoma familiy of tumors? —Preceding unsigned comment added by 68.238.53.30 (talk) 00:58, 28 July 2010 (UTC)[reply]

From desmoplastic small round cell tumor: "Desmoplastic small round cell tumor shares characteristics with other small round cell cancers including Ewing's sarcoma, ...". Looie496 (talk) 01:48, 28 July 2010 (UTC)[reply]
I would interpret both the question and that response carefully. While both desmoplastic small round cell tumours and Ewing's sarcoma are examples of small blue round cell tumours (equivalently, just small round cell tumours), DSRCT and Ewing's are different members of that family. (Here's a free-access paper from Am. J. Clin. Pathol. describing an immunohistochemical approach to differentiating the two.) TenOfAllTrades(talk) 03:24, 28 July 2010 (UTC)[reply]
[I have modified the heading from "A question" to "Desmoplastic small round cell tumor question" to facilitate subject-matter searches. I believe this is in order, but apologies if I am in error.] 87.81.230.195 (talk) 10:21, 28 July 2010 (UTC)[reply]
It's good to do that, but you need to incorporate the words of the original heading when you do so. See Wikipedia:Reference desk/Guidelines#Don't_edit_others'_questions_or_answers Ariel. (talk) 16:20, 29 July 2010 (UTC)[reply]

Evolutionary adaption queuing problem

I’ve been thinking about a problem I see with current evolutionary theory (not that I’m a creationist). Sure, some genetic features for a given species in a given niche are better suited for survival and reproduction than other features. We constantly hear of examples of such features, like the giraffe’s neck. But in the case of the giraffe, and of all other species, each one has thousands of special adaptations, all of which presumably have been selected because of their efficiency. Now, my problem is that I can’t see how multiple features can be selected for at the same time, and yet I can’t see how they could be selected one at a time either. What if the member of some species has a beneficial trait and another member has a different beneficial trait? Now it would seem that if they are in competition, the traits themselves are in competition. But we never hear about this when the subject is being discussed. It’s always examples of beneficial traits discussed in isolation, as if the one being discussed is the only one at that time, in that species being propagated throughout their community. I’ve heard there is a name for this conundrum. Has it ever been discussed here? Eric155 (talk) 06:40, 28 July 2010 (UTC)[reply]

If you never hear of the traits evolving in competition and co-operation, then I can't think what advanced biology texts you're reading! Seriously though, that's pretty much the default way of thinking in modern evolutionary biology. The idea that traits evolve in isolation is responsible for people assuming irreducible complexity when they see things that must have evolved in tandem with other things. 86.164.66.83 (talk) 06:52, 28 July 2010 (UTC)[reply]
(ec) I'm not sure that there's necessarily a problem here. For simplicity's sake, let's assume that the original animal has traits a and b, each regulated by a single gene. It turns out that a mutation affecting a gives rise to dominant trait A, which confers a survival or reproductive advantage. An independent mutation in b gives rise to B, which also confers a survival advantage. The population now consists of a large number of individuals with the original genes ab, and a few with the beneficial new traits, Ab and aB. If A and B confer an advantage over a and b, the number of ab individuals will become depleted in the population as they are out-competed by the Ab and aB animals.
As you get more Ab and aB and fewer ab creatures, the likelihood is that there will be interbreeding between Ab and aB animals; some of whose offspring will now be AB. As the original ab animals disappear, the interesting competition becomes the one between the single-mutant Ab or aB animals, and the double-mutant AB animals. If AB in combination is more effective than (that is, confers a reproductive advantage over) the single mutants, it will gradually become more prevalent in the population. So I guess the short answer to your question is this — animals can acquire multiple beneficial traits because they interbreed, and because there is a pool of animals with a range of genotypes which survive each generation (it's not just one 'optimal' genotype which is cloned for all of the offspring of the next generation; there's significant mixing). TenOfAllTrades(talk) 07:04, 28 July 2010 (UTC)[reply]
I recommend Eric155 reads River Out of Eden by Richard Dawkins. This book is short in length and brilliantly suited to newcomers, like me, to the field of zoology and evolutionary biology. It is very readable and will answer Eric155's questions. Dolphin (t) 07:54, 28 July 2010 (UTC)[reply]
I have long had a similar problem to the OP's, and had been thinking of posing a similar question here along the lines:
given that in any population scores if not thousands of phenotypical traits (mostly at biochemical levels not evident to the eye) are presumably all evolving simultaneously, how is it that the advantageous and disadvantageous ones, proportionally represented in each individual of the population, do not cancel each other out? (To crudely oversimplify, a squonkbird won't pass on its advantageously longer wings if its unconnectedly and disadvantageously weaker heart leads to its death before it breeds.)
Intellectually I agree they can't, because I fully accept that evolution is proven to proceed as extensively described, but intuitively it's hard to visualise. Can the total sub-set with each disadvantageous trait, and ditto for the advantageous ones, be validly considered as evolving separately, with the successful outcomes re-merging? I imagine that there might be some simple mathematical demonstration that the numbers really do work out, and if so I'd like to be able to point to it even if (as is likely) I can't myself understand it. (Hey, I'm willing to trust a mathematician! :-) ) 87.81.230.195 (talk) 10:45, 28 July 2010 (UTC)[reply]
Mutations tend not to be helpful. What would, for example, a wolf (or any other animal) with am extra-long neck have to give it an advantage. Its neck would be extremely hard to hold up unless it has more developed muscles and a strong heart to pump it up, along with extralong tissues of all kinds that are found in the neck. It also has to adapt to leaning down to drink water. All of these helpful mutations have to occur all at once, or the neck is utterly useless (i.e. the animal is a "freak animal"). If you take the mathematical probability of all of those helpful mutations occuring all at once, it is very low, definitely not able to occur in millions or even billions of years. (I am he in disguise). --98.221.179.18 (talk) 10:51, 28 July 2010 (UTC)[reply]
Most mutations are not things like "extra long neck." They are small changes to physiology. They do not have to happen "all at once." If you're actually interested in learning about what evolutionary theory says — and not just the bad parody of it that is given by the ID people — there are plenty of better explanations out there that you seem to have either missed or not understood. I am also not sure you can conceptualize how long a span of time 1 million years is, much less a billion years. --Mr.98 (talk) 12:15, 28 July 2010 (UTC)[reply]
The mistake being made in these questions is one of the most common misunderstandings about evolution - you can't approach evolution from a atomistic, particularistic viewpoint. evolution works by small advantages and small changes to entire organisms, not big advantages and big changes to parts of organisms. Let's take giraffes for an example. There was (by the theory) some paleontological period extending over millions of years in which having a longer neck was an advantage-niche that a species could occupy. the actual advantage is unknown, and probably constituted several advantages - e.g. a longer neck allowed for better detection of predators, better access to certain kinds of food, a more threatening, dominant posture (for purposes of breeding and defense). whatever that primal species was it had a normal but small distribution of neck sizes (just the way humans have have a normal but small distribution of heights), and the animals with longer necks where slightly more likely to survive to pass on their genes - they could see predators sooner, had better nutrition and thus better health, were less likely to be attacked and more likely to gain access to mates. Further, those of that species which did not have the internal biological structures needed to support a longer neck (stronger circulatory systems, exaggerated musculatures, etc) could not utilize the environmental advantages of having a longer neck, and would not gain any of the advantages. The entire system (longer neck and internal support structures) was required to take advantage of the niche.
you need to remember that this has nothing to do with the particular feature. sure, giraffes have long necks, but giraffes did not evolve long necks because it was advantageous to do so; giraffes evolved in a way that was advantageous in the environment, and that just happened to involve long necks. in a slightly different environmental context they might have evolved with very long legs and short necks. There's a strong tendency because of the way we think for us to use teleological reasoning - asserting that the effect we see in the world must have a specific cause that brought it about - when in fact this is less a specific effect than a by-product of other forces. --Ludwigs2 11:55, 28 July 2010 (UTC)[reply]
Increasingly tangential and heated discussion, collapsed by Brammers (talk/c) 17:25, 28 July 2010 (UTC)[reply]
wwwwait! TenOfAllTrades you said about the mutations of genes. mutations ,which are very rare, are 99.9% of the time failures and make the animal, insect,etc. worse. Some are even deadly. And that statement is proven by experimenting; an example of mutation is two-headed snakes and albino squirrels which, as you obviously see, isn't better than the original creature. so, sorry, mutation doesn't make things better.--Horseluv10 12:05, 28 July 2010 (UTC) —Preceding unsigned comment added by Horseluv10 (talkcontribs)
"99.9%" is just a made up, fake statistic, along with your false "experimenting" snake example. It's true that there are a lot of non-beneficial mutations, but that's not the problem. Mutation rates just provide a background rate of variability for which natural selection can then act upon. Most mutations are small physiological changes that can add up over time. There is a lot of real scientific work on this, with precise understandings of gene flow and mutation rates and the mechanism of mutation and so on — not just hand-waving bad Creationism. If you want to have a conversation about how this really works and what the scientific arguments really are, that's fine, but this is not a general discussion forum where ignorant trolling (from any side) is really tolerated. --Mr.98 (talk) 12:15, 28 July 2010 (UTC)[reply]
Horseluv10 appears to be using the notion that mutations must be either good or bad - no random mixture is contemplated. Horseluv10 has cited examples like two-headed snakes and concluded Voila, mutations are bad - all of them! That is nonsense. Mutations are entirely random. We must expect that many of them, perhaps most of them, will be bad but all it takes is for one or two of them to be beneficial and those one or two will be retained in the gene pool whereas the disadvantageous ones will eventually disappear. Dolphin (t) 12:32, 28 July 2010 (UTC)[reply]
Advantageous mutations are really harmful to the creature and result in a lower rate of selection. You can see how many advantageous mutations have occurred in man's history or any animal's history. Mutations generally are corrected or result in cancer or do not affect the creature at all. Mutation is disorder, and it cannot produce order except for the occasional good mutation which is buried under countless bad mutations. If all of those mutations happened to a creature and it impossibly remained alive, then that creature would be unrecognizable. Dolphin51, your statement seems to me to be bordering on ID. How can the good changes be saved and the bad changes be deleted if nature, with its bad mutation-causing sunlight and bad mutation-causing chemicals and other bad stimuli, generally leans to the bad and decaying side? Animals die; they do not become more healthy as they get old, which shows that helpful mutations are practically nonexistent. If you have one real dollar bill buried in a huge pile of counterfeit ones (or whatever your currency is), that whole pile is practically useless monetarily, even though it does contain a dollar bill. But suppose Nature (represented by a windstorm) came and blew away all counterfeit dollar bills, leaving just the real one. That would be evolution in action. If such a thing could be observed, it would prove natural selection is capable of discarding bad mutations and keeping good ones. --Chemicalinterest (talk) 12:57, 28 July 2010 (UTC)[reply]
Dear Chemicalinterest. Please provide some references for your bullshit. And while you're at it, although it'd be hard to make it factually correct, at least try to make your argument logically and linguistically followable. Aaadddaaammm (talk) 13:47, 28 July 2010 (UTC)[reply]
aaadddaaammm I write in a "stream of consciousness" techique. Whatever I think of I write. --Chemicalinterest (talk) 15:19, 28 July 2010 (UTC)[reply]
(EC with above) Your analogy is silly but let's go with it. Let's say I'm so evil dictator and I have millions of slaves. I want that real dollar bill, perhaps it has sentimental value. I send my slaves to each get a bill from the pile. If they come back with a counterfeit I shoot them in the head. (If they come back with nothing I also shoot them in the head). Guess which of my slaves will be alive at the end? The one who found the dollar bill. Let's use a less extreme analogy. This time I'm not quite so evil. I don't kill my slaves. I do require each one of them to pick up a dollar bill. For the one who finds it, I let them spend that dollar bill however and whenever they want. One of my slaves is going to find that dollar bill and they now have some small advantage over their fellow slaves since they have this dollar bill to spend when they want. Nil Einne (talk) 14:03, 28 July 2010 (UTC)[reply]
(more ec) To keep with the analogy: This happens all the time. There is a mountain of rocks mixed with gold, and yet we select the gold out from the mountain. There is a mountain chain of rocks on the rock, and we still manage to pick out the Carrara marble. And while there are plenty of mutations, of course only those in the germ line affect evolution - if I remember correctly, there are on average about 4.6 mutations in the genome of each human. Many of those are neutral, some are advantageous in certain situations, and some are disadvantageous. Of course, the bad ones are selected out fairly quickly, leaving the "good" ones. --Stephan Schulz (talk) 14:09, 28 July 2010 (UTC)[reply]
I would hesitate to use those analogies, simply because they invoke an intelligent agent (a human being) to apply the fitness criteria. In truth, the problem is that the original 'pile of money' analogy is a crappy analogy that is divorced too far from reality to be the basis for drawing meaningful conclusions. An analogy that would be better – though still not good – would be one that involved dollar bills that could reproduce. (Ultimately, reproductive success is the basis for natural selection.) Suppose we take a million one-dollar bills, and impose a different mutation on each. All but one of the mutations is deleterious, preventing those bills from reproducing and passing on their defective (or 'counterfeit') gene. For simplicity's sake, we assume that the gestation time of a healthy dollar bill is one day, and that each healthy bill is able to produce one offspring per gestational period. Today I'll have 999 999 unsuccessful bills, and 1 viable one. Tomorrow, I'll still have the same 999 999 counterfeits, but I'll have 2 live ones.
That sounds pretty unsatisfying — the pile is still mostly junk. Something remarkable happens by the time I get out to about three weeks, however, when I'll have roughly equal numbers of unsuccessful bills and viable ones. By the end of a month, I'll have about a billion live dollar bills; only one in one thousand notes in the pile will be counterfeit. That's the power of natural selection over multiple generations. Note that this process still works even if it isn't a stark alive-dead or fertile-barren choice. Even small advantages add up over sufficient generations. Let's say that a particular mutation confers a very small advantage: on average, a 1% greater number of offspring in each generation. After ten generations, the population with the mutation will have expanded by about 10% more than the population without. After a hundred generations, that 1% advantage turns into 2.7 times as many offspring. After a thousand generations, that tiny little 1% advantage will be parlayed into twenty-one thousand times as many offspring — the original, slightly less advantageous version of the gene will be nearly invisible in the population. TenOfAllTrades(talk) 14:33, 28 July 2010 (UTC)[reply]
That analogy relies on the theory that good mutations are naturally selected, while bad ones are thrown away. It is not the case. Very rarely do cancers disappear (harmful mutations) or "good" mutations such as photographic memory get selected until everyone has photographic memory. --Chemicalinterest (talk) 15:04, 28 July 2010 (UTC)[reply]
Actually, successful mutations are selected in cancer. With loss of checkpoints in replication and increasing genomic instability, the rate of mutation tends to increase within a malignant tumor. Mutations which confer resistance to apoptosis, resistance to immune system attack, tolerance for hypoxia, more rapid cell division, and (after treatment begins) resistance to chemotherapeutic agents are all selected for (and observed in real tumors). As in populations of animals, the tumor cells that develop survival-impairing mutations do die off (or become a smaller fraction of the tumor mass) — but the death of individual cells doesn't eradicate the entire tumor. In real life, late-stage tumors tend to grow faster than early-stage ones; they've accumulated more 'beneficial' mutations. Natural selection isn't some supernatural ethical judgement; there isn't a magical preference for anything that humans would deem 'good'.
Look, it would probably be helpful if you just gave us a link to whatever creationist (or crypto-creationist 'intelligent design') website you're getting your material from so that we can hook you up with one of the many competent, clearly-written rebuttals of these terrible analogies. Better yet, read intelligent design and evolution, and let us know which concepts you're still confused by. TenOfAllTrades(talk) 15:23, 28 July 2010 (UTC)[reply]
Yes, the mutation is beneficial to the cell but not to the organism, which causes cancerous organisms to be ejected from the gene pool. --Chemicalinterest (talk) 15:47, 28 July 2010 (UTC)[reply]
All of this, while emotional, does not actually advance your argument one bit. Having a spontaneously mutating tumor in your organism is not the same as being a mutant yourself.--91.148.159.4 (talk) 16:38, 28 July 2010 (UTC)[reply]
I said above that I am not getting this from any website. It is just what I logically think when I observe the world. --Chemicalinterest (talk) 15:47, 28 July 2010 (UTC)[reply]
What is the evolutionary cost of a photographic memory? Does it get you laid more? Do people with photographic memory have more children? When they mate, their children probably won't have a photographic memory. Such is the highly chaotic nature of personality. Also, it doesn't suit most species to be immortal. It's actually a bad thing for the species -- you get tons of people with "old genes" and adaptation slows down. People usually get cancer after they have children. Evolution is a little cruel sometimes -- it doesn't care what happens to the parents so much after they reproduce. John Riemann Soong (talk) 15:29, 28 July 2010 (UTC)[reply]
There wouldn't be so much variety. If one type of animal was most suited to its area for reproduction, it would dominate and all other animals that are less adapted would be blown away by natural selection. The multitude of types shows that God create the world and enjoyed having large varieties of animals. The lack of missing links (and their very name) shows that animals do not change. --Chemicalinterest (talk) 15:47, 28 July 2010 (UTC)[reply]
Ever heard of ecological niches? I'm sure your god enjoyed making parasitic intestinal worms, and the sound of excellently designed carnivore teeth piercing the necks of small lambs fills him with inexpressible joy. I'm glad I don't know him, though knowing his friends is bad enough.--91.148.159.4 (talk) 16:38, 28 July 2010 (UTC)[reply]
There is no solid evidence for "photographic memory" in the usual sense to begin with. In humans, memory is very much trainable, indicating that genetic variation is at best one factor. And humans have indeed evolved to have outstanding memory over time. --Stephan Schulz (talk) 15:34, 28 July 2010 (UTC)[reply]

nearly insurmountable barriers

Don't forget group selection and sexual selection guys. They massively accelerate evolution and it really becomes less of a "mutation because of chance" and more of "reassortment and recombination of traits".

In fact, there are "insurmountable" barriers that evolution doesn't like to cross. Take for example, the electron transport pathway. In high school they tell you it gives you 32-34 ATP or whatever per glucose molecule, but IIRC a lot of energy is still wasted in that transition from cytochrome c oxidase to oxygen -- that final jump is huuuuge compared to all the previous jumps, and ATP synthase utilises only a fraction of that final redox reaction. The rest is wasted as heat. It would really give all eukaryotes (and all aerobic bacteria) more useable energy to develop a "cytochrome V complex" in between....but you see, the electron transport chain is such a fundamental pillar of cell life that nature usually punishes any mutant cells that mess around with it. (Not to mention, it is probably hidden away in a protected area in most genomes, to prevent damage to these critical genes).

The same goes for why plants don't simply avoid photorespiration by modifying RuBisCO by only allowing carboxylation and not oxygenation (both are Lewis acid'ish reactions) -- they work around the inefficiency of this enzyme by using C4 carbon fixation or Crassulacean acid metabolism on top of RuBisCO. Once complex multicellular organisms get ... complex, they usually refrain from tinkering with their fundamental machinery. The changes would just be too big.

Evolution takes the path of least resistance. High selective pressures can cause massive changes within a few dozen generations. John Riemann Soong (talk) 15:15, 28 July 2010 (UTC)[reply]

Do we get to archive/delete this thread as the unholy lovechild of trolling and soapboxing yet? Aaadddaaammm (talk) 15:59, 28 July 2010 (UTC)[reply]

Wait several more days and then you will get the wish of your heart. --Chemicalinterest (talk) 16:29, 28 July 2010 (UTC)[reply]
Since Chemicalinterest seems to be uninterested in reading the relevant articles on the topic (evolution, intelligent design, or even missing link, which he seems unaware is a misnomer), preferring instead to argue pointlessly, I don't intend to continue responding in this thread. I encourage other editors (who are interested in keeping this a Science Reference Desk) to do the same. TenOfAllTrades(talk) 16:52, 28 July 2010 (UTC)[reply]
The proof that giraffe needed more than one genetic change in order to look the way they do is that their necks are long because their legs are long. A short-necked giraffe would be unable to reach the ground in order to drink - so the long neck and the long legs had to evolve together. If the giraffe had first evolved a long neck - then the long legs would be unnecessary - so they must have started getting longer legs and needed to evolve longer necks in order to reach the ground! There is much more than that going on though - their hearts are stupendously powerful in order to pump blood all the way up those long necks. Read Giraffe#Circulatory_system to see the huge range of problems that long neck causes for the animal. It's heart is two feet long - it's blood pressure is twice that of other mammals - it has special valves in the arteries around the brain so that when it leans down to drink, the extra pressure doesn't cause it to black out! The legs have a specially built blood supply to prevent the huge pressures from exploding the veins and arteries. There is a lot more to the Giraffe than a longer neck!
So evolution had to change multiple aspects of the giraffe more or less at once. Because of the implausibility of genetic change for long legs, long neck, big heart, neck valves, etc all happening at once in one individual, the changes had to have been gradual. A few centimeters of extra leg length making it easier to reach higher branches, a few centimeters of neck length allow the animals to drink from shallower pools, a little extra heart capacity allowing for longer life and increased breeding success as pumping the blood higher put more of a strain on the organ. Round and round that cycle for 20 million years - and you go from a fairly normal-looking, deer-like Climacoceras to a modern giraffe - one centimeter at a time.
I used to live for a while in Kenya as a child - and on frequent occasions we'd go to the Nairobi game reserve and watch giraffes. It's really noticeable that the umbrella-shaped trees in the park were precisely high enough that the local giraffes could JUST reach them by stretching as far as their legs, neck, snout and tongue could reach. As tall as those animals are - they looked like they were standing on tippy-toes to reach the leaves! A giraffe with just a few inches of extra reach would have no trouble finding food...one that's just a few inches shorter would have a much harder time of things. It's not just the legs and neck - the snout and even the tongue is long. Conversely, trees that are shorter lose most of their leaves - they need more water and nutrients to produce leaves that do them no good because the giraffe get them - leaving them less food and energy for reproduction. A tree that grows just a few inches taller than the tallest animal can avoid the worst of the ravages of the Giraffe. Clearly the trees and the giraffe are locked in an evolutionary arms race.
SteveBaker (talk) 03:15, 29 July 2010 (UTC)[reply]
Steve: without disagreeing (because I think what you said is reasonably accurate), be careful of the 'had to' language. saying 'had to' implies all sorts of misleading things (that 'evolution' is a particular thing, that giraffes are somehow a necessary result, even that the modern giraffe is somehow implicitly present in all of its evolutionary ancestors). A few million years ago there were creatures which were distinctly not giraffes; environmental pressures turned them into modern giraffes over time, but environmental pressures could have turned them into a wide assortment of creatures. there is no particular 'thing' called 'evolution' that 'does things' to change creatures; 'evolution' is just a code-word to point to the numerous processes by which species consistently change in response to a consistently changing world. --Ludwigs2 12:38, 29 July 2010 (UTC)[reply]
Consider the weather. A myriad of factors are involved in myriads of locations, yet clear patterns emerge. 67.243.7.245 (talk) 18:58, 29 July 2010 (UTC)[reply]

OP eric155 back. Thanks all for an invigorating discussion. I still feel at sea on this question though I will read more. I'm thinking of the various references to giraffes above. I've long thought that the traditional evolutionary example of the giraffe is hackneyed and a red herring. Everyone sees the long neck, to the exclusion of all else. As Steve and others pointed out, that long neck needs a strong back to support it, and that back needs special legs to support it, and they all need a specially powerful heart to keep them fuelled and so on. It means that the neck has ancillary phenome traits which from the head right down to the toes, probably hundreds of them. I guess my question was how does nature select when so many traits need to work together to effect a functional organism. So, what's the point of a longer neck if the giraffe strains its back from the effort.

One of the most distinctive and apparently retrograde features of evolutionary adaptation, is that any beneficial mutation must have IMMEDIATE beneficial effect, otherwise it is just a bad or neutral mutation, and will not be selected for. That caveat is to many people the most puzzling aspect of the process, especially because human life is so much a planned affair where we forgo immediate benefits in order to achieve advantageous long-term goals. How nature can work without recourse to any teleological vision and yet end up with organisms of infinite and wondrous complexity is often baffling. It would mean that potential traits of immense power are obliviously ignored just because they do not have an immediate effect.

Part of the answer seems to be that the environment selects via the structure of the whole organism, not via single genes. I believe that Dawkins makes this point somewhere. But to me it raises the further question then of how a complex trait, one which includes many sub-traits, such as the giraffe's neck combined with its back, legs, heart etc can be propagated via the genome, which would have to select for multiple traits all across the genome. And of course the kernel of my question is this, briefly: If beneficial mutations are rare, then how much rarer would it be for an organism to have, by sheer happenstance, a multiple of these mutations? Allied to this problem is another one I also have, and that is simply how a beneficial mutation, a mere blip in the organism's history manages to survive. Don't forget that most individuals of a species die before they can reproduce. In many species only 1 or 2 percent make it. Now, the forces leading to extinction are so massive, I keep thinking that they would make some small new "beneficial" mutation nothing more than a tiny noise, inundated by far more powerful forces. It's like saying that having the beneficial mutation of eyebrows will make a difference when your village is attacked by marauders or a volcano explodes. How many millions of beneficial mutations disappear in the face of these vast destructive forces? Eric155 (talk) 07:59, 30 July 2010 (UTC)[reply]

That is actually an important point that often gets overlooked in simplified explanations: many potentially beneficial mutations will not survive following exactly that sort of situation, and neutral mutations will propagate simply because the organisms with them happened to survive and reproduce better. This effect is more pronounced in small populations, where genetic drift is more significant as a result. In large populations, it averages out a bit more, since a potentially beneficial mutation, or a beneficial combination of traits, will be more likely to arise several times, making it more likely that some individuals with it will survive long enough to reproduce. 86.164.66.83 (talk) 14:36, 30 July 2010 (UTC)[reply]

Do blackholes tear holes in space-time?

According to General Relativity mass create "dents" in space-time. But can a gravity well be so strong that it rips the fabric of space-time? 148.168.127.10 (talk) 13:52, 28 July 2010 (UTC)[reply]

You can interpret the singularity and the centre of a black hole to be a hole in space-time, I suppose. In reality, we don't know what happens there. The maths goes crazy, which is a very clear sign that we've made a mistake somewhere in our theory. --Tango (talk) 14:36, 28 July 2010 (UTC)[reply]
It is worth noting that singularities (at least as predicted in general relativity) have literally zero size. Qualitatively, I don't think I would generally refer to a zero-size object as a "hole", since that generally tends to suggest a space large enough to fit something through. Dragons flight (talk) 15:55, 28 July 2010 (UTC)[reply]

What makes more sense is that "black holes" force out all empty space from the event horizon in. That empty space as we now know has an energy and volume that cannot be ignored. this is how gravity works: it is the vacuum energy created by the condensation of extra empty space around the surface of matter trying to displace the matter or get "back in" to where it once was. Something that virtually all physics laws have taken for granted is that empty space is uniform throughout spacetimud. that is why we can't figure out black holes, we are using archaic laws of physics that have the empty space or as I like to call it aether "built in" to the equasions. Basically, we will never be able to understand "black holes" if we keep forcing the current laws of physics down the throats of black holes. it is similar to trying to both locate and measure an electron. the tools we use to do those things inherently prevent us from achieving the goals. —Preceding unsigned comment added by 165.212.189.187 (talk) 13:47, 29 July 2010 (UTC)[reply]

Ring singularities are thought to allow "swallowed" objects to pass through the black hole into a white hole, causing the development of parallel universes. ~AH1(TCU) 01:37, 2 August 2010 (UTC)[reply]

how many people have drifted off to space irrevocably while on moonwalks

how many people have died a cold and lonely death after drifting off to space from one of their moonwalks and not being able to get back to their ship? 92.229.14.166 (talk) 13:56, 28 July 2010 (UTC)[reply]

None? Don't you think you would have heard of it? See also Space accidents and incidents which I found from a simple search Nil Einne (talk) 14:04, 28 July 2010 (UTC)[reply]
"Don't you think you would have heard of it" - I'm an artist, I don't see why I'm supposed to follow space-news. 92.229.14.166 (talk) 15:20, 28 July 2010 (UTC)[reply]
You seriously never heard of the Challenger disaster then? These things are big news. I don't follow space news myself much but these things are big enough news that most people do hear them, whoever they are. This one would be even more so since it would have been the first time it happened (that we know of), and 'lost in space' or 'earth's latest satellite' also makes a great headline (even if in reality the body probably would burn up in the atmosphere before long so would join everyone else on earth). Nil Einne (talk) 07:33, 29 July 2010 (UTC)[reply]
No-one has died during a moonwalk or spacewalk (although Leonov got dangerously stuck during his Voskhod 2 walk). Almost all spaceflight casualties have all arisen during either launch or reentry (the scary energetic bits of flight); only Soyuz 11 happened on orbit. It is impossible to float away from the moon - lunar escape velocity is 2.4km/s, vastly more than the speed a person could attain by jumping. - Finlay McWalterTalk 14:06, 28 July 2010 (UTC)[reply]
Even the Soyuz 11 disaster happened during re-entry, it was just early on in re-entry so it was still "in space". It wasn't in orbit, though: it was after they fired the retrorockets. --Tango (talk) 14:44, 28 July 2010 (UTC)[reply]
I assume you mean "space walk" not "moon walk"? If not, you should know that there's gravity on the moon. You weigh less on the moon (about 1/6), but you certainly don't weigh zero. You couldn't "drift away" from the moon any more than your could on Earth.
You'd be hard pressed to drift away on a space walk, too. Usually they're tethered. They used to occasionally use thruster backpacks instead, but they don't do that anymore for whatever reason.
If you did somehow unhook your tether and drift away you'd probably still be OK. They never leave the ship (or station) completely unattended. If you drifted away from the space shuttle they could track you on radar and follow you. I'm not sure what the procedure is for dealing with an astronaut that somehow drifted away from the space station, but I suspect that they could jump in their Soyuz capsule and go pick you up that way. I'm not sure how fast they can suit up and get that thing ready for launch, though. APL (talk) 16:06, 28 July 2010 (UTC)[reply]
APL is referring to the manned maneuvering unit ("rocket backpack"). The "whatever" reason they no longer use it was a massive overhaul and reassessment of all hazards, risks, and safety considerations, following the Challenger disaster (as described in our MMU article). Though not in any way related to the Challenger incident, the MMU was one of a number of "risky" things that were eliminated from the American manned space program as part of a massive risk reduction campaign. Nimur (talk) 05:05, 30 July 2010 (UTC)[reply]
They still have a smaller version of it, but they don't use it regularly: it's just for emergency purposes. --131.188.3.21 (talk) 09:42, 30 July 2010 (UTC)[reply]
Oh, Just remembered, you may be interested in the legends of the Lost Cosmonauts. The story goes that some number of cosmonauts were lost, but Russia went to great lengths to erase them from history. Almost certainly nonsense, but just plausible enough that it makes a great campfire story. APL (talk) 16:10, 28 July 2010 (UTC)[reply]
You wouldn't drift away very fast (unless you pushed off quite hard, which you just wouldn't do), so they would have until your suit runs out of air/power (whichever comes first, I'm not sure which it is) to come and get you. The Soyuz has an airlock, so the pilot doesn't need to suit up, the drifting astronaut can enter the airlock without decompressing the rest of the Soyuz. Also, spacewalks on the ISS are always done in pairs, so the partner would already be suited up. Whether they could easily get to the Soyuz, I don't know (it would depend on which station airlock they were using, I guess). --Tango (talk) 16:18, 28 July 2010 (UTC)[reply]
Also, they have things like the Simplified Aid for EVA Rescue so they can manoeuvre back to the station on their own. --Tango (talk) 16:24, 28 July 2010 (UTC)[reply]
Also, and I'm REALLY surprised no one has mentioned the bleeding obvious, the astronaut is TETHERED to the space craft. An obvious precaution, and one of minimal cost and inconvenience. So space walks are really very safe, as the only thing that could hurt you, that is, drifting away, has been negated. Your big risks in space travel are your space ship blowing up, crashlanding, or losing its way. The dangers of getting out on a rope for a brief waddle in space is so harmless it will be a special kid's treat when Branson's Virgin Space Ship enterprises get operational. I wonder about some posters sometimes. I kinda imagine that if one of them was asked to go on a space walk, the first thing they would say is "Can I get a rope to secure me to the ship?" But not on Wikipedia. They may go on space walks in pairs on the ISS, but prior to this, most space walks, on MIR and the American ones, were by lone astronauts, all tethered of course. One of the scenes in Kubrick's 2001 : a space odyssey that had me wondering was that one in which Frank dies after he is thrown off course during his space walk by the deranged computer HAL. Why did Frank and Dave make those long space walks without any kind of harness? Just asking for trouble. Eric155 (talk) 05:15, 31 July 2010 (UTC)[reply]
An unexpected solar flare, collision with micrometeorites or even a gamma ray burst could potentially pose a risk to astronauts on a spacewalk, however. ~AH1(TCU) 01:26, 2 August 2010 (UTC)[reply]
Also, if someone were to drift off during a spacewalk, they might burn up in the Van Allen Belt or the thermosphere instead of colliding with the Earth's lower atmosphere while they are above the Kármán line. ~AH1(TCU) 01:35, 2 August 2010 (UTC)[reply]

explanation on weather precip chance

Ok, so I have noticed on weather.com there is an hourly forecast that includes precipitation percent. For the past couple of days, as an example, I have seen it say over an 8 hour period, 10am - 20%, 11am - 20%, 12pm - 20%, 1pm - 30%, 2pm - 30%, 3pm - 30%, 4pm - 30%, 5pm - 30%. Now statistically, that would say to me that there is only a 8.6% chance ((0.8^3)*(0.7^5)) that it will not rain at some point during that 8 hour span, but my experience shows that it does not frequently rain during that 8 hour time. So is there a special meteorological method that one is supposed to use to interpret these precipitation chance %s? Googlemeister (talk) 15:20, 28 July 2010 (UTC)[reply]

I don't think that is a proper calculation. The chance would be found by averaging all of the percents. --Chemicalinterest (talk) 15:50, 28 July 2010 (UTC)[reply]
I have wondered about those percentages as well. You often see the daily chance at around the same figure as the hourly chances, which doesn't seem to make sense. --Tango (talk) 15:55, 28 July 2010 (UTC)[reply]
I don't think the chances are probability. --Chemicalinterest (talk) 15:58, 28 July 2010 (UTC)[reply]
Your calculation is assuming that the hourly chance of precipitation can be interpreted as uncorrelated, independent events. In reality, the chance it rains at 1 PM is highly correlated with the chance it rains at 3 PM, etc. Consider the following example. I tell you that I will give you a dollar at a random time tomorrow. Then the probability you get a dollar between 1 PM and 2 PM is 1/24. Treating those hourly probabilities as independent you might then conclude you have a 1 - (23/24)^24 = 64% chance of getting a dollar tomorrow, but that is false since I have guaranteed you a 100% chance. In the limit of totally dependent events, one might predict the probability that it rains tomorrow is equal to the max of the hourly probabilities (i.e. 30% in your example). Much of the time weather forecasts seem roughly consistent with an assumption like this in that they predict a daily chance of rain not much different from the max of the hourly chance. In general, we would usually expect that the true probability of rain tomorrow is intermediate between the max value and the value calculated via your method, but it is not possible to predict an actual value without additional knowledge of how the hourly predictions relate to each other. Dragons flight (talk) 16:21, 28 July 2010 (UTC)[reply]
See also Probability of Precipitation--it's not quite as simple as the plain-language meaning the chance that it will rain in the city. DMacks (talk) 16:57, 28 July 2010 (UTC)[reply]
The forecast probabilities on weather.com are bogus -- they've annoyed me for years. You can safely assume that a higher "probability" means a greater likelihood of rain, but the actual numbers are pretty much meaningless as far as I can see. The only probabilities you can rely on in my experience are 0% and 100%. Looie496 (talk) 20:40, 28 July 2010 (UTC)[reply]
To those posting here who are, I'm guessing, mostly Americans - be aware that many weather services around the world do not provide percentage chance of rain forecasts. The one I know well is Australia's Bureau of Meteorology. One argument against that I have heard from some of its staff is that a percentage chance of rain forecasts is a cop out. It cannot be wrong. If it rains, they can say "We said it might" and if it doesn't, they can say "We said it was a low probability". HiLo48 (talk) 22:27, 28 July 2010 (UTC)[reply]
I've tried asking a question about adding up POP values, and it simply did not work out. I'm not sure how to calculate probabilities in this way within a general context anyway, but the POP is the probability that it will rain during a day or time period, not the percentage of the time it will rain. I'd say that the chance of rain during that 8-hour period does not exceed 30%, but in real-life circumstances it can rain when the probability is 0% and it can not rain when it's 100%. ~AH1(TCU) 01:22, 2 August 2010 (UTC)[reply]

safety procedures

Are these situations worrisome enough to warrant attention?

  • Perchloric acid and nitric acid being stored next to organic solvents
  • Bottles of acids like TFA, HF and sulfuric acid being stored in rusty metal containers-- is the rust a sign of a leak problem? (Don't worry the HF is stored in polyurethane).
  • A nurse storing a bottle of NaF next to the clinical trial medications in a research unit?

John Riemann Soong (talk) 16:02, 28 July 2010 (UTC)[reply]

In the order that you pose the questions.
  • This is bad laboratory practice. Of course, everything will be fine until you have a major fire in the lab, in which case the laziness will make the fire much worse and much more dangerous to try to extinguish. In most labs, it is really easy to design a storage system which keeps strong oxidizing agents away from flammable organic solvents, and putting such a system in place is a good reminder that you should try to envisage the worst case scenarios when thinking about safety measures.
  • This is fairly normal laboratory practice (in virtually every lab I've worked in). The rust is not necessarily a sign of a leak problem from the bottle inside, the atmosphere in chemical storerooms is often propicious for corrosion (because of drips on the outside of bottles that evaporate, or humidity levels, or condensation). The metal container provides a small amount of shock protection, and also a small amount of containment if the bottle breaks, but that's not the main reason. Simply, the label on the metal container is often easier to read than the label on the bottle!
  • I wouldn't do it, because it seems like a needless risk; on the other hand, you have to ask yourself where else the nurse would have to store it. In a lab, you can usually find an appropriate place to store something, but "storage resources" can be more limited in a clinical context. If you know that it's NaF, then at least the labeling is adequate!
Physchim62 (talk) 16:45, 28 July 2010 (UTC)[reply]

Also, is it normal for bottles of sulfuric acid, etc. to become really sticky? I didn't notice this before -- it was the first time I was rummaging without gloves (was looking for H2O2). John Riemann Soong (talk) 16:17, 28 July 2010 (UTC)[reply]

In my opinion, this question is asking for the kind of personal professional advice that we aren't really capable of giving. We can't really tell you what is safe or reasonable without knowing both the details of what you are doing and the legal / professional standards that apply to your particular workspace. You would probably be much better off discussing the issues with your colleagues and/or a local safety officer. Dragons flight (talk) 16:28, 28 July 2010 (UTC)[reply]
We can't give professional advice, but we can (and often do) reply to general queries like this. JRS is only an undergraduate (as he has made clear in his many previous posts), so has no power to change anything! Physchim62 (talk) 16:33, 28 July 2010 (UTC)[reply]
When someone asks for specific safety advice about their work environment I see that as less a general query and more analogous to looking for personal medical advice. If our advice is good, then there is no harm, but bad safety has the potential to cause personal harm in a way that is very analogous to bad medical advice. In addition, since none of us can see his personal work area, it is very possible that us strangers on the internet might be unaware of factors that change the situation significantly. For example, rusty metal containers might be innocuous 98% of the time, but 2% of the time an inspection might notice that the rust is caused by vapors from an improperly stored strongly oxidizing chemical (e.g. NaClO). Also, there are situations where subpar practices might be tolerated (even routine) in one lab environment while the same practices would be illegal and/or grounds for firing in another lab. We don't and can't have a complete picture, and hence I still think it is much better for JRS to discuss safety concerns with a more senior colleague than to rely on the advice of people on the internet. Dragons flight (talk) 17:54, 28 July 2010 (UTC)[reply]
Well if it's safe to begin with, I don't really want to make a fool of myself. John Riemann Soong (talk) 16:30, 28 July 2010 (UTC)[reply]
Jugs of strong acids (2.5 L concentrated sulfuric, etc.) often are coated with a thin layer of rubber/plastic, and that is what seems to become sticky. I assumed it was either swelling or decomposing a bit in the lab atmosphere (organic vapors in an orgo lab and/or acid vapors in an acid storage cabinet), plus picking up misc dirt in the stickiness. The test would be to remove part of this coating on a new bottle and see what happens over time. DMacks (talk) 16:52, 28 July 2010 (UTC)[reply]
In my experience, the problem is particularly bad with sulfuric acid bottles; with nitric acid, you tend to get the plastic coating hardening rather than softening, especially around the top. I guess its just that university research labs keep these bottles around far longer than the chemical suppliers would like ;) Physchim62 (talk) 16:59, 28 July 2010 (UTC)[reply]
If you have the slightest concerns or questions about safety, I encourage you to ask the person who organizes the lab storage about them. Ordinarily such people are delighted to see that people in the lab are thinking about safety -- usually their problems go in the other direction. Looie496 (talk) 20:33, 28 July 2010 (UTC)[reply]
Seconded. It is never foolish to seek to improve your knowledge of factors affecting Health & Safety. Any professional workplace (including academic or research establishments) should have a Health & Safety Department or at minimum a properly qualified person to consult, and clear procedures should be in place, training be given, and advice be available on request: if the ethos is one where concern for safety is thought foolish, I for one wouldn't care to work there at all (and yes, I have worked on sites involving laboratory, and mechanical & electrical engineering, facilities). You, JRS, are clearly well educated and intelligent, so if you're unsure about something's safety you either haven't been properly/fully instructed regarding your workplace's H&S by those who should have done so, or there is genuine cause for investigation and reassurance - or correction. 87.81.230.195 (talk) 23:27, 28 July 2010 (UTC)[reply]
It's not just my lab. Fellow interns in other labs describe a lot of the same things - clogged/disorganised fume hoods where the hoods are treated as storage areas. I mean, in training we were told to look out for these things, but I'm not sure in practice if it matters? If it does, it's the sort of thing I'd want to fix quietly since I've had a lot of favours done for me and I don't want to cause too much trouble.
Btw, can lab vapors cause that much rust? The containers are thoroughly corroded. Should HF be stored out in the open? With its translucent plastic container I almost thought it was just another bottle of alcohol -- I freaked out for a bit after I touched it and realised what it was, lol. John Riemann Soong (talk) 00:06, 29 July 2010 (UTC)[reply]
A volatile strong oxidizer (of which NaClO is perhaps the most common example) can thoroughly rust all available iron pieces after less than 24 hours if you leave a bottle open in an enclosed space. So it is certainly possible, and will depend on what is available. When not immediately in use volatile corrosive compounds should always be kept in closed containers. After all, if the vapor can eat iron it is generally likely to be bad for other lab equipment as well as the skin of young researchers. Dragons flight (talk) 03:07, 29 July 2010 (UTC)[reply]
JRS, only you can be fully aware of your own circumstances and constraints, but if I were in your position and seeing the things you are describing, and finding they were representative and not isolated, I would be investigating opportunities to transfer to a better-run establishment, and should I succeed in transferring I would consider passing my observations, perhaps anonymously, to whatever government authority has legal jurisdiction over Health & Safety in the locality concerned, in order to prompt an official inspection. In the UK that authority would be the Health & Safety Executive. Some might feel that it would be better to make representations to a senior level in the establishment or its parent organisation, if any, but in my experience standards are set from top management on down, and if things are that bad, the top isn't caring.
You say " . . . I'm not sure in practice if it matters?" H&S rules are never dreamed up for the sake of inconveniencing everyone for no good reason (though they can be inappropriately applied). Where H&S rules are flouted, it may not appear to matter for long periods of time without incident, right up to the moment when someone is maimed or killed. 87.81.230.195 (talk) 17:42, 29 July 2010 (UTC)[reply]

woodpecker question

This is an uncited and questionable excerpt from the Red-headed woodpecker article. "Like all woodpeckers, this species's tongue is actually one-half to two-thirds longer than the length which the bird uses for feeding. The extra length is wrapped longitudinally around the brain, between the brain and the skull, to provide added padding for the bird's head's repeated decelerations against wood. The tongue thus helps in preventing the brain from bouncing against the skull, which would cause brain damage." This sounds extremely suspect to me, but I am far from expert in the anatomy of these creatures, so could someone enlighten me as to if the tongue fact is true or not? Googlemeister (talk) 16:07, 28 July 2010 (UTC)[reply]

it's true. 92.229.14.166 (talk) 16:19, 28 July 2010 (UTC)[reply]
It is partially true. First, it is not true of "all" woodpeckers. Some have evolved different means of storing their long tongues. Second, the reason could be just for storage. It may have nothing to do with protecting their brain. -- kainaw 16:27, 28 July 2010 (UTC)[reply]
It hardly seems to have anything to do with "protecting the brain", otherwise there would also be structures at the front of the head. The line in the article is wrong. Physchim62 (talk) 17:20, 28 July 2010 (UTC)[reply]

on an "evolutionary" note, I find it completely implausible that this arrangement could have "evolved" in five generations. this would be like having your grandkid's grandkids suddenly all sprouting penises that go all the way around their butts to give them a cushion to sit on in school and at their desks. evolutionary biologists just have absolutely no proposed mechanisms that could account for this development in five generations. 92.229.14.166 (talk) 16:45, 28 July 2010 (UTC)[reply]

Nor should they have to give explanations for such ridiculous conjectuures! Physchim62 (talk) 17:20, 28 July 2010 (UTC)[reply]
No - it's totally impossible for 5 generations to do it. The population of these birds is declining - not increasing - so the average number of baby woodpeckers raised to maturity by a breeding pair over their lifespans MUST be less than two. So if some mutation happened - all at once (meaning, probably a single gene changed) in a particular bird - and if it's offspring out-bred birds without the gene 4:1 (so birds with the gene had, on average, 8 successful offspring rather than the more usual 2) and if the gene were passed perfectly onto 50% of the next generation then there could be 4x4x4x4x4 birds with the mutant tongue gene today. Sadly, that's only just over 1000 birds. So even with such crazily optimistic assumptions, there is no way that the entire species could have gotten this change in just 5 generations! Changes like this require many, many changes to the genome - each making a small difference to the survival of the birds (a few percent improvement, maybe, per gene change) - the spreading of that gene is slow - it's gotta take hundreds of generations. Worse still, you have to ask yourself - how are the birds that DON'T have the gene surviving without smashing their brains out? Since the species has been around for a long time - they would already have evolved an adequate means to avoid problems. Only some kind of a change of habitat (like maybe the trees that they inhabit are getting harder) could explain this. SteveBaker (talk) 19:34, 28 July 2010 (UTC)[reply]


Where is this "five generation" thing coming from? I see nothing that claims the tongues on anything evolved in five generations. -- kainaw 17:39, 28 July 2010 (UTC)[reply]
If you mean that it evolved in the some one hundred million years birds have been around, well, that's a lot more credible now, isn't it? 92.229.14.166 (talk) 18:44, 28 July 2010 (UTC)[reply]
It would be after then, analysing woodpeckers with this feature and closely related birds without this sort of feature and trying to determine how old the last common ancestor is would give some clue. As with Kainaw, I have no idea where this five generations thing came from, and you were the one to bring it up. Even for a young earth creationist, it makes little sense since five generations of woodpeckers is probably like 10? years ago. If it evolved in five generations and didn't exist 10 years ago we wouldn't even be discussing this since it would be another example of evolution creationists like to ignore. Nil Einne (talk) 19:26, 28 July 2010 (UTC)[reply]
Yes. Yes, it is more credible. So ... Was this some ham-fisted attempt to say "If something is ridiculous in ten years then it must also be ridiculous in ten million years?", I hope not, not only because that is seriously flawed thinking, (Try applying it to continental drift, erosion, or building the great wall of china.) but because it means you were intentionally trying to muck up someone's question with a debate. APL (talk) 20:18, 28 July 2010 (UTC)[reply]
I now realise 98.229 is not Chemicalinterest or one of his/her sisters which I had initially thought so is perhaps not a YEC although I still don't understand where the 5 generations thing came in to it from. BTW SB is right that it's impossible for it to happen in 5 generations, I didn't mean to suggest it was possible, just that suggesting it happened in 5 generations makes little sense no matter how unusual your world view. Nil Einne (talk) 07:35, 29 July 2010 (UTC)[reply]
What do you mean by your first comment, Nil Einne? --Chemicalinterest (talk) 12:18, 29 July 2010 (UTC)[reply]
When I first responded I mistakenly thought 98.229 was you or one of your sisters (and therefore a young earth creationist) as the IP looked similar to your IP and their comments sounded a bit like what you would say in a topic you've been active in recently. Nil Einne (talk) 16:47, 29 July 2010 (UTC)[reply]
Please note that the IP is 92.229 not 98. --Chemicalinterest (talk) 18:10, 29 July 2010 (UTC)[reply]

I don't understand part of General Relativity

General_relativity#Relativistic_generalization

I don't understand this section at all. What is it saying? 148.168.127.10 (talk) 16:22, 28 July 2010 (UTC)[reply]

Wow — I don't really understand it either. It's either a restating of rather basic GR principles in a very stilted, technical style, or it is trying to be a very technical digression/exposition. In either case, it is not written clearly, or encyclopedically, at all, and it assumes far too high level of an audience understanding of the concepts to be very useful to most readers. It should be rewritten, at the very least. Other sections are much more clear, even when discussing complicated or technical aspects of the theory. --Mr.98 (talk) 19:01, 28 July 2010 (UTC)[reply]
I agree, it's too complicated for someone that hasn't studied at least some maths at university to understand. I haven't read through it fully, so I'm not sure how much of it ought to be included in the article. Some aspects of GR can't really be explained to the layman (not without simplifying it to the point of it being wrong), but those parts are probably not appropriate for an encyclopaedia. At the very least, it should be split out into a separate article. --Tango (talk) 19:39, 28 July 2010 (UTC)[reply]
Right, that's more what I was trying to convey. It's not that Wikipedia can't contain technical information only understandable to experts, but these should be "drilled down" a bit, not part of a general overview. Some of the concepts it appears to explain can certainly be done in more straightforward, non-technical language, like the equivalence principle, or the fact that GR is SR+gravity, or that light cones are important, etc. --Mr.98 (talk) 20:49, 28 July 2010 (UTC)[reply]
I've put a "confusing" tag on the section, for what it's worth. I invite everyone to comment on the talk page. Comet Tuttle (talk) 06:37, 29 July 2010 (UTC)[reply]
The article is giving a description of what is the meaning of the equivalence principle using a language that can only be understood by readers that have some experience with riemannian geometry. Dauto (talk) 18:31, 29 July 2010 (UTC)[reply]
Try Introduction to general relativity, a featured article. ~AH1(TCU) 01:14, 2 August 2010 (UTC)[reply]

30% hydrogen peroxide...usually by weight or volume?

Sorry for this question -- it's kind of urgent-- googling doesn't help me. The particular catalog number I'm looking at is H325-500 (Fisher Scientific). Do people usually sell these things by weight or volume? I have to make a 200 micromolar solution of H2O2 in liquid HeLa cell culture media. I've calculated I need about 0.00465 microlitres of pure hydrogen peroxide per every mL of cell culture, but don't know how much 30% H2O2 to add. I figure I can ignore the 70% water in the Fisher stock solution, either way? John Riemann Soong (talk) 16:42, 28 July 2010 (UTC)[reply]

These guys say by weight. But it doesn't matter - it decays and won't stay at 30% once you get it, the decay rate depends on temperature, how often the bottle was opened, light, shaking, etc. I think you should be able to measure the percent content by measuring the density, since H2O2 is much more dense than water (1.44 g/cm^3). (It will decay while you are working with it, so work fast, avoid shaking it, and avoid catalysts.) Ariel. (talk) 17:00, 28 July 2010 (UTC)[reply]
My mass balance says 1.13 g/mL. (I truncated the extra digits since I probably got some latex dust on there.) Ummm, I'm really embarrassed to ask this, but how do I back calculate a weighted average? I knew how to do this! Once! John Riemann Soong (talk) 17:36, 28 July 2010 (UTC)[reply]
water = 1.00g/mL, H2O2 = 1.44g/mL, mixture = 1.13g/mL, x = percent H2O2
mixture = water * (1-x) + H2O2 * x
solve for x = (mixture-water)/(H2O2-water)
x = .295
(I should turn that into math markup.) Ariel. (talk) 19:33, 28 July 2010 (UTC)[reply]
Doh. I was trying to find a quick fix using google calculator and it wasn't coming to me. Google doesn't support algebraic notation, I guess. John Riemann Soong (talk) 20:09, 28 July 2010 (UTC)[reply]
Use http://www.wolframalpha.com/ next time. Ariel. (talk) 20:24, 28 July 2010 (UTC)[reply]
(Okay, it's close to 30% ...good enough for me! But I found that out by guess and check. Kinda unsatisfying.) John Riemann Soong (talk) 17:38, 28 July 2010 (UTC)[reply]
I think CRC Handbook has a quick-lookup table (per "Viscosity and Density of Concentrated Hydroxide Solutions" new-feature listed in the ad for it in my recycling bin:). DMacks (talk) 17:45, 28 July 2010 (UTC)[reply]

OK thanks guys. I'm basically following a protocol outlined here, except using a549 cells and not astrocytes. H2O2 shouldn't oxidise the nutrients in cell culture media...right? Well I mean, it will probably do it to some extent, but when I make 200 micromolar H2O2 in cell culture media, most of it should remain unconsumed for 2 hours, right? John Riemann Soong (talk) 18:10, 28 July 2010 (UTC)[reply]

Well I'm going to take the plunge, but if I mix 1:1 solution of H2O2 and citric acid, should I observe a pH increase after a few hours? What about 1:1:1 H2O2, glucose and casein? John Riemann Soong (talk) 18:41, 28 July 2010 (UTC)[reply]

Smardale Gill viaduct - sandstone or limestone

Different sources describe this as being built from either "local pink sandstone" [8] or "local limestone" [9] [10]

I can't tell the difference from a photograph at 1km away.. Can anyone else? Also is it possible that it is made of both - in which case would either be more likely to be used as quoins/dressing. Also not to be confused with "Smardale viaduct" which is different. Thanks.87.102.76.166 (talk) 19:14, 28 July 2010 (UTC)[reply]

I would trust the nvt.org websites info over others, since this is their project. Also the stone doesn't look pink to me. Ariel. (talk) 19:28, 28 July 2010 (UTC)[reply]
It looks sort of pink here http://discoverbutterflies.com/how.html about half way down.. But mostly soot.87.102.76.166 (talk) 19:36, 28 July 2010 (UTC)[reply]
On the best picture of the viaduct I could find [11], it looks like limestone (from the grey colour). Mikenorton (talk) 23:50, 28 July 2010 (UTC)[reply]

do you have enough calories to get to the moon? (or: how fat do you have to be)

this is a purely armchair, arithmetic question: does a person have enough joules on their person to escape Earth's gravity and propel themselves to the moon. If not, how fat would a person have to be (at 3500 Calories per pound of fat) to have enough joules on their person for such a journey? This is just a classical physics armchair arithmetic problem, no need to include the usage of any equipment at all... —Preceding unsigned comment added by 92.229.14.166 (talk) 22:01, 28 July 2010 (UTC)[reply]

Earth's escape velocity is about 11.2 kilometers per second; you'll need just shy of that amount of kinetic energy to get over the slightly smaller barrier between the Earth and the Moon. In round figures, that's about 50 megajoules per kilogram. The energy content of fat is (as noted) about 3500 Calories per pound, which runs to (roughly) 40 kilojoules per kilogram. Back of the envelope says that if you were able to instantaneously convert all of the body's fat to kinetic energy and impart that to the non-fat body bits, you'd have to be a bit more than 99.9% fat. (Assuming I've not lost some orders of magnitude anywhere.) The problem gets even worse if you try to burn fat continuously on the way up, because then at least some of the energy goes into lifting fat that will later be burned. TenOfAllTrades(talk) 22:16, 28 July 2010 (UTC)[reply]
Oops. For some reason I was reading the energy content of fat as 37 joules per gram, rather than 37 kJ. Pay no attention to my numbers; they are wrong by three orders of magnitude.... TenOfAllTrades(talk) 22:43, 28 July 2010 (UTC)[reply]
Fat has an energy of 38 MJ/kg, the quoted Calories are kcal. That corresponds to lifting it 11,000 10,000 km.--Patrick (talk) 22:24, 28 July 2010 (UTC)[reply]
It doesn't work. Sorry. (Unless I did my math wrong.)
v = 25010 mph (escape velocity)
e = 1/2 mv^2 (energy)
e = m*38 MJ/kg (energy from fat)
m = x lb (x is the unknown amount of fat)
x lb*38 MJ/kg > 1/2 * x lb * (25010 mph)^2 (substitute variables, and note that it's an inequality)
solve for x. Result: x < 0. Meaning it doesn't work.
BIG NOTE: This is assuming you are lifting all the fat into orbit. Which you may or may not be doing depending on how you burn the fat. Ariel. (talk) 22:30, 28 July 2010 (UTC)[reply]
If we assume a person weighs m without any fat, and we need to get them to 11.2 km/s, that's about m*6.3 x107 J/kg. That's about 1.5 x104 kcal/kg, and fat stores about 9000 kcal/kg so that's about 1.6*m of fat. If you could burn it all instantly and convert 100% efficiently into kinetic energy then you could escape Earth. So if a person weighed 50kg without fat, they would need about 83kg of fat. Rckrone (talk) 22:37, 28 July 2010 (UTC)[reply]
As Rckrone mentioned, if you don't carry any the fat with you (i.e. you burn it all at once), then it's possible (but note a real rocket does not do that). For a 150lb person it's: 246.716lb of fat. Ariel. (talk) 22:41, 28 July 2010 (UTC)[reply]

If you climbed a really long ladder to space instead, would you consume more or less energy? John Riemann Soong (talk) 10:36, 29 July 2010 (UTC)[reply]

Climbing a ladder would mean that you are consuming fat via your cell processes, not actually releasing all of its energy in a theoretical situation. Since cell processes aren't 100% efficient, it may take more energy (please correct me if I'm wrong). --Chemicalinterest (talk) 11:03, 29 July 2010 (UTC)[reply]
Plus in the ladder climbing scenario, you would starve to death before reaching the moon since it would take years to climb that ladder, and you have baseline metabolism robbing you of energy. Googlemeister (talk) 12:46, 29 July 2010 (UTC)[reply]
Well let's say you have exceptional climbing ability. What I meant to ask is -- since you're relying on a normal force pushing you back, you don't need to reach escape velocity. Does that save energy, or no? John Riemann Soong (talk) 13:21, 29 July 2010 (UTC)[reply]
If the earth did not rotate then it doesn't help at all - you could climb to the top, but to actually reach the moon you need the same total energy as the escape velocity. And escape velocity takes into account the energy of climbing out of gravity. But the earth does rotate, so by climbing up a ladder you allow the earth to spin you, if the ladder is long enough to reach geosynchronous orbit then the earth provides most of the energy you need. You just need enough to climb up against gravity. Ariel. (talk) 16:31, 29 July 2010 (UTC)[reply]
In the ladder scenario, one you got to the height of the moon, it would at some point, crash into you at lethal velocity. Googlemeister (talk) 18:28, 29 July 2010 (UTC)[reply]
Looked at another way, how much matter, in the form of fat, if converted to energy, would be needed to propel a 100 kg person to the moon, if E = MC2? Edison (talk) 05:08, 30 July 2010 (UTC)[reply]
There is no practical way to get a hydrocarbon, let alone cellulite, to undergo the kind of nuclear reaction that would release even a small amount of energy in this way; and there definitely is no practical way to annihilate the entire fat molecule either to convert its entire mass to energy; so if this is strictly a gedanken-experiment, the fact that you are using "fat" as the mass to be converted to energy is irrelevant. Nimur (talk) 05:16, 30 July 2010 (UTC)[reply]
To accelerate 100kg to the escape velocity of the earth will require the energy in 69.54 micrograms (6.954x10^-5 g) of matter - any matter, not just fat. Ariel. (talk) 05:22, 30 July 2010 (UTC)[reply]

The potential energy increases from -63 MJ/kg at Earth to -4 MJ/kg at the Moon (-1 MJ/kg w.r.t. Earth, and -3 MJ/kg w.r.t. the Moon), so with constant mass it is an increase of 59 MJ/kg. If fat is burned instantly the amount needed is 59 / 38 = 1.55 times the mass of the payload, and to infinity 63 / 38 = 1.66 times the mass of the payload (the 1.6*m already mentioned above). Ignoring the rotation of Earth, for climbing the ladder with 100 % efficiency while burning fat it seems we have a mass reduction by a factor e for every increase of the potential field by 38 MJ/kg, so to infinity mass is reduced by a factor e ^ 1.66 = 5.3. For a ladder to the Moon we have to climb to the point where we start descending. From there we can recover energy with suitable equipment, but to convert that back to body fat requires some material to be found along the way.--Patrick (talk) 09:22, 30 July 2010 (UTC)[reply]

Direction of current

http://www.tqa.tas.gov.au/4DCGI/_WWW_doc/009120/RND01/PHY5C_Paper.pdf
Question 9 (a) (ii)
How do you do it? 115.178.29.142 (talk) 22:21, 28 July 2010 (UTC)[reply]

Well, how far have you gotten, and what has you stuck? TenOfAllTrades(talk) 22:24, 28 July 2010 (UTC)[reply]
Check you understand or already know Fleming's left hand rule for motors and Fleming's right hand rule. Because the ions are moving that's like an electric current - big clue - the positive and negative ions moving in the same direction are like currents in opposite directions. I'll give more info. if you are still stuck. (also remember that the direction of the electric current is from positive part to negative part) Sf5xeplus (talk) 23:23, 28 July 2010 (UTC)[reply]
It's the second part of the question that I can't do (part (ii) i.e. the direction of the current through the voltmeter), not the first part (part (i)).115.178.29.142 (talk) 00:11, 29 July 2010 (UTC)[reply]
Huh? The second part is the simple bit, if you can do the first part! Maybe if you read our small section on conventional current, you will see the answer (the question is asking if you know what conventional current is). Physchim62 (talk) 04:05, 29 July 2010 (UTC)[reply]

can you tell just by the honk whether a man or woman is blowing their nose?

can you tell just by the honk whether a man or woman is blowing their nose, a child or adult, etc.? —Preceding unsigned comment added by 92.229.14.166 (talk) 22:31, 28 July 2010 (UTC)[reply]

How about you, can you do it? Is this a poll? Some people are presumably better than others at this sort of thing. I probably could tell a child, but not the difference between a man and a woman. Is this really a serious question? Ariel. (talk) 22:48, 28 July 2010 (UTC)[reply]
obviously by "can you" I mean "can one". I was thinking by analogy with voice (clearly different for males and females, very few voices are hard to tell which), due to anatomical differnces. In some ways there is anatomical similarity with honk production, namely lungs are also involved. where is honk produced? in the nose, presumably. so, again, would men have larger or more cavernous noses on average? Anyway these are just possibilities, the answer could be "unlike voice, there is no discernible difference between male and female honk". But I would like to know your reasoning if that is your answer. 92.229.14.166 (talk) 23:20, 28 July 2010 (UTC)[reply]
It's really hard to believe that anyone would study such a strange matter - so it's unlikely that we'll find a good answer for you. SteveBaker (talk) 02:18, 29 July 2010 (UTC)[reply]
I agree that it is most unlikely that any definitive answer is available, but I would put my money on the answer being "no" (discounting the fact that, in many cultures, women find it less acceptable to blow their noses loudly). Perhaps an ENT specialist would know about anatomical differences (if any) between genders, but I suspect that variability within each gender completely swamps any slight gender difference. Is there a significant difference in head size between genders? Dbfirs 07:03, 29 July 2010 (UTC)[reply]
I smell an Ignoble prize. Googlemeister (talk) 12:46, 29 July 2010 (UTC)[reply]
Absolutely not. I know a woman whose nose honk is quite loud and impressive, such that one would have thought it came from Paul Bunyan or the Jolly Green Giant. Edison (talk) 05:01, 30 July 2010 (UTC)[reply]

Are we all epigeneticists?

From some book:

"one of the great controversies, that of preformation versus epigenesis and this was settled primarily by reference to chick development. The preformationists thought that the entire individual was present in miniature in the egg or, alternatively, within the sperm and that it simply grew until it hatched; there was thought to be no differentiation, merely growth. The epigeneticists considered that both growth and differentiation were involved and were necessary. Today, we are all epigeneticists."

Is this the correct use of the word epigeneticist? --92.25.227.155 (talk) 22:46, 28 July 2010 (UTC)[reply]

The roots epigenesis (biology) and epigenetics are clearly different in meaning; "epigenesis" applies to a theory of developmental embryology and "epigenetics" relates, in a very general sense, to structural changes that control the expression of genes (though interestingly quite important in regulating embryogenesis). I suppose one could call a scientist who studies epigenetics to be an epigeneticist, but I don't hear that term used very much. Given that preformation has been soundly debunked in favor of a complex developmental process resembling epigenesis, the author of your quote is presumably saying that there is no one left who still thinks preformation is a viable theory. Maybe he or she thought the term "epigeneticist" was just better than "epigenesisicist". --- Medical geneticist (talk) 23:32, 28 July 2010 (UTC)[reply]
We are all epigeneticists if you are using the terms of the 19th century embryological debates — we do believe that development is crucial, and even the most hard-line hereditarians do not believe in the preformation hypothesis in the terms it was believed in the 19th century. We are not all epigeneticists if you are using the term as it came into usage in the late 20th century as applied in the term "epigenetics". Personally I think it is a misleading phrase — a more accurate way to put it would be to say that we are no longer preformationists. (What we "are" here is more defined by the negative by the positive, because we don't really believe everything the 19th century "epigeneticists" believed, I am sure.) --Mr.98 (talk) 01:16, 29 July 2010 (UTC)[reply]
When everyone takes the same label - there is no need to use the label anymore because it loses all linguistic value. There used to be heliocentricists and geocentricists (those who believed the sun was at the center of the solar system and those who believed the earth was at the center) - but now that we are all heliocentricists, we don't need to use the word anymore. That doesn't make the use of the word incorrect - but rather just obsolete. SteveBaker (talk) 02:16, 29 July 2010 (UTC)[reply]
I don't think it's quite true that the word is obsolete if the position is debunked. We still want to be able to talk about the historical debate. Rckrone (talk) 05:27, 29 July 2010 (UTC)[reply]
True, but we also need to be careful about our use of terms: probably everyone on this page accepts that there are things called "atoms", but that doesn't make us "atomists" in the sense of Ancient Greek philosophy! Physchim62 (talk) 06:50, 29 July 2010 (UTC)[reply]
But we're not really heliocentrists — we don't believe the sun is at the center of the universe, which was what the original heliocentrists believed (see Heliocentrism). That's kind of my point — we actually don't subscribe to the beliefs of the "winning" group so much as we have repudiated the beliefs of the "losing" group. It's not that we all became heliocentrists, but rather that we are simply not geocentrists. The nature of this kind of scientific change (if one can generalize it) is not so much that we all switch to some winning view, but we go away from the losing view, and a whole new set up complicated views opens up. It is more accurate to just say that nobody (to a rough approximation) believes the old view anymore, rather than saying that everybody believes the new view (which is a vast oversimplification). --Mr.98 (talk) 13:36, 29 July 2010 (UTC)[reply]

Sun Exposure, Skin Cancer and Vitamin D

I read a New York Times article about how being outside, minimally clothed and unprotected by sunscreen, is an excellent way for one to acquire his healthy dose of Vitamin D. I have also been told for my entire life that doing these things will give me sunburns and horrible skin cancer. I would like to believe that the NYT version is true, because I wouldn't need to worry about sunscreen anymore. Is there a dermatologist in the house? Can anyone sort this out for me? Thanks. Postprehistoric (talk) 23:31, 28 July 2010 (UTC)[reply]

you could try Risks and benefits of sun exposure which quotes from a study "there is not enough information to determine a safe level of sun exposure at this time" ,, also "Despite the importance of the sun to vitamin D synthesis, it is prudent to limit exposure of skin to sunlight". You also still would want to avoid sunburn for all the reasons. 87.102.76.166 (talk) 23:52, 28 July 2010 (UTC)[reply]
My understanding is that vitamin D can prevent skin cancer, but it only takes 15-20 minutes to max out the vitamin D production, anything more than that is not necessary. See also Sunburn#Controversy over sunscreen and Sunscreen controversy. Sunscreen might not help at all. If you do use it I would pick one with titanium dioxide or zinc oxide, because they are considered far more effective at preventing real damage (as opposed to reddening, which all kinds prevent). This subject has massive debate going on right now, no one can give you a definitive answer. Some studies even show that using sunscreen actually increases the risk of melanoma. The only thing (I think) everyone agrees with is: Get less sun and take vitamin D supplements instead. (But simply less sun without vitamin D might be worse than more sun.) Ariel. (talk) 00:21, 29 July 2010 (UTC)[reply]
I don't have a reference to hand, but apparently you can get enough vitamin D by spending about 10 minutes outside on an overcast day with just your face and arms bare. In fact, you can get enough vitamin D just from eating a balanced diet without any sunlight. So, in short, don't worry about vitamin D. (This is all assuming you are reasonably fair skinned. If you black and live somewhere at reasonably high latitude, then vitamin D can be more of an issue.) --Tango (talk) 00:36, 29 July 2010 (UTC)[reply]
It isn't a question of getting enough vitamin D. Vitamin D is a powerful antioxidant, and there is reason to think that high levels of it may have a protective effect against cancer. But that's based on theory and isn't backed up by much actual evidence, one way or the other. Looie496 (talk) 00:41, 29 July 2010 (UTC)[reply]
Medical advice? Aaadddaaammm (talk) 07:26, 29 July 2010 (UTC)[reply]


About vitamin D, I think the evidence now strongly points in the direction to the recommended levels of intake being way too low, the optimal levels being so high that it even exceed the current official maximum safe dose. It takes time for the health officials to update the recommended dose, but as this article points out, the expert themselves are using vitamin D supplements themselves at levels they think are healthy which is of the order of 5,000 IU per day.

One has to note that a vitamin D dose of the order of 10,000 IU per day is entirely natural, it cannot be compared to e.g. taking a high vitamin C dose that you can only get that from supplements. E.g. bushmen in Africa don't wear much clothes and hunt for meat during the day for hours when the UV index can be as high as 12. They then get a dose of 10,000 to 20,000 IU of vitamin D and not much more than that, because of a dynamical equilibrium between vitamin D synthesis in the skin due to UV radiation and vitamin D being broken down by UV radiation. To get an overdose of vitamin D you need to take 100,000 IU every day for several weeks.

Since the 17th century more and more people have been living and working indoors causing extremely low vitamin D levels leading to problems with calcium absorpion from food. To prevent this, you only need to take very low doses of vitamin D, a few hundred IU per day is enough. This is what the current recommendation is based on. But new evidence suggests that this is low dose is similar to the miniscule dose of vitamin C you would need to take to prevent scurvy.

If we had all stopped eating fresh vegetables and fruits millennia ago and getting the minimum amount of vitamin C to prevent getting scurvy, we may well have set the recommended dose of vitamin C to be a lot less than found in a single apple and one apple would perhaps be the maximum safe dose as there would not have been the rigorous medical evidence that eating several pieces of fruit per day does not cause some unknown horrible illness many year later. This logic would set the officially recommended dose but, of course, the experts in the field would know that from a more reasonable point of view this is all nonsense and there would likely be evidence that a higher dose is actually beneficial. What you would see is that the experts themselves would not stick to the official guidelines, which is exactly what is happening in case of vitamin D.

References:

http://www.eurekalert.org/pub_releases/2010-07/tes-vdd072710.php

http://www.naturalnews.com/029312_immune_system_vitamin_D.html

http://www.webmd.com/parkinsons-disease/news/20100713/vitamin-d-may-lower-parkinsons-risk

http://www.aafp.org/online/en/home/publications/news/news-now/health-of-the-public/20100727vitamind-cognition.html

http://www.medscape.com/viewarticle/724646

http://news.xinhuanet.com/english2010/health/2010-07/03/c_13381512.htm

http://www.webmd.com/baby/news/20100504/high-doses-of-vitamin-d-may-cut-pregnancy-risk

http://www.reuters.com/article/idUSTRE62I3MK20100319

http://www.webmd.com/asthma/news/20100128/vitamin-d-may-ease-asthma


Count Iblis (talk) 18:40, 29 July 2010 (UTC)[reply]

A tan from sun exposure by itself can provide an SPF of 4. However, this wouldn't prevent harmful sunburns. Using sunlight to fill up on Vitamin D would have been viable in the past, but nowadays it's more difficult to do this safely due to the ozone hole, but instead of sunscreen you could use sunblock. ~AH1(TCU) 01:04, 2 August 2010 (UTC)[reply]

July 29

Turbojets

The article on turbojets seems to indicate that they are more efficient than turbofans above Mach 2, but it doesn't really give a clear answer. Is that true? --The High Fin Sperm Whale 00:48, 29 July 2010 (UTC)[reply]

According to this chart (in the Jet Engine article) a turbojet will be more efficient (measured in Isp) than a turbofan starting at about Mach 1.5. I suspect this is because the turbojet will have a higher average exhaust velocity than the turbofan. A turbofan exhausts a large volume of lower velocity air, but since the thrust has to do with the difference between airspeed and exhaust velocity, the lower exhaust velocity means lower thrust at a high airspeed. The turbojet exhausts a smaller volume of high velocity air, so it maintains higher thrust at a high airspeed.
anonymous6494 02:14, 29 July 2010 (UTC)[reply]
Thanks, but the chart does not show turbojets. Thanks anyways. --The High Fin Sperm Whale 02:31, 29 July 2010 (UTC)[reply]
For these purposes, you can think of a turbojet as a low-efficiency turbofan - so on that chart, they will lie just below turbofans. For example, the engine that powered the infamous Concorde will have an Isp of around 3000 s at cruising speeds (just over Mach 1). You can verify this for yourself by converting the thrust specific fuel consumption in to specific impulse (the equations are given in our article for a variety of units). In this case, Olympus 593 engines have a SFC of ~1.2 lb/lbf·h - so convert by (3600/1.12), yielding Isp around 3000 s at cruise speed of Mach 1. In reality, the distinction between a turbofan and a turbojet is a semi-subjective distinction about what counts as "low pressure" bypass. There are many engines that could pass for either description. Compare the cartoon diagrams of turbofans to turbojets - as you can see, the big difference is expansion ratio of the bypass flow - and there is not a clear demarcating line between the two. In the extreme case, of course, we can clearly see that a large amount of ducted air is not "jet-like", and you can keep making the bypass ratio higher and higher... at some point you stop bothering with a nacelle and end up with an unducted turboprop (even more efficient! but much slower). Nimur (talk) 03:50, 29 July 2010 (UTC)[reply]

What is the lightest/least dense non-porous solid?

So if Aerogel is the least dense porous solid, which non-porous solid has the lowest density? —Preceding unsigned comment added by 71.163.18.235 (talk) 01:51, 29 July 2010 (UTC)[reply]

Lithium is the lightest element that's solid at room temperature at 0.53 g/cm3. I don't know whether there are any less dense compounds out there - but I suspect not. SteveBaker (talk) 02:02, 29 July 2010 (UTC)[reply]
If we can freeze non-solid-at-RT things, solid hydrogen I think is probably around 0.07 g/cm3. Or at least that's what hydrogen says the liquid is at the melting-point, so probably close? The article specifically about the solid has few physical properties. DMacks (talk) 02:38, 29 July 2010 (UTC)[reply]
It's 0.088 g/cm^3 which is the lowest of all known crystalline solids. Dragons flight (talk) 02:44, 29 July 2010 (UTC)[reply]
Could you add that with cite to the solid hydrogen, and maybe also hydrogen articles? DMacks (talk) 06:18, 29 July 2010 (UTC)[reply]
For this one, we might have to define "porous". Everything would have exactly the same density if it were not for the void-spaces. It just so happens that some solid materials have void-spaces on molecular scales that are defined by crystal lattices; some solid materials have void-spaces on macro-crystalline or polycrystalline scales; some solid materials have void-spaces that are large enough to be filled by air or gas; and so on. One could conceivably build a sturdy steel shell and pump out the air from its interior: would it be fair to call that a low-density material? After all, there would be nothing but steel in it; nothing would be "diluting" its volume or mass. And it wouldn't really be porous. How small of a pore is required before you call a material "nonporous?" Nimur (talk) 04:05, 29 July 2010 (UTC)[reply]
It should also be noted that an aerogel is not a material per se. It is a colloid; which is to say it is a type of mixture. As a mixture, it is a combination of multiple materials; aerogel being a mixture of the solid matrix and air. The density of a mixture is dependent not only on the properties of the components, but the proportions in which they are mixed. Pure substances generally have well-defined densities, for example look at the infobox of iron or or sodium chloride or sucrose. However, not being a pure substance, one cannot define a specific density for aerogel in general, only for specific formulations of aerogel. --Jayron32 06:42, 29 July 2010 (UTC)[reply]

The Dirac sea perhaps. Cuddlyable3 (talk) 17:59, 29 July 2010 (UTC)[reply]

If you could get enough Di-positronium together and crystallize it you would get something about 1000 times less dense than solid hydrogen. However it is too radioactive with too short a life for this to happen in our current technology. Graeme Bartlett (talk) 10:23, 30 July 2010 (UTC)[reply]

Airport Runways

I would like to know if Airport Runways are built in one direction taking Earth's rotation into consideration. Are they built East-West? —Preceding unsigned comment added by Shyamlal777 (talkcontribs) 02:17, 29 July 2010 (UTC)[reply]

I don't think that the Earth's rotation needs to be taken into consideration, since the atmosphere spins with it. --The High Fin Sperm Whale 02:20, 29 July 2010 (UTC)[reply]
A trip to any reasonably-sized airport would disprove "Airport Runways are built in one direction". And even single-strip airparks would be likely to disprove "Are they built East-West?". They're built at whatever directions space allows and that is viable for prevailing weather conditions. Our runway article talks about it in a whole section about orientation. Airplanes care a lot about wind, which can blow in any arbitrary direction regardless of the earth' rotation, though there may be some places that almost always have wind in a certain direction. Weather systems are certainly affected by the earth's rotation on a large scale, but not "rotation -> that's how the breeze goes in general". Rockets and other things that are not relying on air motion for their own motion and that need to move in a certain direction relative to the earth could rely on the rotational boost. DMacks (talk) 02:29, 29 July 2010 (UTC)[reply]
Right. Most places have a prevailing wind, actually, and airplanes can take off and land more easily facing into the wind than facing with it or sideways to it. Looie496 (talk) 03:44, 29 July 2010 (UTC)[reply]
SImple land geography is a big factor too. Runways have to go where hills and mountains aren't. HiLo48 (talk) 04:28, 29 July 2010 (UTC)[reply]
I'm a pilot, and I can assure you that there is no set way that a runway must be oriented. DMacks, Looie496, and HiLo48 are all correct. They are built with the local environment, "normal" winds, etc in mind. Not all runways are the same. Some have [slight] hills in them, so they have a pronounced dip or crest in the middle, some are longer, some are shorter, some are wider, some are narrower, some can handle more weight, and there are some where you take off and land in one direction because there's a mountain on one end. It really depends almost entirely on what the runway is to be used for and where it is. Falconusp t c 05:06, 29 July 2010 (UTC)[reply]
Indeed. The airfield just down the road from me (LEIG) is planning on extending the runway: the main consideration was where they could buy the land from (although it's also one of those one-way runways with a biggish hill at one end). Physchim62 (talk) 05:30, 29 July 2010 (UTC)[reply]
Other then what's already been said, I think the consequences of the flightpath the runway will impose will also get consideration. For example a runway which means the planes will be largely taking off over water or flat, uninhabited land would generally be preferred over one which results in planes over taking off or landing over houses (meaning when height is not a concern) not just for safety reasons but because planes tend to be noisy particularly when low flying Nil Einne (talk) 07:27, 29 July 2010 (UTC)[reply]
Landing flight paths over uneven terrain are avoided if possible because signal reflections can distort the radio guidance beams. See the article Instrument landing system#Limitations and alternatives. Cuddlyable3 (talk) 17:44, 29 July 2010 (UTC)[reply]

About CAN networking

Dear Sir, i have a fiat car wit me some days back its stop working , some engineer came and said some one is putted cut in the main network line(CAN Network) bcoz of that its stop start working,now they have rectify that error.Now i want to know that whether dis cut can produce any error in future nd what kind of cut should i make in the networking so that my sub woofer(Amplifier) work properly .


Thanks —Preceding unsigned comment added by Kuks222 (talkcontribs) 02:46, 29 July 2010 (UTC)[reply]

The CAN-bus is an automotive network technology. Think USB for a car and it's a close approximation. As such, the repair shouldn't lead to errors down the line. Presumably they replaced a cable and, unless that cable is cut again, that's not a part that's likely to fail any more than USB cables go bad (which is to say, rarely). Of course, a future cut would produce the same problems. As for the subwoofer, as far as I know, car stereos don't use CAN-bus to carry audio signals. They'd be analog speaker wire rather than digital CAN-bus. — Lomn 03:54, 29 July 2010 (UTC)[reply]
I'm wondering if the person who installed the subwoofer cut the CAN bus wire, thinking it was the audio line and trying to re-route it to the subwoofer amplifier. In that case, you should make them pay for any repairs necessary. Nobody should have touched your CAN bus to install an amplifier. If the CAN bus is damaged, depending on your particular FIAT model, there may be serious safety concerns operating the vehicle - some engine control units and safety features like brake or airbag controllers may use the CAN for communication. Most cars have a separate CAN bus for critical vehicle systems and "passenger-compartment" things like radios and DVD players; but the information provided is insufficient to know what has been cut. Have a mechanic you can trust analyze the problem, and make whomever cut the wire pay for it. Nimur (talk) 04:13, 29 July 2010 (UTC)[reply]

iitmadras

Hi I am from Hyderabad. I want to do my MTech from IITs but my aggregate is less than 60% in my BTech. But in IIT Madras website I saw the minimum qualification is a degree and GATE score http://gate.iitm.ac.in/mtechadm/gelig.php Can I get the admission if my GATE score is good?? —Preceding unsigned comment added by 115.109.194.75 (talk) 12:09, 29 July 2010 (UTC)[reply]

why do stocks prices move during the day even when there is no new information!

why is it that even in boring times with no new information, multibillion dollar companies' stock prices fluctuate at least a few percentage points continuously every second or few, all day, every day? For me, it would make more sense for them to stay in a flat line for a few day (or hours, if a piece of news comes unexpectedly) then jump to the new price based on the new information and stay in a flat line until it jumps to a new price due to the next piece of news. But this never, ever, ever happens even within a single day (for the largest companies) let alone for days or weeks. Why is that? You don't get a different price at the gas station every fifth of a second, why do you on the stock market? 84.153.183.42 (talk) 16:31, 29 July 2010 (UTC)[reply]

The price does not depend only on the single company, it also depends on the industry group. It also depends on the relative risk between that company and other companies, and between that company and bonds. So other things change, which causes a relative change, which moves the price, i.e. other stuff might suddenly be slightly better or worse. Ariel. (talk) 16:37, 29 July 2010 (UTC)[reply]
so? Information changes in the real world second by second, but the gas price isn't updated at the pump that fast. What's the difference? 84.153.183.42 (talk) 16:39, 29 July 2010 (UTC)[reply]
There isn't a single, efficient market for gas, there are lots of different gas stations. If I want to buy a share in a company, I will buy the cheapest share on offer. If I want to fill my gas tank, I go to the most convenient gas station and pay whatever they are charging (within reason). --Tango (talk) 16:46, 29 July 2010 (UTC)[reply]
Gas prices do actually change every second, it's just the gas stations refuse to adjust the price because it would make people nuts. They do earn and loose money on the changes! They hate volatility for this reason. It's even worse for them - they may buy a truck full of gas for a high price, and the next day prices are lower, and they loose money on all the gas they are storing. And they reverse too, and they hope it works out long term. Ariel. (talk) 17:00, 29 July 2010 (UTC)[reply]
There is a constant influx of new information: all the buy and sell orders that are going on. Friday (talk) 16:43, 29 July 2010 (UTC)[reply]
Yeah, the price is determined by what people have recently traded it for. When the price fluctuates that means there have been trades and since different people have slightly different ideas about what the stock is worth the price will change. What people are paying in turn affects what other people think about the value. There is also new information all the time in the sense that lots of things can subtly affect the market and people are working very hard to analyze the incoming data and update their valuations. When the stock of some other company goes up, when a new trade agreement gets signed, when a new oil reserve is discovered, that can affect the value, even if the company has little to do with these other industries. There are things happening in the world all the time. Rckrone (talk) 17:13, 29 July 2010 (UTC)[reply]
Yes. Reported stock prices are snapshots of continual trading activities. The only way to get a prolonged flat "noise free" price would be for nobody to trade in that stock. Gas (petrol) stations that have electronic price displays can use them to play "gotcha" knowing that different market segments refuel at characteristic times e.g. commercial vehicles on weekdays, private vehicles at weekends. However changing the retail price too often would be counterproductive as bad PR. Cuddlyable3 (talk) 17:31, 29 July 2010 (UTC)[reply]
Also, consider some investor who holds 10,000 shares of BoringCo. It has had no news for a month and will have no news for another month. But that investor is also investing in and monitoring ExcitingCo, which you can find 20 news stories on every day. When news on ExcitingCo makes the investor think the shares are going to go up, he may sell 5,000 of his BoringCo shares to invest the money in ExcitingCo instead; and when ExcitingCo announces all their iPads are overheating, the investor may dump it all and invest the money back in safe old BoringCo. Multiply this by many investors and there's plenty of price fluctuation, though all this buying and selling of BoringCo has nothing really to do with news coming out of BoringCo. Comet Tuttle (talk) 18:08, 29 July 2010 (UTC)[reply]

This is really a question about human behaviour related to financial greed. That also applies where computers are assisting because humans have programmed the computers. It's all about people trying to gain the tiniest advantage, and then gain financially from it by multiplying that tiny advantage thousands of times. HiLo48 (talk) 19:49, 29 July 2010 (UTC)[reply]

Dog repellent

I need to keep dogs away from a part (not very large) of my garden and I'd like to find out if there are some substaces whose smell acts like a deterrent/repellent for them. Obviously not something armful. The best would be a "homemade" substance relatively odourless for humans but as disgusting as possible for dogs. --151.51.156.20 (talk) 18:27, 29 July 2010 (UTC)[reply]

I have seen some products sold, though I cannot recall the company. --Chemicalinterest (talk) 19:37, 29 July 2010 (UTC)[reply]
Suggestions I've heard include lemon slices, or ammonia, but the latter can make it pretty unpleasant for humans too. And I've had a beagle that decided he liked to eat citrus fruit, along with the million other things beagles typically found attractive. It can depend on the breed of dog, and what it is in that part of your gardent that could attract them. Scent hounds can be attracted to something that seems heavenly to them but quite unobservable by humans. Any human created product has to overcome a scent we can't even detect, quite a big ask. A bit of trial and error may be needed. Can't do a wire fence? HiLo48 (talk) 19:59, 29 July 2010 (UTC)[reply]
If the dogs are yours and it's acceptable to have them wear shock collars while outside, you could also bury a wire around the forbidden area and make an invisible fence. Comet Tuttle (talk) 20:06, 29 July 2010 (UTC)[reply]
And if the dog is not yours—or you just want to keep things simple—just build a visible fence. --Mr.98 (talk) 13:39, 30 July 2010 (UTC)[reply]

Double S(not T)P in one day

If the pressure of the atmosphere gradually increased to double its normal pressure within a day, what would be the effects on human life and industry? --Chemicalinterest (talk) 20:36, 29 July 2010 (UTC)[reply]

I think it would still be safe to breath, SCUBA divers breath standard air at that pressure all the time.
The boiling point of water would go up by about 30 degrees C. So your spaghetti would cook a lot faster.
Fires would probably burn hotter and stronger with twice as much oxygen per unit volume of air. You can imagine that both these things would effect large industrial processes like power generation. Not all of them would be able to adapt. APL (talk) 21:10, 29 July 2010 (UTC)[reply]
If the boiling point went up, wouldn't spaghetti take longer to cook and not cook faster? ~AH1(TCU) 00:52, 2 August 2010 (UTC)[reply]
Climbing Mount Everest would be a walk in the park. HiLo48 (talk) 21:13, 29 July 2010 (UTC)[reply]
Hmm I don't know about your last point. I still think it will be hard to climb 20,000 feet even though you could breathe easily. --Chemicalinterest (talk) 21:49, 29 July 2010 (UTC)[reply]
True, but, as mountains go, Everest is apparently quite an easy one to climb. You can basically walk up most of it. The cold, wind and lack of oxygen are the main problems, rather than the physical act of climbing that high. --Tango (talk) 00:15, 30 July 2010 (UTC)[reply]
You'd need to put more helium in your Zeppelin, but once you did that it would have greater carrying capacity. APL (talk) 22:14, 29 July 2010 (UTC)[reply]
You'd get a lot more horsepower out of your car too - no need for a turbocharger! But even if humans could endure that - I'm sure that there would be abrupt and spectacular decline in animal and plant populations for one reason or another. It's hard to predict exactly what all of the details might be but I would guess that forest fires would be much more common - and insanely difficult to control, more gasses would dissolve into the oceans - and that could change their pH and cause all manner of other nastiness for oceanic life. It's pretty certain that it would lead to total disaster of one kind or another within a matter of months. SteveBaker (talk) 22:25, 29 July 2010 (UTC)[reply]
Be sure to check your tire pressure before trying out your new-found horsepower! You'll find it's nearly 15psi lower than it should be. APL (talk) 00:04, 30 July 2010 (UTC)[reply]
Not overnight, but eventually you'd get really big bugs. Most insects rely on diffusion (rather than an active circulatory system) to provide oxygen to their tissues and this is a key factor in determining their maximum sustainable size. Increasing the pressure of oxygen will allow insects to evolve large sizes. Dragons flight (talk) 22:43, 29 July 2010 (UTC)[reply]
As evidenced by the giant insect fossils found from times in the Earth's past where the proportion of oxygen was much higher, eg. the Meganeura. Not all scientists agree about the role of higher oxygen in allowing large insects to survive, though. (That article gives some details.) --Tango (talk) 00:15, 30 July 2010 (UTC)[reply]
Well that's convinced me not to follow through on my evil plan to double air pressure and thus rule the world. -mattbuck (Talk) 19:30, 30 July 2010 (UTC)[reply]
Major changes in weather and climate. The amount of solar heat escaping to space would be greatly reduced, so the Earth would get quite a bit hotter and wouldn't cool nearly as much at night. Looie496 (talk) 00:13, 30 July 2010 (UTC)[reply]
The troposphere would expand, making the tropopause higher, and thus producing taller thunderstorms, larger hail, more intense lightning, etc. Sea levels would likely initially fall due to higher surface pressure exerted downward (can anybody confirm?). The pressure gradient would likely be greater, making for stronger winds. ~AH1(TCU) 00:52, 2 August 2010 (UTC)[reply]

what type of endocytosis is this?

Surely it's not phagocytosis right? These are lung cancer a549 cells, not white blood cells. (Watch the gold particle in the upper left corner of the centre region.)

John Riemann Soong (talk) 22:12, 29 July 2010 (UTC)[reply]

Image converted to link. Animated gif was more than 2MB — that's rather large to include inline. TenOfAllTrades(talk) 23:34, 29 July 2010 (UTC)[reply]

helium,

How many pounds of helium is necessary to support an object with a weight of 200lbs,utilizing one square meter of material filled with the gas?The height of the object is six feet.What type of material should be utilized to constrain the pressure? The apposing force to the object is water. Thank you Sincerely, Tom Ambron email {email removed} —Preceding unsigned comment added by 75.82.85.84 (talk) 23:08, 29 July 2010 (UTC)[reply]

We do not respond by email here at the ref desk -- you'll just have to check back. DRosenbach (Talk | Contribs) 23:22, 29 July 2010 (UTC)[reply]
Do you mean that you wish to construct a balloon with 1 square meter of surface area? This will limit you to a maximum sized balloon based on the surface-area of a sphere (the most efficient volume-to-surface-area structure). If you use a balloon, the necessary consideration is "how many kilograms of helium are needed to inflate the balloon? If the material is inelastic, you can fill it with helium at atmospheric pressure (or the pressure of the water at the depths you are considering). Then you can use the ideal gas law to calculate the mass of helium that will inflate the volume at that pressure. Now the question is whether the resulting density of helium is sufficiently low to provide 200 pounds of buoyant force. I think you need to restate the question - it is not really clear what you're looking for. We can solve all kinds of equations, but when the requirements are unclear, such solutions may be inapplicable. Nimur (talk) 23:33, 29 July 2010 (UTC)[reply]
It's simply not possible. Forget the helium, even the mythical vacuum balloon couldn't do it. If you mean that the balloon is a cubic meter, the displaced air weighs only 2.6 pounds. If you mean that the envelope is made of exactly one square meter of material, then the displaced air is only ¼ pounds! A balloon, no matter what it's filled with, cannot lift more than the weight of the air it has displaced.
You may be of the common misconception that helium has negative weight and that the more you cram into a balloon the lighter it gets, I'm afraid that's not true. Balloon lift comes from buoyancy, the same force that causes boats to float. A common analogy is that wood floats, but if you pile enough wood onto a ship, it will eventually sink. APL (talk) 23:58, 29 July 2010 (UTC)[reply]
Is there any theoretical particle that exerts negative gravitational force with which we could fill the ballon? —Arctic Gnome (talkcontribs) 01:19, 30 July 2010 (UTC)[reply]
No, there is no such particle. Antigravity is firmly in the realm of fiction. As far as exotic matter with negative mass, there is currently no consistent explanation for how such a particle would behave; there is no need for such a particle to play any role in the standard model; and any speculative theories about negative mass or repulsive gravitational force is exactly that - speculative. Nimur (talk) 04:00, 30 July 2010 (UTC)[reply]
The anticharm quark exerts negative gravity. --142.104.53.238 (talk) 22:50, 1 August 2010 (UTC)[reply]
I don't understand that bit about water. Are you saying the object is underwater? Helium at above atmospheric pressure would be worse than helium at atmospheric pressure, since there is more helium taking up the same amount of space, so it will weight more while displacing the same amount of air/water. The absolute best buoyancy you could get with 1m2 surface area would be a complete vacuum (you would need a strong container to hold a vacuum, which would therefore be heavy, but let's ignore that and assume a massless container). Such a vacuum-filled container submerged in water would be able to lift the weight of water displaced by the container. As Nimur says, the best shape is a sphere. The surface area of a sphere is and the the volume is . If we substitute A=1 into the first and rearrange, we get . We substitute that into the other formula and we get . That is 94kg or 207lb, so you could just lift your object. If we're using helium at atmospheric pressure, rather than a vacuum (which is wise, due to the weight of material required to cope with the pressure), then we have to minus off the mass of that helium, which is about 16 grams (if I've calculated it right - I expected it to be more, but I don't know why), so not significant. If I misunderstood your mention of water and your object is actually just in air, then you don't stand a chance. --Tango (talk) 00:06, 30 July 2010 (UTC)[reply]
Instead of using the ideal gas law directly, under standard temperature and pressure, one mole of any gas has a volume of just over 22.4 litres. Nitrogen-14 has a molar mass of 28 grams, oxygen-16 is 32 grams, helium-4 (which is mono-atomic) is 4 grams. Therefore air (80% nitrogen, 20% oxygen approx), has a density of 28.8 grams / 22.4 litre. A helium-balloon has a buoyancy of 24.8 grams/22.4litre or 1.1 grams/litre.
As for the water analogy, imagine an (theoretical) advanced life form that lives on the sea bed. Their density is well above that of water, so they can't swim up. If they want to lift an object to the surface of the water then they can attach a balloon to the object. As long as the object+balloon is less dense that water then it will rise. If the balloon is filled with vegetable oil (900grams/litre) then there is 100grams/litre of lift in water. The balloon still has real mass, it's just that the density is less that water, so it will rise.
BTW, a wooden boat, full of wood, will float, but very close to the water. It will act like a solid wooden object. In fact they were used for demining sea areas during WWII, see Naval_mine#Mine_breaking.
CS Miller (talk) 03:38, 30 July 2010 (UTC)[reply]
Sure, but you couldn't keep piling wood on indefinitely. Eventually the weight of the wood would exceed the boat's capacity and the boat would sink. (Once the boat fills with water and the cargo itself becomes buoyant, the comparison to a balloon in air breaks down, of course.) APL (talk) 00:29, 31 July 2010 (UTC)[reply]

July 30

in a cell culture medium, pH 7.4, will hydrogen peroxide oxidise colloidal gold?

I incubated a549 cells in 200 micromolar H2O2 for 90 minutes, washed the cells with fresh media and then observed them under DIC. (The cells had already been incubated in the presence of colloidal gold prior to incubation with H2O2.) The results were rather spectacular: an insane amount of intercellular bridges and connections (on all sorts of levels, criss-crossing everywhere) could be found, compared to situations without hydrogen peroxide. Oh was I happy!

But then 40-60 minutes into the experiment, my cells started dying from the bottom of the slide upwards. It was strange -- normally apoptosis is a stochastic event and is a rather random process that increases in chance as time goes by, but here it was like there was an invisible wave of apoptotic death slowly creeping up the slide. Is it nutrient exhaustion? I am sure I sealed the slide with nail polish to prevent evaporation.

A few papers I found said that H2O2 at 200 micromolar shouldn't decrease the viability of a549 cells too much...but they used 24h assays. Maybe in the time frame of a few hours, the H2O2 kills a lot of cells only to have the really mitogenic ones produce lots of viable ones?

Or maybe they are not apoptotic at all? The normal "apoptosis bubbles" were hard to see -- the entire cytoplasm changes viscosity almost -- it becomes very viscous and the cell's "insides" appear to slow down and look very different. Maybe it's the actin polymerising? I wonder if it's because the hydrogen peroxide is oxidising the colloidal gold, producing lots of Au(III) oxide and at the same time, decreasing the pH since Au(III) seems like a strong Lewis acid. John Riemann Soong (talk) 00:06, 30 July 2010 (UTC)[reply]

I thought that Au2O3 was insoluble, so the Lewis acidity was "locked away". --Chemicalinterest (talk) 11:58, 30 July 2010 (UTC)[reply]
Mmm, I didn't see any precipitate, but then again this is colloidal gold, not bulk gold, so Au(I) and Au(III) atoms would remain attached to Au(0) on the nanorod. I believe the sulfide thioether linkages popular for functionalising colloidal gold with are made by having the functionalised thiol precursor bind to the residual Au(I) sites (on an otherwise Au(0) rod) that weren't reduced in the initial manufacture of these nanorods. (Someone correct me if I'm wrong). John Riemann Soong (talk) 13:58, 30 July 2010 (UTC)[reply]
Oh it just hit me. According to the thioether article, they can be oxidised to sulfoxides and sulfones. I do pipette most of the old media (containing the nanorods) before I add the H2O2 but maybe I should do a few buffer rinses first? H2O2 won't react with membrane-bound gold rods inside endosomes in cells, right? Are sulfides bound to Au(I) more reactive or less reactive? Do they become more or less nucleophilic -- I mean, the Au(I) should sap some of the sulfide's nucleophilicity, right? (The linkage looks like: (rod)=Au(I)-S-(short alkyl/polyether chain)-COO-. John Riemann Soong (talk) 14:20, 30 July 2010 (UTC)[reply]


Also, why is the standard half-cell reaction potential for Au(III) + 3e- ---> Au(0) less than the reaction potential given for Au(I) + e- ---> Au(0)? (The latter has a higher potential than the two electron reduction of hydrogen peroxide in the presence of H+). I'm trying to figure out how favourable gold oxidation by hydrogen peroxide is, and I only spent a third of a semester on electrochemistry. John Riemann Soong (talk) 14:28, 30 July 2010 (UTC)[reply]

If bromine can oxidize gold to gold(III) bromide, then H2O2 should be easier. Also the colloidal property means it has much more surface area. --Chemicalinterest (talk) 15:12, 30 July 2010 (UTC)[reply]
Yeah but for the noble metals, halide compounds are more stable than oxide compounds. I think? Also from gold(III) chloride: "Reaction with reducing agents such as hydrogen peroxide or Fe2+ causes elemental gold to be precipitated from solution." Hydrogen peroxide is a reducing agent??? (My main issue is oxidation of the sulfur linkage -- if the sulfide is attached to an Au(I) atom, will hydrogen peroxide break the Au(I)-S bond, leaving free Au(I) and sulfonic acid? John Riemann Soong (talk) 16:41, 30 July 2010 (UTC)[reply]
A common example of H2O2 as a reducing agent is its reaction with NaClO, where it reduces Cl+ to Cl-, itself oxidized to O2. --Chemicalinterest (talk) 19:43, 30 July 2010 (UTC)[reply]

Stress Energy Tensor

I don't completely understand this. Why is this used instead of mass in General Relativity? What's the difference? ScienceApe (talk) 00:23, 30 July 2010 (UTC)[reply]

Because energy and mass both exert gravitational force; in Newtonian physics, mass is used in gravity calculations. The stress energy tensor (hint: read that article) takes energy and momentum into account; remember that according to General Relativity, gravity is a pseudoforce caused by curvature of spacetime. In most layperson explanations of GR gravity, it is explained that large masses curve spacetime, and that gravity is really just objects moving staight (as objects do), but doing so in curved spacetime. However, it turns out that mass is not the value which describes the curvature of spacetime, the stress energy tensor is the relevent value. --Jayron32 00:38, 30 July 2010 (UTC)[reply]
Momentum as well will curve space-time? So if we take an a 1kg object, it will curve space-time more if it's moving at 99% the speed of light? ScienceApe (talk) 01:16, 30 July 2010 (UTC)[reply]
Well, yes, but not because of momentum, per se, but because it has more energy, via KE = 1/2 mv2. Well, not exactly, but via the GR equivalent at relativistic speeds near the speed of light. Objects moving at a higher velocity have more energy, and thus curve spacetime more than than those which are stationary. Velocity affects energy, and energy is mass. --Jayron32 01:26, 30 July 2010 (UTC)[reply]
How does momentum bend space-time? ScienceApe (talk) 01:32, 30 July 2010 (UTC)[reply]
How? By inducing a noether current that must be counterbalanced in order to conserve mass/energy/momentum flux. Consequently, some other "thing" needs to either move or change velocity, or else the flux conservation is violated. But Newton tells us that things don't just start "moving" for no reason - we have inertia! A body at rest remains at rest! So, if we want to remain at rest, and also change our momentum, we need to start warping space so that maintaining a constant position yields a change of momentum - ergo, gravitation. The exact geometry of the warping is nontrivial - and requires solutions of the general relativistic equations - but in the simplest case, we see elliptical planetary orbits, - planets that are tracing out an <x,y,z,t> vector path, when in fact all good sense tells us that they should be inertially stationary. Nimur (talk) 04:32, 30 July 2010 (UTC)[reply]
In a bit more details on why the tensor includes momentum; its because there are three fundemental conserved quantities in any physical interaction between bodies: energy, mass, and momentum. This is true even in classical physics. So, when two objects interact, even classically, momentum must be conserved. Even under the Newtonian definition of gravity, as two objects interact with each other gravitationally, their total momentum must remain constant throughout the interaction. Think of it this way. If you are a chunk of space rock flying by the earth; as the earth slows you down by "pulling" on you via gravity, the earth itself must speed up by the exact same amount.
In GR physics, its FAR more complex, and requires some pretty advanced mathematics. However, the law of conservation of momentum still cannot be violated. So, any calculation of gravity must take momentum into account. The stress energy tensor itself is described by a matrix of values. It isn't a single number like "mass". It is a tensor, which is a multi-dimensional quantity. If you know what a vector is, you can think of a tensor as a vector on steroids. Where as a vector quantity has two values (origin and direction), a tensor can have dozens of values, which are manipulated via matricies. The stress energy tensor uses four "types" of values (though there are multiple groups of values for some of the types). There is the "energy density", the "momentum density", the "energy flux" and the "momentum flux". Basically, in order to correctly calculate the effect of gravity within a system, say a simple two-particle system, you need to know a) the mass-energy of each object (energy density) at your initial conditions b) the momentum of each object initially (momentum density) c) the mass-energy changes experienced by each object as they interact (or curve spacetime, whatever your perspective) (energy flux) d) momentum changes experienced by each object as they interact. Now, it is even more complex because each location in the matrix isn't a simple number like "three", but rather a complex polynomial which describes, say, the energy density of the entire system in three dimensions (that is, how much mass-energy is at every point in space in our system). This is of course a gross oversimplification, since you actually have to do the math to get the implications here. I can't do the math myself, really, but this is how I understand it as a layperson. Perhaps someone with a better physics background could fix my errors or explain it better. --Jayron32 01:44, 30 July 2010 (UTC)[reply]
And just as an addendum, you may want to read Stress-energy-momentum pseudotensor, which is the actual value used in GR. The stress-energy tensor has other uses, but it is not strictly applicable to GR apparently. Again, I lack the math to understand exactly why, but I do read english, and that's what the article seems to say. --Jayron32 02:00, 30 July 2010 (UTC)[reply]
Another way to rephrase Jayron's explanation: the stress energy tensor allows a physicist to describe the gravitational interaction of a body in the form of a matrix that describes all the relevant concepts - energy, momentum, position, and so on, in generalized coordinates. The way that the body behaves depends on the mathematical laws that are expressed as degrees of freedom of the system dynamic. In a simple case, like a ball rolling down a hill, the only relevant degrees of freedom are position (x,y,z); and the respective derivatives with respect to time (velocity in x, y, z). These parameters are constrained by Newton's laws of motion - a set of empirically derived relationships between position, velocity, and acceleration due to gravity. General relativity allows the physicist to consider these relationships in a more general way, (hence "general"); part and partial to this generalization is that the position, velocity, and acceleration must be described only in a relative way, and not with respect to any fixed frame. Now, there are lots of relativistic theories, including some that are classical and consistent with Newton's laws - Galilean relativity, for example. But if you make the theory extremely generalized, you can no longer make the relationships simple: "F = m a" is not a general description of a gravitating system, because Force due to Gravity is a sort of "imagined" quantity that Newton invented. (Sure, it mathematically explains the rate and distance at which the moon circles the earth, but it doesn't do a very good job at explaining why the moon keeps magically changing direction instead of flying off in a straight line). So, general relativity seeks to eliminate that unnecessary invention by reformulating the relationships between the free parameters (position, velocity, mass, energy, and so on). The only sensible way to reformulate those relationships and still describe the kind of behavior we observe is to write out a stress energy tensor in a generalized coordinate space. The result is a description of gravity without any "spooky action at a distance" - in fact, the solution to the general relativistic equations that describe a planetary system yield accurate predictions about orbits; and do not rely on any weird spurious "force" of gravity. Gravitation is instead a property of the system - it is a description of the way the masses influence each other. Nimur (talk) 04:12, 30 July 2010 (UTC)[reply]

Piper nigrum and alcohol

So I was making steak au poivre the other day, and once the steaks were seared and removed from the pan, the recipe called for cognac to be added to the still-hot pan and the vapors ignited. Quite the impressive show resulted---the flames themselves were only about a foot tall, but the plume of hot air was at least six feet tall. I don't really want to recreate that indoors, but I don't really want to have to find an outside area free of flammables before I can make the dish. I'm wondering if I can skip the flashy vapor-igniting and just cook the alcohol off, though I'm concerned that without that step that some of the harshness of the cognac will remain. I certainly don't want to skip the alcohol both for its deglazing and solvent usefulness. Thanks for the help!72.219.136.28 (talk) 06:37, 30 July 2010 (UTC)[reply]

You should be okay to simply cook out the alcohol, if you make some changes. Igniting the flames like that serves two purposes: first, it is theatre (i.e. simply a show); second, it serves to remove the alcohol quickly. Depending on what else you plan on adding to the dish, you may want to significantly reduce the temperature to avoid burning something during the longer cook time. Incidentally, the alcohol never cooks "out" completely. Well, never is a long time, but you catch my point; don't try to just cook out (or flambé out!) alcohol for someone who is intolerant or something. Matt Deres (talk) 13:58, 30 July 2010 (UTC)[reply]
My wife (who is French and therefore "correct-by-definition" about all culinary matters) says that you're probably using WAY too much cognac. To cook a couple of steaks "au-poivre", she'd first cook the steaks in cracked soft/green peppercorns with a simdgeon of butter and ground black pepper until they are essentially "done". Then she'd use just one tablespoon of cognac for two steaks - it's a strong flavor and you don't want to overpower the cream and make it bitter. The flames are generally rather unimpressive, particularly because alcohol flames are almost invisible. It's hard to get the cognac to ignite in the pan - so she lights it in the spoon and gently pours it onto the steaks while it's still burning. Don't pour straight from the bottle into the pan! She douses the pale blue flames very quickly by pouring on the cream - then turns off the heat and allows maybe 10 more seconds in the pan to get the cream hot and mixed in with the cognac, pepper and meat juices - and then serves it straight to the table. But you do need to burn off the alcohol - it has to be added towards the end so that the flavors in the cognac don't get destroyed by the cooking - and you don't want to overcook everything else while you wait for the alcohol to simply evaporate off. If you are getting a gigantic and colorful fire - then you may have set the fat from the steaks on fire - and that's "A Very Bad Thing" for both safety and flavor. Since you're going to be serving the sauce from the pan - and you don't want a lot of ikky beef fat in it - if there is a lot of fat in the pan - then remove most of it before adding the cognac. She suggests not cooking more than a couple of steaks in a pan - if you need to cook more then two or three steaks then use a second pan so that you can control everything more closely in that busy half minute at the end of cooking. Also, avoid leaving the steaks too rare with this recipe because the blood in the cream doesn't look appetizing - so limit yourself to medium-rare. The plates you serve onto need to be really hot. She uses a sprig of parsley to decorate (and, I suspect, to meet the "7 ingredients rule" of French cooking that I've deduced from watching her cook!) Because the timing of all of this is so important, she tends to serve vegetables that can stand being a little over-cooked - asparagus is generally her choice - because then she can give full attention to the steaks without having to panic about not over-doing the veg. SteveBaker (talk) 14:32, 30 July 2010 (UTC)[reply]

Very helpful advice! I think in the future I'll probably just "cook it out" to use the term instead of the big old show. Mr. Baker, you have a good point about the color of the flame---it was orange, and even though I poured off the excess fat, there was likely still some in the pan. There was 1/3 cup cognac (I used brandy, but anyways) in the recipe, so I think I agree on the sentiment of using too much of it. Not that I really tasted it at all...anyways, thanks so much!72.219.136.28 (talk) 21:13, 30 July 2010 (UTC)[reply]

An orange flame definitely indicates that something OTHER than the alcohol was causing the fire. Alcohol flames are notoriously hard to see...and when you can see them, they are pale blue. I think you had a fat fire - possibly exacerbated by the cognac. When a liquid flash-boils because it's hitting fat that's higher than the liquid's boiling point then it causes an 'aerosol' effect that sprays a fine mist of hot oil into the air. If there is a source of ignition (and you handily provided one when you tried to set light to the cognac) - then you have a fat fire, which can be almost explosive. Perhaps you need to turn down the heat and let things cool off a little before you add the alcohol? SteveBaker (talk) 06:06, 31 July 2010 (UTC)[reply]
And if you're cooking on a gas range, please make sure you turn it OFF before you start adding combustibles like alcohol! Matt Deres (talk) 14:50, 31 July 2010 (UTC)[reply]

Quantum Mechanics Problem

I'm working through a quantum mechanics book and have encountered a problem with one of the exercises and I need a little help. The problem defines a phase flipping operator in d dimensions to be a unitary operator with eigen values +1 or -1. Then, it wants me to show what happens if we take the whole collection of such operators relative a specific shared basis of eigen vectors and average their effect on a a density operator. However, by linearity, this should be the same as applying the average of the unitary operators to the density operator, but since there will be as many +1's as -1's for any eigen |k>; the overall operator appears to average to 0. This does not seem to be the answer the book wants since it says that it should preserve the populations of the density operator, but wipe out its coherencies. Thus, I suspect I am missing something really simple or the book is wrong, can anybody point me to which one of these is the case; if the former, can I have a small hint as to what I am missing. Thank you:) 66.202.66.78 (talk) 09:29, 30 July 2010 (UTC)[reply]

Nevermind, being half asleep, it somehow seemed reasonable to average the unitary's, get 0, then apply them; obiviously that does not follow from linearity. Instead, I should obtain that only the diagonal of the density matrix remains:) 66.202.66.78 (talk) 09:39, 30 July 2010 (UTC)[reply]

practical consequences of existence

philosophically, existance in an open question. what are the practical consequences of whether I exist? Does it really mean anything for me if in the end it is proved/discovered that I don't? —Preceding unsigned comment added by 92.224.207.77 (talk) 10:14, 30 July 2010 (UTC)[reply]

http://plato.stanford.edu/entries/existence/ --Zomzom12 (talk) 13:14, 30 July 2010 (UTC)[reply]

Philosophers are a waste of quarks. What is the definition of the word "exist"? Sadly, it's not a very well defined concept. Wiktionary says "to be; have existence; have being or reality". So what is "reality"? Well, again, Wiktionary comes to the rescue with:
  1. The state of being actual or real.
    The reality of the crash scene on TV dawned upon him only when he saw the victim was no actor but his friend
  2. A real entity, event or other fact
    The ultimate reality of life is it ends in death
  3. The entirety of all that is real.
  4. An individual observer's own subjective perception of that which is real.
If we discount circular definitions of reality as "that which is real", then we're left with only the last (and still somewhat shakey) definition - and since your own subjective perception of reality is that you exist - it is completely impossible for someone to prove to you that you don't exist. It might be possible to prove to me that you don't exist (Maybe you're an AI program designed to test reference desk response times!) - but that cannot ever (by the very definitions of the words) prove to you that you don't exist. Since it can't be proved - the question of what that might mean for you is entirely moot.
Philosophers love to argue about this circular definition of what words in English mean - but, as I said, they are a waste of quarks and we may safely ignore everything they say.
SteveBaker (talk) 14:04, 30 July 2010 (UTC)[reply]

Wow, you are very dismissive of philosophy... Ironically, you yourself are engaging in philosophical argumentation above. Best regards. Zomzom12 (talk) 15:29, 30 July 2010 (UTC)[reply]

Ah - but I'm not pretending that anything terribly deep is going to come out of this. I'm merely pointing out that it can only be a matter of linguistics. If these words mean X then Y is true because that's how the words are defined. This is only a discussion about the meaning of words. If I were to define the word "existance" as "Having an account on eBay" - then the conclusions of the debate would be wildly different. Rational debate can only be undertaken when everyone agrees on the definition of the words being used because these are (mathematically speaking) a part of the axioms of the theory you're trying to prove. The big philosophy debate on the nature of existance isn't anything to do with producing some deep understanding of the universe or the human condition - it's just a matter of defining a tricky word. SteveBaker (talk) 05:58, 31 July 2010 (UTC)[reply]
Cue the cartoon showing why engineers aren't invited to philosophers' parties. Comet Tuttle (talk) 17:09, 30 July 2010 (UTC)[reply]
He was engaging in philosophical argument in order to demonstrate how pointless it is. Philosophy was replaced centuries ago by science, but there are still a small number of philosophers that keep going for no gain whatsoever (well, they get paid, so I suppose that is a gain for them). --Tango (talk) 17:32, 30 July 2010 (UTC)[reply]
If you don't exist, how could you have asked this question? Looie496 (talk) 17:13, 30 July 2010 (UTC)[reply]
I think that the subject you are talking about is ontology. Ontology may have some ethical consequences. For example, imagine a nearly-perfect computer simulation of a universe with some humans in it. Does their "ontological inferiority" mean that they don't really exist, or do ethical obligations extend to them? Paul (Stansifer) 17:24, 30 July 2010 (UTC)[reply]
That doesn't mean we need philosophers to debate it endlessly, we would just need to make a decision. It's no different from decisions that politicians make every day (assigning value to different people's lives) and we don't rely on philosophers for those decisions. --Tango (talk) 17:32, 30 July 2010 (UTC)[reply]
There is no word "existance" though it sounds a bit like the British punk band Exitstance. Wikipedia has an article about the metaphysical concept Existence.
Existence is apparent only in terms of interactions with an Observer. Thus a table does not exist to a blind man until he bumps into it. The OP did not exist (as OP here) to us until we observed his/her question. Some practical consequences of the OP existing are that we pay him/her attention and he/she pays us attention. If we discovered that the OP does not exist, which could happen if someone found that the question was posted not by a person but by a random sentence generating machine, it would mean our interactions would change. I might conclude I have better things to do than talk to a machine. The machine wouldn't notice anything except that its battery would eventually run down and probably not be replaced. Cuddlyable3 (talk) 18:36, 30 July 2010 (UTC)[reply]
But that example isn't really true. The table does exist for the blind man because it exerts a slight gravitational pull on him - it deflects and absorbs sound waves and thereby alters the acoustics of the room - it also alters the behavior of other people who live in the house - if there is a table, they might eat in that room, and if not then they wouldn't. That too would have subtle ramifications for the blind man, and that change affects him even if he isn't actively aware that the table is there. There is a measurable difference on him because the table is there. So you can't say that it doesn't exist for him.
Because every atom in the observable universe eventually affects every other one (to an admittedly small degree) everything 'out there' interacts with us as observers. Since there are no totally unobserved things, you can't say that anything in the universe doesn't exist for us using a clear black-and-white definition. To make this point about the table not existing for the blind man until he bumps into it, you have to set an arbitrary threshold for how much it's allowed to influence him without it "existing for him". Maybe objects that produce less than 0.0000000001 g's of gravity and reflect electromagnetic radiation at us at less than lumen, etc, etc can be said to "not exist" for us...but that's an exceedingly unsatisfying answer. Nobody is ever going to agree on such an arbitary threshold for "existence". SteveBaker (talk) 05:58, 31 July 2010 (UTC)[reply]
Philosophers don't debate it endlessly; they mostly talk about other things. Computer scientists don't talk about ones and zeroes forever, and historians do stuff other than memorize dates. (However, I now wish I'd taken that philosophy class on nothingness in college, since it turns out that there are a lot of different varieties of nothingness, absence, and the void in computer science.) And politicians might benefit from paying attention to what the philosophers have to say! Well, at least humanity might benefit. Paul (Stansifer) 18:57, 30 July 2010 (UTC)[reply]
Endlessly does not mean constantly. The debate on the nature of existence hasn't ended, has it? --Tango (talk) 20:45, 30 July 2010 (UTC)[reply]
They don't ever seem come to any widely agreed conclusions - so there is no end to their discussions. SteveBaker (talk) 05:58, 31 July 2010 (UTC)[reply]
All that money I spent on a philosophy course and what have I learned about? Nothing! Cuddlyable3 (talk) 21:01, 30 July 2010 (UTC)[reply]
Take a look at existentialism, consciousness, reflexive self-consciousness, homunculus argument, qualia, and metaphysical solipsism. And...sigh. Metaphilosophy. ~AH1(TCU) 00:37, 2 August 2010 (UTC)[reply]

Misinterpretation of UN population report?

Has the media and the public completely misinterpreted the findings of the UN report "World Population in 2300" (http://www.un.org/esa/population/publications/longrange2/longrange2.htm)?

Here are some quotes:

"Fears that the human race may drive itself to extinction through over-population are challenged by new United Nations figures suggesting an eventual equilibrium as people in poorer countries come to understand the need for smaller families. " http://www.guardian.co.uk/world/2004/nov/06/population.brianwhitaker

"The world's population will stabilize in 300 years to about nine billion people, with an average life expectancy of 95 years, the United Nations predicts." http://www.cbc.ca/world/story/2004/11/05/population_041105.html

All the Report says is that "According to the medium scenario, world population rises from 6.1 billion persons in 2000 to a maximum of 9.2 billion persons in 2075 and declines thereafter to reach 8.3 billion in 2175. The return to replacement level fertility coupled with increasing longevity in the medium scenario produces a steadily increasing population after 2175 that reaches 9 billion by 2300. If the effects of increasing longevity are counterbalanced by fertility, population size remains constant at 8.3 billion from 2175 to 2300 as in the zero-growth scenario (figure 1)."

Does this mean that this medium scenario is the most likely?

Thanks for your help!

Zomzom12 (talk) 12:37, 30 July 2010 (UTC)[reply]

I don't see that implication anywhere in the report. SteveBaker (talk) 13:51, 30 July 2010 (UTC)[reply]
Thank you for the input. Zomzom12 (talk) 15:30, 30 July 2010 (UTC)[reply]
Predicting something 300 years in the future is pretty much impossible. I can't imagine why they would even try. The biggest flaw is usually that they extrapolate existing trends without accounting for changes (not that they could). Another common error is mistaking Exponential growth for Logistic growth. Ariel. (talk) 21:32, 30 July 2010 (UTC)[reply]
A world population near nine billion sometime this century and again in 300 years does not imply the population stablizes at that level in 300 years. See also our articles Malthusian catastrophe and population growth. ~AH1(TCU) 00:28, 2 August 2010 (UTC)[reply]

canteloupe

how can you tell when a canteloupe from the garden is ready to be picked?--Horseluv10 13:02, 30 July 2010 (UTC) —Preceding unsigned comment added by Horseluv10 (talkcontribs)

http://www.youtube.com/watch?v=11-XzxukFoM --Zomzom12 (talk) 13:11, 30 July 2010 (UTC)[reply]

To summarise that video (since Horseluv has previously implied they are confined to a Whitelist that only allows Wikipedia).
  1. The cantaloupe will get paler until it isn't green anymore, but rather a more buff-like colour. This will presumably vary between varieties.
  2. The cantaloupe will start to smell of melon.
  3. The melon will start to separate from the stem, so they no longer look like the stem and melon are all one piece.
  4. You will be able to separate the cantaloupe from the stem with very gentle pulling/pushing. If you have to pull properly, or cut the melon off the stem, it isn't ripe.
Anyway, that's what the video says, while showing you so that it's clearer. 86.164.66.83 (talk) 14:52, 30 July 2010 (UTC)[reply]
As a onetime seller of the melons, I agree that color change is one sign of a canteloupe being ripe, but the smell of ripe melon is the best indication. Unlike watermelons, the sound when it is thumped does not become deep and dull. Edison (talk) 23:18, 30 July 2010 (UTC)[reply]
thanks for the info from the video. i couldnt get onto it. im blocked.--Horseluv10 01:10, 31 July 2010 (UTC) —Preceding unsigned comment added by Horseluv10 (talkcontribs)
The video is a great guide for growers to decide when to pick. Smell is the best guide for a consumer to decide which canteloupe to buy, since you can't tell, in the store, what the vine looked like. Edison (talk) 04:14, 31 July 2010 (UTC)[reply]

Atomic theory

Atomic theory tells us that valence electrons in an atom are the particles that are mostly involved in many phyisical or chemical activities of substances. In lasers, electrons move from one energy level to another and thereby releasing coherent and monochromatic photons. During chemical reactions,valence electrons move from one atom to another and thereby changing the properties of the substances involved.Duruing charging,substances lose or gain electrons.DOES AN ELECTRON MOVE AWAY FROM ITS MOTHER ATOM WITHOUT ITS MOTHER ATOM COMING TO CONTACT WITH AN ATOM THAT THE ELECTRON IS INTENDING TO GO TO.JUST AN ELECTRON(ALONE) MOVING AWAY FROM ITS MOTHERATOM,POSSIBLY IN A VACUUM. —Preceding unsigned comment added by Mlondo (talkcontribs) 14:37, 30 July 2010 (UTC)[reply]

I have added a header separating this question from the one above. 86.164.66.83 (talk) 14:43, 30 July 2010 (UTC)[reply]
I think that the electrons separate in a plasma. --Chemicalinterest (talk) 14:58, 30 July 2010 (UTC)[reply]
Are you talking about chemical reactions, electrons moving in metals, or lasers? John Riemann Soong (talk) 15:01, 30 July 2010 (UTC)[reply]
The OP was asking whether there is an instance that an electron moves away from the mother atom without an ionic bond occurring. I responded by saying that I think in a plasma the electrons are stripped from the mother atom and sort of float around it. --Chemicalinterest (talk) 15:18, 30 July 2010 (UTC)[reply]
The OP describes well what happens in a metal conductor. Atomic nucleii rest in fixed positions while electrons can drift in the intervening space. An electric current occurs when electrons move from orbiting one nucleus to orbiting a neighbouring nucleus. Cuddlyable3 (talk) 16:15, 30 July 2010 (UTC)[reply]
It does happen in a metal to and is the basis of electrical conductivity; this moving away of electrons from mother atoms without a redox reaction going on. An exception would be electrical conductivity in ionic solutions. --Chemicalinterest (talk) 12:26, 31 July 2010 (UTC)[reply]
The OP describes a standard gas-phase ionization reaction: see ionization energy for more details. Physchim62 (talk) 14:57, 31 July 2010 (UTC)[reply]
Maybe in cases of positron radiation where the β+ particle comes into close contact with an atomic electron? ~AH1(TCU) 00:10, 2 August 2010 (UTC)[reply]

water

Ages ago I saw someone with a glass of water and a weird device with two metal prongs which they put into the water. Then the water started to go grey. I asked them what the fuck they were doing and they said it was putting silver into the water and then they'd drink it for health benefits. What the hell were they doing? —Preceding unsigned comment added by Fdge45y4uwuetj (talkcontribs) 15:01, 30 July 2010

I think it was an electrolytic apparatus. --Chemicalinterest (talk) 15:03, 30 July 2010 (UTC)[reply]
What makes you think it was anything other than putting silver into their drinking water? While unconventional, this is not really that weird considering the other weird stuff people consume in the name of health. I would suspect the 'device' was some sort of soluble rod that allowed for easy introduction of silver into the water. --144.191.148.3 (talk) 15:05, 30 July 2010 (UTC)[reply]
Here it mentions electrolytically dissolved silver. --Chemicalinterest (talk) 15:09, 30 July 2010 (UTC)[reply]
It may well have been exactly what they said. Silver sometimes used as a quack medical cure-all, (Presumably because of its weak antiseptic abilities.) See : Colloidal_silver#Current_alternative_medicine_use
Fun Fact : Overuse of silver 'supplements' can turn your skin blue. See : Argyria
APL (talk) 15:38, 30 July 2010 (UTC)[reply]
Fun idea: Argyria turns skin blue where it is exposed to light. Could sitting in front of a slide projector eventually produce a skin tattoo of a photograph? Cuddlyable3 (talk) 16:00, 30 July 2010 (UTC)[reply]
What happens when the sun shines on your tattoo? You would need a "fixer" to lock the color in so additional light will not make the whole area blue. --Chemicalinterest (talk) 19:59, 31 July 2010 (UTC)[reply]
Seems "safer" to do it topically in a small area...obviously "do not try this", but paint on a solution of some silver salt, expose self, wash off. Essentially make yourself into a photographic plate. DMacks (talk) 19:41, 30 July 2010 (UTC)[reply]
Lest you think I'm just proposing bad ideas, the idea of using a slide projector to trigger photochemical reactions that produce an image of what was projected is actually pretty cool! One neat application is DOI:10.1021/ed800170t. DMacks (talk) 19:46, 30 July 2010 (UTC)[reply]
After reading your first two sentences and before reaching your third sentence, I was sure they were using one of those portable, unsafe, electric gadgets that you can put in a mug of water to boil it. You plug it into household AC, and it boils the water super-quick. Of course, if you drop the gadget and the two prongs touch your skin, you're then plugged into household AC yourself. Comet Tuttle (talk) 16:57, 30 July 2010 (UTC)[reply]
I don't think those "water heaters" have live AC current. If it was just two electrodes, then it would fizzle the water furiously, creating a highly flammable oxyhydrogen mixture. --Chemicalinterest (talk) 21:04, 30 July 2010 (UTC)[reply]
I have a humidifier that works like that. It's simply two electrodes directly connect to power, and in the instructions they say to add a little salt to the water. Ariel. (talk) 21:27, 30 July 2010 (UTC)[reply]


Silver is antibacterial, so it makes sense that they would want it. --138.110.25.31 (talk) 19:37, 30 July 2010 (UTC)[reply]

Well, it doesn't "make sense", because silver doesn't kill billions of bacteria on contact in the way that antibiotic ointment does. Comet Tuttle (talk) 20:09, 30 July 2010 (UTC)[reply]
Read medical uses of silver. Even the WHO adds silver ions to drinking water. --138.110.25.31 (talk) 20:12, 30 July 2010 (UTC)[reply]
Ok 138.110.25.31! Let's read that article!

At present, there are no evidence-based medical uses for ingested colloidal silver. There are no clinical studies in humans demonstrating effectiveness, and a few reports of toxicity. The U.S. National Center for Complementary and Alternative Medicine has issued an advisory indicating that the marketing claims made about colloidal silver are scientifically unsupported, and that the silver content of marketed supplements varies widely and that colloidal silver products can have serious side-effects to the consumer, including "argyria,... neurologic problems (such as seizures), kidney damage, stomach distress, headaches, fatigue, and skin irritation. Colloidal silver may interfere with the body's absorption of some drugs, such as penacillamine, quinolones, tetracyclines, and thyroxine."

Any questions? WHO does not use it for medicinal purposes. They do sometimes use it to disinfect contaminated water in third-world countries, but in western countries with good public water systems that doesn't apply at all. APL (talk) 00:24, 31 July 2010 (UTC)[reply]

Isn't silver poisonous? Count Iblis (talk) 23:06, 30 July 2010 (UTC)[reply]

Yes. It is. Alternative medicine fads are often dangerous. APL (talk) 00:24, 31 July 2010 (UTC)[reply]
No; silver is not poisonous. It is an unreactive element. Silver compounds are poisonous.[citation needed]Yeah I know--Chemicalinterest (talk) 00:53, 31 July 2010 (UTC)[reply]
Elemental silver can cause a number of health problems. The bit I quoted above mentions some of them, the articles mention more. (Besides, if it's so 'unreactive', how's it killing all that bacteria in third world water supplies?)
Please don't post things that you thought up off the top of your head. Especially when the actual facts are so close at hand. APL (talk) 19:29, 31 July 2010 (UTC)[reply]

Silver kills bacteria and is therefore useful as a disinfectant/antiseptic. --138.110.206.101 (talk) 01:25, 31 July 2010 (UTC)[reply]

Perhaps, a bit anyway. But you don't drink disinfectants! APL (talk) 03:40, 31 July 2010 (UTC)[reply]

Silver nanoparticles can be poisonous due to the presence of residual Ag+. John Riemann Soong (talk) 03:15, 31 July 2010 (UTC)[reply]

One additional word of caution about colloidal silver and gold is that when it changes your skin color, it does so permenantly. There is no cure and it doesn't fade or dissipate over the remainder of your life. Since there is zero evidence of beneficial effects of putting the stuff into your body - you'd definitely have to be crazy to take the stuff. The antibiotic effects of silver outside of the body are really rather mild - not even a fraction as good as modern antibiotics. SteveBaker (talk) 05:13, 31 July 2010 (UTC)[reply]
But don't you want your cells to be all bright and sparkly like these a549 cells? John Riemann Soong (talk) 05:44, 31 July 2010 (UTC)[reply]
My opinion is that the disinfectant in colloidal silver is too weak to be of any use in vivo. --Chemicalinterest (talk) 11:02, 31 July 2010 (UTC)[reply]
ChemInt, I don't mean to be rude, but no one cares what your opinion is. Not in the slightest. APL (talk) 19:29, 31 July 2010 (UTC)[reply]
I don't care. But since I don't know (haven't read the studies), I can only have an opinion. --Chemicalinterest (talk) 19:57, 31 July 2010 (UTC)[reply]
I did read a few studies while trying to mediate a wiki-dispute once, and my slightly-informed opinion matches the mainstream view - colloidal silver is utter crap and relies on confusing people with studies that examined ionic silver. Franamax (talk) 21:16, 31 July 2010 (UTC)[reply]
Then why would you imagine you needed to answer? I have an opinion on lots of things I haven't got evidence or sources for, but I'm not going to offer it when there are plenty of people offering more informed views. If those people do not present themselves, the second best option is to inform myself so that I can give an actually informed answer with sources. Giving an uninformed opinion is absolutely the last-ditch option when I can't find any information and nobody else has offered anything good and the question has been up for days. And even then, it has to be an opinion that might lead them to greater understanding of the topic, swathed in acknowledged limits. 86.164.66.83 (talk) 01:25, 1 August 2010 (UTC)[reply]
Except in burn treatment where for instance "Silver sulphadiazine continues to be the antimicrobial agent most often used in burn care facilities" [12] and silver ions counteract problems with bacterial resistance.[13] Note this is not colloidal silver. Entire sheets of metallic silver used to be used for burn treatment too, and I think there may even still be active research on this. Franamax (talk) 21:12, 31 July 2010 (UTC)[reply]

Flossing

I am writing a work of historical fiction surrounding the invention of flossing. The novel begins when the protagonist notices that on some days her teeth are very tight and it's impossible to get the floss between them without breaking it (the floss), while on other days it just glides through with no problems. She sets out to discover whether it's possible/normal for the teeth to cycle through this kind of tightening and relaxation and what it's called. Anyone know? Thanks. --Sean 16:59, 30 July 2010 (UTC)[reply]

You ask a good question. It's an evidence-based assertion that occlusion, or the manner in which teeth of the upper arch meet with the teeth of the lower arch, changes ever so slightly throughout the day. Normal physiology dictates that hard tissue join not directly to other hard tissue but rather via ligaments. Teeth are no exception, and they are attached to the alveolar bone via fibers of the periodontal ligament. As such, there is some physiologic mobility to healthy teeth (contrast this to dental implants which, when properly osseointegrated into the jaw bone, fuses directly to bone and exhibits no mobility. So, to respond to your question directly, I cannot say that I've heard of any research on that front, but it's certainly biologically plausible for contact between teeth to shift enough for a layman to detect with a floss test -- I question if the magnitude of change would be enough to tear the floss one day and allow it to pass though, as you described it, a nearly open contact. Moreover, I have a pamphlet that describes the invention of floss, and if you email me and give me a few days, I'll scan it and send that to you. DRosenbach (Talk | Contribs) 20:23, 30 July 2010 (UTC)[reply]
Thanks so much for the thorough answer. I'd never even considered that teeth were held in by ligaments, though that makes sense, of course. Don't bother with the scan, but thanks; I've got enough to go on about the variable tightness issue from here. DRosenbach, I've thoroughly enjoy your dental-themed contributions to these desks over the last year or so. Cheers! --Sean 21:25, 30 July 2010 (UTC)[reply]
(non EC-flagged EC) Whew. I can't think of anything that would cause that particular cycle, though hopefully our resident dentist can come up with something. I hope you don't mind me saying this, but that seems like a really bizarre way to start... anything. Seriously - the central problem of the story is why floss sometimes gets stuck? Good luck with the publishers, my friend. :-) Matt Deres (talk) 20:26, 30 July 2010 (UTC)[reply]
Oh, I assure you it will be a ripping yarn as we watch our intrepid flosser invent all manner of ludicrous backstories as she outwits the various bureaucratic regimes that stand in the way of her idle wonderings, even when she agrees with those regimes in principle! --Sean 21:25, 30 July 2010 (UTC)[reply]
Yeah, when you're heroine is busy being subtle, make sure she doesn't push her luck by gloating in the presence of people who played along with her. That wouldn't make her a terribly sympathetic protagonist. 86.164.66.83 (talk) 23:37, 30 July 2010 (UTC)[reply]
... I was actually wondering if this was just an elaborate ploy to circumvent our medical advice policy. "So I was writing a work of historical fiction where my protagonist's foot looks like this. Do you think it's infected?" Does dental advice fall under the desk medical guidelines? Nimur (talk) 23:55, 30 July 2010 (UTC) [reply]
It does, but I think this is one of those cases when we can AGF Nil Einne (talk) 11:03, 1 August 2010 (UTC)[reply]
Floccinaucinihilipilification? ~AH1(TCU) 00:05, 2 August 2010 (UTC)[reply]

paradoxical redox reactions

Oxidising and reducing power should be a transitive property, right? According to the article hydrogen peroxide: "In acidic solution, H2O2 is one of the most powerful oxidizers known—stronger than chlorine, chlorine dioxide, and potassium permanganate."

Why then does hydrogen peroxide reduce potassium permanganate in acidic solution? Hydrogen peroxide would appear to be less reactive in alkaline solution (what with it being deprotonated and all) yet it would oxidise in alkaline solution? How can an oxidising agent A be a more powerful oxidiser than an oxidising agent B at say, pH 1.5 and yet get oxidised by agent B? John Riemann Soong (talk) 17:00, 30 July 2010 (UTC)[reply]

(That section is missing info [14], however this doesn't affect the answer)
Kinetics is the key here when considering products - since the reaction of H2O2 >>> 2H+ + O2 + 2e- is doable by permanganate there is nothing to stop it happening .. in this case it is the faster reaction. So that is what happens.
In fact since it is more thermodynamically favourable: half cell emf of above reaction ~0.2V giving an overall potential of 1.8V (cf the overall potential for the H2O2 + MnO2 > MnO4- + H2O reaction is about 0.2V) it can be expected (but not certainly) to be the faster reaction anyway. (Sf5xeplus (talk) 20:09, 30 July 2010 (UTC)[reply]
Additionally there isn't actually anything the permanganate could be oxidised to. Sf5xeplus (talk) 20:20, 30 July 2010 (UTC)[reply]
It might not be stronger than KMnO4 in acidic solution though. Reduction for MnO4-- is +0.59 in basic solution but +1.70+1.51 (the article states reduction to Mn++) in acidic solution. H2O2 is +1.78 in acidic solution. As reducing agent its oxidation potential is -0.70. --Chemicalinterest (talk) 00:49, 31 July 2010 (UTC)[reply]
The curve of increase in oxidizing power as a result of pH may not be the same for both chemicals. So in an extremely acidic solution (all H+), H2O2 might be stronger, while in a more weakly acidic solution MnO4 might be stronger. Just an idea. --Chemicalinterest (talk) 00:51, 31 July 2010 (UTC)[reply]
Also bear in mind that one of the possible reduction products for permanganate, Manganese dioxide, is a catalyst for the (non-redox) decomposition of hydrogen peroxide.77.86.5.67 (talk) 12:05, 31 July 2010 (UTC)[reply]
77, it is a redox reaction; specifically, it is disproportionation of 2 O22- into O20 and 2 O2-. --Chemicalinterest (talk) 12:22, 31 July 2010 (UTC)[reply]

I have grown a number of seed-containing pods. What should I do with them? Kittybrewster 18:39, 30 July 2010 (UTC)[reply]

Kill them. Wisteria is a highly invasive and troublesome plant. If you decide to put it in your yard, you will have to prune it daily. Even then, it will likely spread underground and take over not only your yard, but all yards around you. A lady three houses down planted Wisteria in her yard two years ago. Now, the entire neighborhood has an impossible Wisteria infestation that is not only problematic, but also causing property damage as Wisteria is a strong plant that can take down fences, trees, and crack the foundations under houses. -- kainaw 19:40, 30 July 2010 (UTC)[reply]
It is hardly kudzu. Kittybrewster 20:25, 30 July 2010 (UTC)[reply]
Out of curiosity, has anybody gotten the neighbor to pay for the damage? Comet Tuttle (talk) 20:07, 30 July 2010 (UTC)[reply]
Rylands v Fletcher? Kittybrewster 20:25, 30 July 2010 (UTC)[reply]
Wisteria in the UK is a beautiful slowish growing climber that rarely causes damage if planted in the right place. I have seen several that have been growing close to houses for decades with no obvious dammage. It is not invasive in the sense that it propagates in an uncontrollable way. I'd plant those seeds in the autumn in a gritty loam and see what happens. I have doubts that much will but just imagine - your very own wisterias. Good luck. Richard Avery (talk) 22:02, 30 July 2010 (UTC)[reply]
From what I recall reading, even in the UK they cause damage to brickwork, and they do not support their own weight so have to be supported. On the other hand there is an attractive wisteria growing up the side of a building in one of the Inns Of Court in central London. 92.29.119.4 (talk) 22:31, 30 July 2010 (UTC)[reply]
We (UK) have some wisteria growing that supports itself, but we do have to prune it back every year to stop it getting too heavy for itself, and too intrusive. 86.164.66.83 (talk) 23:35, 30 July 2010 (UTC)[reply]
Here (in the southeast US), Wisteria grows so fast that you can literally watch it grow. Every weekend, I have to trim back foot long vines such that no Wisteria can be seen. It all grows back in just a week - over an inch a day. -- kainaw 17:19, 31 July 2010 (UTC)[reply]

veneer core plywood

what is veneer core plywood —Preceding unsigned comment added by Tomjohnson357 (talkcontribs) 19:19, 30 July 2010 (UTC)[reply]

You can find an explanation of the different types of plywood here. Looie496 (talk) 19:24, 30 July 2010 (UTC)[reply]

Strange patterns in the water

There is a lake on the University of Nottingham's Jubilee Campus (it's the north one if you want to look on Google Earth) and it has several what I assume are oxygenating bubble jets which cause the water to bubble. When it rains, there is always a disc visible around these jets where the water is a different colour (see picture). Can anyone explain why this would be the case? I'd have thought it might be the jets adding some sort of chemical, but the effect is too clear-cut than diffusion would suggest. I have uploaded a sunny picture for comparison. -mattbuck (Talk) 19:21, 30 July 2010 (UTC)[reply]

Heh, I live in Nottingham and spend a bit of time at that campus. How strange. Can't answer your question though, sorry! Regards, --—Cyclonenim | Chat  20:07, 30 July 2010 (UTC)[reply]
Possibly a convection cell - oxygenated water forced up by the pumps spreads out a specified distance and then rolls under. that would imply a density difference of some sort, however (either the oxygenated water is colder than the surrounding water or the oxygenation process changes water density). You can see the pattern in the sunny picture as well, if you look closely - it's just more muted. --Ludwigs2 20:14, 30 July 2010 (UTC)[reply]
Could be a visible effect from the submerged heat-exchangers in the lake. Nanonic (talk) 20:36, 30 July 2010 (UTC)[reply]
Is there something under the disk meaning the water is shallower there? --Tango (talk) 20:47, 30 July 2010 (UTC)[reply]
I honestly have no idea. All I know is there are a lot of big koi living in there, and that it leaks because they need to keep topping it up every week. -mattbuck (Talk) 22:10, 30 July 2010 (UTC)[reply]
If it's not artificial, it could be some form of natural spring. ~AH1(TCU) 23:55, 1 August 2010 (UTC)[reply]


The upwellings caused by Langmuir circulation are clearly visible
Upwellings can cause a texture on the water surface that give it another colour. This is seen on a larger scale in Langmuir circulation. EverGreg (talk) 07:39, 2 August 2010 (UTC)[reply]

Irrational fear

The other day me and a couple of friends were sitting and talking on a park bench pretty late, and the conversation came up about fear and how we all seem pretty susceptible to fear even when we know it's very, very illogical. For example, if you sit and look out into the distance in the dark, you can imagine things there that really quite obviously aren't, and the presence of fear can remain. Is there a term to describe irrational fear, other than just that? Also, is there a term describing when you see something that isn't there (not a mirage, I think you get the point)?

Cheers! Regards, --—Cyclonenim | Chat  20:05, 30 July 2010 (UTC)[reply]

Phobia, and hallucination? Looie496 (talk) 20:14, 30 July 2010 (UTC)[reply]
Not really what I was looking for. 'Phobia' doesn't quite fit as it's not an inherent fear that's there all the time, it's only there when you're really focusing on what you're actually thinking about! Hallucinations don't normally come across as hallucinations, do they? I mean, people having hallucinations usually think what they're seeing is real. I don't. Regards, --—Cyclonenim | Chat  20:31, 30 July 2010 (UTC)[reply]
It sounds a bit like paranoia. --Tango (talk) 20:48, 30 July 2010 (UTC)[reply]
For phobia I take your point. For hallucination I don't think you have the word down correctly -- it doesn't imply believing that the thing is real. Looie496 (talk) 20:54, 30 July 2010 (UTC)[reply]
Fear of the dark seems to cover this. It is a normal phenomenon, as distinguished from the pathological fear of darkness, nyctophobia. Not great articles, but maybe that's a start? --- Medical geneticist (talk) 20:58, 30 July 2010 (UTC)[reply]
Thanks, that does pretty much cover it. What about the phenomenon of actually seeing things that aren't there? I take Looie's point above with respect but it still doesn't occur to me as an hallucination. This only really happens in the dark and it's basically if you stare at some space in the distance your mind wrongly decides that, say, a post by a tree is actually a person. Maybe I'm just overthinking this and it is a form of hallucination, I don't know. Regards, --—Cyclonenim | Chat  22:43, 30 July 2010 (UTC)[reply]
Imagination is not just for kids. It's easy to imagine things, and if you are good at it you can even (for a while) believe it. Imagine is often used as a negative word, but it doesn't have to be. Ariel. (talk) 23:25, 30 July 2010 (UTC)[reply]
Fear, anger, sadness and love are always irrational emotions but may be based on good cause. Hallucinations may be chemical and don't have to induce fear. Eg seeing a bunch of horses galloping past your right field of vision in your dining room. Kittybrewster 02:11, 31 July 2010 (UTC)[reply]
Also see fear of ghosts and boogeyman. ~AH1(TCU) 23:54, 1 August 2010 (UTC)[reply]

Name this climbing plant?

What is this please? http://img201.imagevenue.com/img.php?image=27784_climbingplant_122_47lo.jpg Thanks 92.29.119.4 (talk) 22:28, 30 July 2010 (UTC)[reply]

My guess: Virginia creeper Parthenocissus quinquefolia --Digrpat (talk) 01:48, 31 July 2010 (UTC)[reply]

Looks like it. Thanks. 92.15.12.218 (talk) 13:41, 31 July 2010 (UTC)[reply]

Silent cars?

Will it ever be possible to have silent cars, and hence no traffic noise? Or is the current noise-level of modern cars as quiet as we can get? I assume most of the noise comes from the tyres and road surface rather than engine noise. Thanks 92.29.119.4 (talk) 22:55, 30 July 2010 (UTC)[reply]

While not directly answering your question, I'll direct you to Electric vehicle warning sounds, and ask you to keep in mind that the majority of modern cars are still operating with standard internal combustion engines. --LarryMac | Talk 23:00, 30 July 2010 (UTC)[reply]
Personal experience, but it's relevant. When the first Prius cars came out I was at an art festival in the Blue Ridge mountains and whilst walking through a parking lot had the very disturbing experience of a Prius appearing out-of-nowhere into my peripheral vision and then passing me as I walked along. Now - the Prius obviously had been driving through the parking lot at a safe speed and simply passed me as any other car would, but because of its engine design, the only thing I heard was the light sound of tires on gravel. Had it been blacktop, I'd not have heard a thing. That's pretty silent!
Assuming in some alternate reality everyone switches to electric or quiet hybrid cars, though, you're still going to have horns and the occasional need to use them. So if you include horns and aftermarket stereos in your "traffic noise" category, then the answer is clearly "no" regardless of engine noise levels... 61.189.63.171 (talk) 23:35, 30 July 2010 (UTC)[reply]
Maybe silent cars of the future will need radar-operated horns to honk and automatic brakes to operate when some doofus or blind person steps out in the street without looking both ways. Edison (talk) 04:12, 31 July 2010 (UTC)[reply]
Actually, at high speed on a freeway the dominant sounds are tire and wind noise. Engine noise is there, but it's nearly drowned out by the tire noise. An all-electric car with normal tires is nearly as loud as any other car on the freeway. If cars start using the tweel instead of inflatable tires, they'll be even louder (my company tested this for a military application; the rubber spokes flexing and snapping taughttaut as the wheel turned generated a lot of noise). ~Amatulić (talk) 05:47, 31 July 2010 (UTC)[reply]
You mean taut. 92.15.12.218 (talk) 13:23, 31 July 2010 (UTC)[reply]
Yes. Thanks. ~Amatulić (talk) 06:02, 1 August 2010 (UTC)[reply]
Perhaps for the safety of pedestrians, electric cars should be fitted with external loudspeakers playing recorded or simulated internal combustion engine sounds in synchronisation with their speed and acceleration :-) . 87.81.230.195 (talk) 12:02, 31 July 2010 (UTC)[reply]
Or else Good Humor truck jingles, just to confuse all the kids. DMacks (talk) 15:16, 31 July 2010 (UTC)[reply]
Need I point out a lot of this is already somewhat discussed in the link LarryMac, the first respondent, provided? Incidentally I don't think people tend to cross on highways, looking first or not Nil Einne (talk) 19:34, 31 July 2010 (UTC)[reply]
ZENN vehicles (zero emission, no noise) are said to produce no "noise", but it's difficult for any car to be completely silent unless it integrates some kind of external sound absorption mechanism. ~AH1(TCU) 23:50, 1 August 2010 (UTC)[reply]

July 31

Have cognitive scientists explained why it's tedious to not use pronouns?

Please see thread at

Wikipedia:Reference_desk/Language#Have_cognitive_scientists_explained_why_it.27s_tedious_to_not_use_pronouns.3F

thanks

Andrew Gradman talk/WP:Hornbook 01:44, 31 July 2010 (UTC)[reply]

To quote your example from the Language desk: "After Jane Smith ate a sandwich, Jane Smith took a walk, where Jane Smith saw Jane Smith's favorite tree next to Jane Smith's favorite house ..." - I think it's pretty obvious. "Jane Smith" is much longer and slower to say, write or type than "she" and "her". SteveBaker (talk) 05:04, 31 July 2010 (UTC)[reply]
I think it's more than that. Try "Jo" for instance. Still more tedious. I think it's because we use pronouns to keep track of people "already" in working memory and we get puzzled at having to introduce 'new instances' of the same person. John Riemann Soong (talk) 06:00, 31 July 2010 (UTC)[reply]
Or even just "M" or "Q" or something other than "I", where the letter is meant to be a name. "After Q made James Bond a sandwich, Q took a talk, where Q saw Q's favorite gadget..." --Mr.98 (talk) 13:06, 31 July 2010 (UTC)[reply]
Yes - but having established that in most cases it is quicker to use the name once at the start - then use to 'her' and 'he' after that, we have that as a 'normal' way of speaking - so we use that same pattern even though it's no faster in the case of 'Jo' and 'Q'. SteveBaker (talk) 15:33, 31 July 2010 (UTC)[reply]
Languages such as Italian ("null subject languages") have an even faster way — once it's clear who's being referred to, you just omit the subject altogether, without replacing it with a pronoun, except for emphasis. Continually repeating the pronoun in Italian sounds as stilted as continually repeating the name would, in English.
Of course it's a bit easier to do that in Italian because it's a more inflected language than English; the verb ending (usually) tells you first-, second-, or third-person, and singular or plural. --Trovatore (talk) 21:41, 31 July 2010 (UTC)[reply]


1) After establishing who a personal pronoun represents, one has available a standard set of pronoun versions tabulated below. They are usually quicker to read than the corresponding name version. The name versions for ownership and reflexive seem particularly tedious compared to the pronoun versions. Only the pronoun versions distinguish between nominative and object immediately without the reader needing to look for context.

                  NAME VERSION      PRONOUN VERSION
Nominative        Steve             He
Object            Steve             Him
Ownership         Steve's           His
Reflexive         Steve himself     Himself

2) The first sentence below using names requires some effort at anaphore resolution which is obviated by the second version.

Bob shot Robert K. StJohn-Hollyoaks.
Bob shot himself.

Cuddlyable3 (talk) 23:00, 31 July 2010 (UTC)[reply]

3) Personal pronouns can resolve gender ambiguity.
It was Pat's mother that chose Pat's name.
It was his/her mother that chose Pat's name.

Cuddlyable3 (talk) 14:35, 1 August 2010 (UTC)[reply]
That's not really an advantage though. Most languages have sexual dimorphism among names for a reason; sexually ambiguous names are exceptions to a trend. Besides, it's usually a problem when you don't know the gender of said person. Pronouns aren't used for these rather trivial "advantages". Some languages use kinship terminology rather than pronouns -- see Vietnamese pronouns. John Riemann Soong (talk) 19:17, 1 August 2010 (UTC)[reply]

Speed of lightning

What is the range of lightning speed observed? I'm a bit confused by some documentaries and websites that explain lightning discharge can travel at speeds near from the speed of light. --Email4mobile (talk) 04:04, 31 July 2010 (UTC)[reply]

Lightning propagating at near light speed is hard to believe. I can see the progress of a lightning stroke across the sky, and it looks orders of magnitude slower than light should be able to travel through the same portion of sky. There should be some quantitative answers from high speed photography done in the course of lightning research. Edison (talk) 04:09, 31 July 2010 (UTC)[reply]
Check out this [15] incredible video. It shows lightning propagating - filmed on a 9,000 frames per second camera - so the lightning is slowed down maybe 300 times. The initial strokes take several seconds to grow in the video - which means 10's of milliseconds in reality. It's hard to judge how high the clouds are in the video - but it's likely to be in the 500 meter-ish range. So 500 meters in maybe 10 milliseconds is 50km/sec or 100 thousand miles per hour. Light travels at 670 million miles per hour - so the lightning bolts seem to be propagating at a very small fraction of light speed. This is a pretty rough calculation though - there is room for maybe a couple of orders of magnitude of error - so maybe at most 1% of light speed? SteveBaker (talk) 05:01, 31 July 2010 (UTC)[reply]
Lightning leader propagates at 105 - 106 m/s, or sometimes even slower, see "Lightning: Physics and Effects" by Rakov and Uman. The subsequent "strokes" of lightning (the ones that propagate in the established channel), however, are much faster, sometimes in excess of 108 m/s. Speed of light is 3x108 m/s. --Dr Dima (talk) 05:28, 31 July 2010 (UTC)[reply]
You can see that latter effect in the video - once the channel is established, the subsequent strokes along that same path happen instantly - even at 9000 frames per second. I was talking about establishing the initial path. SteveBaker (talk) 15:28, 31 July 2010 (UTC)[reply]
As usual, the problem stems from a problem of definition. What do you define as the "speed" of lightning? Lots of complicated things are happening in the course of a lightning bolt. There are electromagnetic waves, that propagate at the speed of light, carrying information about bulk changes in ion distributions. These waves help "prepare" the air mass for the next phase - dielectric breakdown. The dielectric breakdown of air propagates at a difficult-to-determine speed. At every point along an (as-yet undecided) path, the static electric field must be greater than the breakdown-strength of air - something like a megavolt per meter. But that exact number depends on air density and humidity (among other parameters). These values fluctuate on the microscopic scale - and are affected by the breakdown of neighboring regions - so it's a mess of electromagnetic wave propagation and fluid-dynamics. The next phase is that after breakdown, current begins to flow. Just like any electric current, we have both an electromagnetic wave (propagating at the speed of light in ionized air), and we have microscopic drift motion of electrons (and, unlike a copper wire, bulk drift velocity of ions). Every constituent ion of air has a different ion drift velocity - because it depends on mass. So, O- will drift at a different speed than O2- or N2- and so forth. The current will flow at the speed of the electromagnetic wave that conveys information along this ionized path (which is still forming as the instantaneous static electric field overcomes the breakdown strength in new areas at the edges of the strike path). Finally, the slowest part of all, is the incandescence - what we actually see. The current releases so much energy that it heats the air. The rate of heating depends on the current strength (which can very from a few hundred amps in a small lightning strike, up to nearly a mega-amp in a giant bolt). The rate of heating depends on air parameters, (density, humidity, and so forth). So when you finally see the "light" of the lightning, the bolt may have already existed for anywhere from a few nanoseconds to a few milliseconds before the air got hot enough to glow. Lastly, the thermal and electromagnetic change that has been building up over these microsecond scales will seem instantaneous to the neutral air surrounding the lightning strike path. This air is not ionized; it has not been affected significantly by the electromagnetic waves or the ionization that has communicated the change of state through the bolt path. As far as this air is concerned, the change from "normal air" in its neighboring region to "ionized plasma with enormous electric current and heat" is "instantaneous". So, there is a rapid shock front - a thunderclap - as the hot air acoustically transfers heat, pressure, and momentum away from the bolt at the sound speed in air (perturbed by pressure and humidity, not to mention rain-drops, which are non-negligible - they are a dense field of acoustic scatterers with dramatically different acoustic speeds, reflecting, dispersing, and attenuating any acoustic energy). The amazing thing about lightning is that it occurs across many orders of magnitude - microscopic voltages between individual ions accumulate (each individual electron charge) and result in a bulk ion concentration of thousands of coulombs of static electricity. Tiny perturbations in air pressure (chaotic variations of nano-meter size) between air particles dictate which air parcel will be most susceptible to instantaneous dielectric breakdown and ionization. Electromagnetic waves travel at near the speed of light in vacuum; and then when the ion channel forms, they travel through that channel much slower (because of the plasma properties); but they still travel through the neutral air at near-vacuum speeds, so there is a complicated interplay there. The rate of heating and incandescence are derived from this massively complex interaction, and the illuminated bolt you see tracing its way across the sky eventually expands at speeds on the order of kilometers-per-milliseconds. Finally, just for the interested reader - transient luminous events associated with lightning, including jets, sprites, and elves, are not well explained. Our physical understanding of the appropriate propagation speeds in sparse plasmas tell us that the jets appear above a lightning strike too soon at too-high an altitude - indicating an unknown form of information- or energy-transfer. The most likely explanation is a nonlinear interaction between weak electromagnetic waves and sparse, ionized plasmas, somehow amplifying the energy and generating a "second" illumination about a hundred kilometers above the lightning strike. These are rarely observed from the ground (though it has been done); but they are regularly observed by satellite. Red Sprites and Blue Jets, from Univ. of Alaska's Geophysical Institute, and Lightning-induced Electron Precipitation from Stanford's Low Frequency radio group both have good descriptions, pictures, and links to active research on lightning physics, including efforts to experimentally verify the relevant timescales for each physical process in a lightning strike. And, the Lightning Research Lab at Univ. of Florida experimentally triggers lightning strikes using rockets - again, attempting to characterize the physical parameters of the strikes, including the ion distributions and speed of bulk charge transfer. Nimur (talk) 17:18, 31 July 2010 (UTC)[reply]
The article says that lightning can travel at speeds near 60,000 m/s. While the flow of current can travel close to the speed of light in a lightning strike, the electrons would move much slower than that and the lightning itself slower still. ~AH1(TCU) 23:47, 1 August 2010 (UTC)[reply]

Rechargeable batteries not recharging

Because I take lots of photos, I maintain eight NMH rechargeable AA batteries; at any given time, four are in the charger (although not always plugged in), two are in the camera, and the other two will be backups in the camera bag. Last time that I switched my batteries, I noticed that the ones in the camera and the ones in the camera bag seemed to hold only a small charge; apparently they're older. Today, I charged them, and when they were done, I put them into the camera, taking care to separate them and the other four. It turns out that two of the batteries, when placed in the camera, have only enough energy to produce a "Change batteries" message, even though they've charged all day. Should I assume that they're just too old and past their ability to hold a charge, and thus trash them? Or is there some practical way for me, the non-scientist, to do something useful with them? Finally — it's been so long since I got rid of rechargeable batteries that I don't remember what to do with them; is the trash safe, or is there a better option? I live in a rural area without any realistically-nearby HAZMAT centers. Nyttend (talk) 04:11, 31 July 2010 (UTC)[reply]

In my experience, there is a substantial chance that the charger goes bad and not the batteries. You can ask your friends to lend you their charger for a day, and see if the batteries charge better with a different charger. --Dr Dima (talk) 05:42, 31 July 2010 (UTC)[reply]
I own Energizer NiMH batteries that are old and although fully charged at the beginning, die within about 4 days because of current drainage. --Chemicalinterest (talk) 11:04, 31 July 2010 (UTC)[reply]
Dr Dima, this isn't an option because the other two batteries that I charged don't even cause the low battery indicator to appear, while the four batteries that I had in the camera also work for quite a long time. Nyttend (talk) 11:18, 31 July 2010 (UTC)[reply]
If some are older than others, then the older ones may behave as I have stated above. --Chemicalinterest (talk) 11:43, 31 July 2010 (UTC)[reply]
Rechargeable batteries should be recycled, but NiMH do not contain hazardous substances like other batteries do. --Chemicalinterest (talk) 11:44, 31 July 2010 (UTC)[reply]
I find that my camera seems to require fairly new NiMH batteries and that cells that have been recharged lots of times give a battery low indication almost immediately, but that when they no longer hold sufficient charge for the camera, they can still be used for less-demanding equipment for many more charge cycles. Slower charging (14 hours at 1/10 rating) and slower discharging in low-current applications seems to prolong their life, but eventually they have to go to recycling. Dbfirs 12:40, 31 July 2010 (UTC)[reply]
Most probably the batteries are getting older so don't hold a full charge, so won't work in a high drain device like a camera, but would be fine in an alarm clock/remote control or other low drain device. Here in the UK most large supermarkets accept all types of AA and AAA batteries for recycling. Rjwilmsi 16:37, 31 July 2010 (UTC)[reply]
Hello most active Wikipedian! I don't normally see you at the desks. --Chemicalinterest (talk) 18:50, 31 July 2010 (UTC)[reply]
Hmm, I've never recycled batteries, because I didn't know that they could be. I'll see what can be done about using them in other devices; no reason to get rid of batteries that should work with those things. Thanks, everyone, for the input! Nyttend (talk) 12:59, 1 August 2010 (UTC)[reply]
Of course, the link to battery recycling would also be useful. ~AH1(TCU) 23:41, 1 August 2010 (UTC)[reply]

Blu-Ray video files?

I have Sony PMB software, and average graphics card which my Cyberlink video software tells me needs upgrading on a dual core computer running Windows XP. The Cyberlink software is needed to write to my new Blu-Ray drive, because unless there is some sort of upgrade for XP(?) it only "knows" about DVD/CD... —Preceding unsigned comment added by 80.1.80.10 (talk) 06:21, 31 July 2010 (UTC)[reply]

Question moved to to the computing/IT reference desk by CS Miller (talk) 09:55, 31 July 2010 (UTC)[reply]

difference between taxis and tropism

What is the popular explanation of difference between taxis and tropism in biology? —Preceding unsigned comment added by 113.199.157.152 (talk) 11:34, 31 July 2010 (UTC)[reply]

Try the articles on taxis and tropism where this distinction is amply covered. If you have any specific questions, feel free to ask and we'll try to help. --- Medical geneticist (talk) 14:06, 31 July 2010 (UTC)[reply]

Working prototypes

How are working prototypes of machinery or consumer goods usually built, without going to the expense of a robotic production line? Supposing I had a cool idea, and I was sufficiently trained/experienced in the relevant science to know that it could work, but I wasn't a master craftsman in all the dozens of disciplines required to produce a working model by hand. If I was working on it by myself, how could I get one built within a realistic budget? I have read the rapid prototyping article, and I think 3D photocopying is ace :-)) but isn't the plastic used by an additive modelling machine too weak for moving parts? 213.122.216.120 (talk) 12:05, 31 July 2010 (UTC)[reply]

There are companies that specialise in making prototypes. Or you could try Meccano or woodwork or making it at a different scale that would be easier to make. 92.15.12.218 (talk) 13:27, 31 July 2010 (UTC)[reply]
It's normally chiselled from universal matter, obviously you can only make substances for which you know the molecular composition this way, but you don't have to start with machined or mass-produced parts 84.153.192.227 (talk) 13:55, 31 July 2010 (UTC)[reply]
See also Reprap#Materials , Fab lab 92.15.12.218 (talk) 14:25, 31 July 2010 (UTC)[reply]
Very often, companies will use the 3D printer (aka rapid-prototyper) to make plastic parts to check that everything fits together right and that parts can move as desired without interfering with each other. They may also be painted and assembled to show to marketing departments, etc. Then they may use the plastic version to make molds from which metal parts may be made. There are also 3D printers that can use Direct metal laser sintering to produce metal parts directly. Alternatively, CNC milling machines can automatically make some types of part by 'subtractive' approaches (they carve the part out of a solid block) rather than using additive techniques. Failing that, they may just use an old-school engineering shop to hand machine the parts in small quantities for prototypes using drills and lathes and other traditional tools.
For doing something yourself, commercial 3D printers and CNC machine tools are still hideously expensive. If you are moderately good with your hands, you could build your own RepRap or a three-axis router (I built one of those!). But if you aren't skilled with your hands - then you may have to find a small engineering company who'll make one-off parts in appropriate materials. It's not going to be cheap though. Depending on what you're trying to make, you might be able to get creative with stuff like Lego Technics, Erector-set/Meccano, whatever - or build it from available parts from places like McMaster-Carr who have everything from nuts and bolts to gearwheels, motors, you name it. A lot depends on what you're trying to make. If you've invented a new kind of super-tanker or airplane - then you've got a big problem - but if you've invented a new kind of mousetrap - then it shouldn't be too hard to figure out how to prototype it.
Electronics and computer parts are a whole different set of problems - and again, there are companies that will do that for you - and (if you have the right skills) there are lots of ways to build this kind of thing yourself. If you need a computer-controller, then consider the Arduino range of systems. You can get a computer board for under $30 and program if from your PC...but again, what skills do you have along those lines?
If you don't have the skills to make things yourself and you fancy yourself as an inventor - then you should definitely consider learning those skills. There are plenty of courses at community colleges and such like that will teach you many of these skills and give you access to their workshops and machinery to practice on. I took a welding class like that and was able to use their equipment to do all the welding on a classic car I was restoring. The $200 I spent on the course (plus welding gloves & mask) were well worth it because in the end, I didn't have to go out and spend $2000 on a welding machine - or pay someone else $800 to weld up my car - and now I have a skill that I didn't have before. A lot of basic metal and wood-working skills are easy to teach yourself - just buy tools and potter around in the garage making stuff - and you'll get better at it. Useful shop tools like used drill presses can be bought very cheaply on Craig's list as engineering companies fail in our declining economy.
We could give you a much more concise answer if we had more specific information about what you're trying to build and what skills you have.
SteveBaker (talk) 15:13, 31 July 2010 (UTC)[reply]
I kept the question vague because I don't have an invention of my own (yet!). I'm just honestly curious about how prototyping works. Never understood how ideas made the leap from the Eureka moment in the bath, to having a working example that might convince investors it's worth hiring specialist craftspeople or setting up a production line. Guess I shouldn't be too surprised that it's mostly self-improvement plus trial-and-error, if you haven't got money to burn. Thanks a lot for the replies. 213.122.216.120 (talk) 17:43, 31 July 2010 (UTC)[reply]
Where I work we've gone through this a few times, and let me tell you when we started we had the same questions as you because we're all software developers and none of us had ever developed hardware.
The first steps are obviously brainstorming and coming up with ideas and sketches. First broad ideas are sketched out on the white board, then the best ones are sketched out in more detail. Our artist likes to use a 3d modeling program to sketch out how things fit together, but he could have used pen and paper without too much more difficulty. If the device has electronics (and if we're designing them ourselves) a draft circuit diagram is drawn at this point. Probably using specialized software, but again, pen and paper wouldn't be too much of a hassle either.
Next, usually a very rough prototype is made to make sure we're not crazy. This was done with scrap wood and everyday tools of the sort that most people already have hanging around somewhere. None of us are what you would call 'carpenters' or 'craftsmen', so this is always good for a laugh! But it also provides the important function of making sure that the basic idea of what we're building is right. (If the device has electronics, a rough draft of the electronics would also be made on a breadboard or something. Or maybe just a rat's nest of wires!)
Finally, if we still thought we were on the right track, we find a local plastics shop and give them our sketches and 3d models. Ideally, at this point you would want the design finalized by someone who knows what the heck they're doing, but we've found this isn't strictly necessary if you're willing to put up with a lot of trial and error. The plastics shops makes us some prototype pieces of the plastics parts. (Using 3d printing if we only want one or two, some sort of molding process otherwise.) We also get circuit boards made. (This is very cheap if you don't mind soldering the bits and pieces on yourself.) Then we spend another fun day putting the pieces together ourselves.
The result is a professional-looking prototype that does what it's supposed to do and if you don't look too closely you can't tell it's held together with superglue!
I'm sure companies that do this stuff on a regular basis have a much more streamlined and efficient process, but I think our experience more closely approximates the 'garage inventor' scenario that you're imagining. APL (talk) 19:55, 31 July 2010 (UTC)[reply]
That "streamlined and efficient process" is to find a professor with a vaguely similar research idea and make his graduate students build a prototype for you. Graduate students "know how to build things!" But seriously, there's an important distinction between a prototype that does something useful, and a prototype that is packaged professionally. If you need a quick turnaround with a slick interface, you need to find a good machinist, a good electronics assembly technician, and so forth. They will make a device that looks clean, works smoothly, to whatever spec you ask for. If you care less about presentation and packaging, and are concerned with innovating, then you spend less effort on things like appearance, usability, and reliability - "proof of concept" - to demonstrate that it's worth throwing more resources at a project in the future. Then you find somebody with resources. Nimur (talk) 20:16, 31 July 2010 (UTC)[reply]

Plant health question (Podocarpus macrophyllus)

I have a Podocarpus macrophyllus that I bought several weeks ago. It's shaped like a lollipop, with a roughly spherical clump of branches on top of a straight stem. At a glance, it looks green. But upon closer examination the "core" of that spherical clump is largely composed of brown or wilting needles. The outer "layer" all around is dark green or lighter green (new growth). I've never owned anything similar before, and would like to know if this inner browning is a natural part of the growth of the plant, or whether I should be concerned. Any tips regarding pruning would also be welcome. Thank you! 61.189.63.171 (talk) 13:24, 31 July 2010 (UTC)[reply]

Is the soil very dry or very wet? 92.15.12.218 (talk) 13:37, 31 July 2010 (UTC)[reply]
If the outer layer is, as you say, green with new growth, then the plant is growing. I can't imagine a podocarpus being this shape naturally so it sounds as though it has been continuously trimmed to make it a lollipop shape. I'm thinking maybe the trimming has caused a dense outer covering of shoots that is preventing light and air getting to the centre which has started to die back. There would be cut stems over the surface of the plant, under the new growth, to indicate previous trimming if my idea is correct. If you have new growth then you should not be too concerned, but it is odd. Richard Avery (talk) 07:44, 1 August 2010 (UTC)[reply]

Farm Pond Spray Fountain

Would like step by step information on how to construct a spray fountain in a large pond to assist with aeration. What specific parts are needed and how to assemble. ≈≈≈≈ —Preceding unsigned comment added by 198.203.175.175 (talk) 18:48, 31 July 2010 (UTC)[reply]

The article Water aeration has some links, or search for "pond fountain spray how to" or similar. Basicially you put a fountain in the pond with a spray nozzle. Note that over aeration can be harmful to plants.
There are literally hundreds of pond fountain options - they will have instructions for installation.Sf5xeplus (talk) 12:41, 1 August 2010 (UTC)[reply]

Who coined the term 'fine-tuned Universe'?

Or when? I was horsing around in the web, but couldn't figure out. Twilightchill t 20:52, 31 July 2010 (UTC)[reply]

The earliest source I found is 1990 Richard Swinburne Argument from the fine-tuning of the universe in Physical cosmology and philosophy, J. Leslie, Editor. Collier Macmillan: New York. pp. 154-73. Wikipedia has an article on Fine-tuned Universe. Cuddlyable3 (talk) 22:17, 31 July 2010 (UTC)[reply]
I think it may have been George Coyne, a Jesuit astronomer who was using the term in talks and publications as early as 1982. Coined by Coyne, heh. Looie496 (talk) 00:58, 1 August 2010 (UTC)[reply]

The modern idea of Fine-tuned Universe can be traced back to Leibniz's optimism that "we live in the best of all possible worlds", famously lampooned by Voltaire in his novel Candide in 1759..Cuddlyable3 (talk) 13:08, 1 August 2010 (UTC)[reply]

Corner reflector

What is a three surface corner reflector? --The High Fin Sperm Whale 21:20, 31 July 2010 (UTC)[reply]

Imagine a hollow cube with three faces removed. What you have is three squares joined along three edges:
        /|\
       / | \
      /  |  \
     |   |   |
     |  / \  |
     | /   \ |
     |/     \|
      \     /
       \   /
        \ /
The deal is that whichever surface an incoming beam hits, it will bounce off again, hit a second surface, then a third - then head back in exactly the exact reverse direction it arrived in. SteveBaker (talk) 21:34, 31 July 2010 (UTC)[reply]
Also see Corner reflector. --Cookatoo.ergo.ZooM (talk) 21:38, 31 July 2010 (UTC)[reply]
Wait a minute, if a beam of light comes at an angle close to 0 degrees relative to the normal of one of those surfaces, could the beam miss any other surfaces completely and reflect in a non-reverse direction? ~AH1(TCU) 23:36, 1 August 2010 (UTC)[reply]
Well, in an idealized corner reflector, the three surfaces are infinitely large and eventually, either the ray hits another of the surfaces - or it's literally at zero and bounces right back into the source. Of course, real corner reflectors aren't infinitely large - so they don't reflect back 100% of the light. But for most practical purposes, they are highly effective. SteveBaker (talk) 01:51, 2 August 2010 (UTC)[reply]

August 1

Question 9, part (d)(i)

http://www.tqa.tas.gov.au/4DCGI/_WWW_doc/008614/RND01/PHY5C_paper.PDF The Right hand grip rule tells me that current is flowing clockwise, which means the electrons are flowing anticlockwise, but apparently that is not correct. Am I missing something?--220.253.172.214 (talk) 01:08, 1 August 2010 (UTC)[reply]

The grip rule refers to the field created by moving charges. However, in this case the field is externally applied (i.e. not related to the electrons), and you are asked to explain how the charges move in response to the field (rather than the other way around). If you apply F = qv × B and the right hand rule you'll arrive at the correct answer that the electrons move clockwise in the applied field. Dragons flight (talk) 01:22, 1 August 2010 (UTC)[reply]
I don't see how F=qvB is usable here since you are not given values for F, v or B. I also fail to see the correct application of the right hand rule, sorry.--220.253.172.214 (talk) 01:35, 1 August 2010 (UTC)[reply]
You don't need the values, just the relationship and signs. In order to move in a circle, F must point towards the center. B points into the page. q is an electron, so it is negative and flips the sign. Given those factors, you use the right hand rule (specifically this) to decide whether clockwise or anti-clockwise v fits the other constraints. Dragons flight (talk) 01:45, 1 August 2010 (UTC)[reply]
To be clear, you need , not just F = qvB. Dragons flight (talk) 01:48, 1 August 2010 (UTC)[reply]
You don't want or need to use the right hand grip rule - you need the Fleming's left hand rule for motors (again remember that current is in the opposite direction to elecron flow). You have the field, and the force (the force must be directed into the centre of the circle to make the electrons turn) - one of the two directions of current gives a force inwards (the other outwards) - from this you can find out whether the current is clockwise or anticlockwise).Sf5xeplus (talk) 12:35, 1 August 2010 (UTC)[reply]

Identical twins and children

Suppose a woman has a threesome with two identical male twins and gets pregnant from this. Is there any way to find out which twin is the father? 68.237.21.90 (talk) 01:27, 1 August 2010 (UTC)[reply]

Theoretically, yes. But you'd have to use a more sensitive technique than traditional paternity testing which only looks at a few loci at which the identical twins are almost certain to be identical. There is a certain mutation rate inherent during gametogenesis, so if you had an appropriate "sample" from each of the potential fathers you could potentially identify positions at which the offspring might differ between them using whole genome sequencing. --- Medical geneticist (talk) 01:38, 1 August 2010 (UTC)[reply]
How does that work? Why would one sperm from a man have the same mutations during gametogenesis as another sperm from that man? And, if that is the case, why would it be different mutations in an identical twin? --Tango (talk) 01:46, 1 August 2010 (UTC)[reply]
The gametes of the two identical twins will contain different de novo mutations. By chance, some of those mutations will be present in the majority of that individual's sperm (while others may be present in only a fraction of the sperm). This is called gonadal mosaicism. If the two sperm samples were sequenced deeply enough using "next-generation" methods, it would be theoretically possible to identify new mutations unique to either of the twins (those would be the positions at which their respective offspring would differ). Then you could just sequence those sites using standard methods in the child. Please note, however, that the OP asked if there was "any way to find out which twin is the father", not whether that method was "practical", "likely to succeed", "admissible in court", or "achievable by the average person". Even though what I've outlined is possible, it would require an extraordinary effort by today's paternity testing standards. --- Medical geneticist (talk) 11:27, 1 August 2010 (UTC)[reply]
I'm sorry, but you aren't making sense. You seem to be using "gamete" and "sperm" as if they are different things. Sperm are the male gametes. When you say "gamete" do you actually mean "gonad"? --Tango (talk) 15:10, 1 August 2010 (UTC)[reply]
Spermatozoa are the mature male gametes. However, de novo mutations occur in the earlier gametic progenitors (during DNA replication), which is why I called them gametes and not sperm. See spermatogenesis and meiosis. The gonad is the sex organ (testis or ovary) that contains and supports the developing gametes, along with having endocrine functions. There is certainly a difference between "gamete" and "gonad". When we talk about "gonadal mosaicism" or "germline mosaicism", the implication is that there can be gametes (progenitors and mature gametes) gametic progenitors with different genetic compositions -- either chromosomal or at the level of individual nucleotides. In the context of the OP's question, the most likely sample to be analyzed would be the mature spermatozoa as opposed to a testis biopsy specimen, which is why I specified that it would be the sperm sample that would be sequenced in this highly improbable scenario. Does it make sense now? --- Medical geneticist (talk) 18:00, 1 August 2010 (UTC)[reply]
No, the same problem exists: if the mutation happens in the production of individual gametes then there will be no correlation between the mutations in different gametes. Without such a correlation, you can't identify the father from new sperm. I think you are trying to say that the mutations happen in the production of gametocytes, not gametes. --Tango (talk) 23:16, 1 August 2010 (UTC)[reply]
Point taken. It was sloppy for me to use the word "gamete" to refer to gametic progenitors. However, gametocyte isn't quite right either. The proportion of sperm carrying a given de novo mutation will depend on how early in the process of gametogenesis the mutation was introduced. In order to generate a significant degree of germline mosaicism, the new mutation would have to occur in the primordial germ cell or spermatogonium. If the mutation occurred as late as the gametocyte, there would not be enough affected mature gametes to allow determination of paternity. --- Medical geneticist (talk) 02:31, 2 August 2010 (UTC)[reply]
(edit conflict)AFAIK, not with current technology. Children of identical twins have the same genetic similarity as half-siblings (rather than as cousins, which is to be expected as the children of non-identical siblings). While it should be noted that one could find genetic difference between identical twins at the full genome level, genetic fingerprinting and paternity testing use a tiny tiny fraction of the total genome, such that one cannot tell the difference. Remember that the human genome project took over 10 years to complete the first full sequencing of a human genome; and that was just a generalized genome for humans in general. An individualized full genome for even one person is a practical impossibility with current technology.ed note: striking. It appears it has been done, but it is still generally impractical to do for purposes such as this Hypothetically, if you COULD obtain a full genome from the child and both twins, you MAY be able to tell the difference. But that isn't possible under modern methods of DNA fingerprinting. See Twin#Genetic_and_epigenetic_similarity for a (IMHO) altogether too-brief discussion of twins and genetics. --Jayron32 01:42, 1 August 2010 (UTC)[reply]
You've blinked and missed the revolution. Sequencing a full individual human genome only costs a few thousand dollars right now, and is expected to drop to less than $1000 in a year or two. (Although I think it's correct that this problem would be extremely difficult even given both full genomes.) Looie496 (talk) 02:01, 1 August 2010 (UTC)[reply]
Note that "a few" still means around $20-15 thousand. But yeah, getting cheaper all the time. --Mr.98 (talk) 02:17, 1 August 2010 (UTC)[reply]
Part of the reason why DNA fingerprinting is legally admissable is that their is solid experimental backing that it works; there are now millions of successful uses which allow us to say that matching two samples via DNA fingerprinting methods is reliable as can practically be. Full genome sequencing indicates that the current number of published full genomes of individuals runs somewhere in the dozens at the outside; while comercialization in the next few years may increase this number, it is not in any way currently a reliable method of identification, which is what is needed to use it to determine which of two identical twins is the father of a child. --Jayron32 02:33, 1 August 2010 (UTC)[reply]
Well, it hasn't been tested in court because of the minimal number of people whose genomes have been sequenced, but I can't imagine it would be seen as unreliable. Basically DNA fingerprinting is sort of like saying two books must be the same because all 20 chapters have the same number of pages. A match based on full genome sequencing is like saying two books must be the same because they match word for word and letter for letter. There just isn't any way it could conceivably give a wrong answer. Looie496 (talk) 06:40, 1 August 2010 (UTC)[reply]
Oh, I am with you on that point. However, I am also married to a forensic scientist, so there is also the other end of it. Being reliable for a scientist isn't the same as being reliable to a lawyer. They are overlapping, but not necessarily identical, sets of data... --Jayron32 03:41, 2 August 2010 (UTC)[reply]

Where is the center of the universe?

An explosion radiates energy equally in all directions. As the energy loses heat, it decays into matter. The Big Bang was such an explosion, and it created a universe, which continues to expand, after nearly 14 billion years. So one would assume that the center of the universe is empty, since all its matter continues to move outwards, and away from other matter. But I am told that our solar system is at the center of the universe, as is our galaxy, the Milky Way. How is this possible? On the other hand, Penzias and Wilson were able to measure the residual microwave radiation of the Big Bang, in frquency and temperature, at the outer edges of the universe. They found that the values were equal in all directions, within minuscule uniformity. How would this be possible, if some of their measurements were taken from one side of the universe to the other, across its empty center? Or from closer to one edge of the universe than to another, more distant edge? Where are we, in relation to the center of the universe? ---- —Preceding unsigned comment added by Geepod2 (talkcontribs) 01:57, 1 August 2010 (UTC)[reply]

The universe has no center; an alternate perspective is everywhere is part of the universes center. The universe does not expand from a single point, like an explosion, it expands from every point, like a rising loaf of bread. One standard analogy is to think of the universe as being like polka dots on a balloon surface. If you draw polka dots on a balloon, and blow it up, all of the dots move away from each other, yet no one of the dots is actually the "center" dot. This is because the universe is not expanding into empty space, it is creating the space itself. The article shape of the universe discusses some of the common theories about the shape of the universe, and many of the best fit shapes are "edgeless". --Jayron32 02:02, 1 August 2010 (UTC)[reply]
When thinking on the scale of the universe, Newtonian physics is no longer a good approximation. HiLo48 (talk) 02:21, 1 August 2010 (UTC)[reply]
You said you've been told the solar system is at the center of the universe. Whoever told you this is completely off the mark since, as described above by Jayron, the universe has no center. You are probabily better off desregarding anything else that person told you about cosmology. Dauto (talk) 05:55, 1 August 2010 (UTC)[reply]
That person maybe meant the Observable universe, since the universe is expanding uniformly and the light has had an equal time to travel in all direction we are of curse in the centre of our Observable universe. Every point is in the centre of the universe observable from that point.Gr8xoz (talk) 10:11, 1 August 2010 (UTC)[reply]

The "center" of the universe isn't in the universe at all! Think of it like an expanding balloon, but a 3-dimensional surface on a 4-dimensional sphere instead of a 2-dimensional surface on a 3-dimensional sphere. --138.110.206.100 (talk) 12:54, 1 August 2010 (UTC)[reply]

Let's pretend that there are only two dimensions of space for a minute. If the universe were infinite in size, then clearly there is no boundary and nowhere can be called "the centre". But if the universe were finite in size (like a piece of paper) then there is a clear boundary and one can define a central point. The same goes in 3D: if the universe is infinite then there is no centre. If it is finite, there it.
There is an interesting alternative however. Imagine we're back in 2D again. Imagine curving the piece of paper into a sphere. The universe (the surface of the paper) is still finite, but now there is no boundary and so no centre on the piece of paper. Or you could form the paper into a doughnut shape or something even crazier. These "curved back on themselves" model universes are said to have "non-standard topologies".
So what do we think applies to our universe? Well, some cosmologists are working on non-standard topology theories, and there are astronomers looking at the sky for signs that we live in such a universe. If light could go "all the way around", we might be able to see the same star at opposite points in the sky. So far we haven't found any evidence for non-standard topology but we certainly haven't ruled it out. But the "standard model" (our best theory of the universe at the moment) says that either that the universe is infinite, or so huge that each point in our observable universe is so far from the boundary that we can't tell which is the central point. And of course if the universe is finite in size (but much much bigger than the observable universe) then the centre of the universe will not be in our observable universe anyway. Most cosmologists don't think that the universe has a wrap-around topology like in the balloon example, so that's not a good explanation as to why there is no centre in my opinion.152.78.128.149 (talk) 13:51, 1 August 2010 (UTC)[reply]
Can you give some examples of cosmologists that seriously think the universe has (or is likely to have) a boundary? My understanding of modern cosmology is that we either live in an infinite universe, or a finite universe without boundary (such as a torus). I've never heard anyone seriously suggest the universe might have a boundary - what would that boundary be like? --Tango (talk) 15:16, 1 August 2010 (UTC)[reply]
True. I don't think I've ever heard of anyone considering a universe with a boundary. Having said this, I work in particle physics (I've just started a PhD, so I'm no expert!) and a model that I've looked at proposes that there is a fourth spatial dimension of (small) finite length that is simply a line segment and not compact like a circle. Technically though, it can be formed by orbifolding a circle. This extra dimension thus has two boundaries. But the orbifolding process naturally leads to boundary conditions on the fields that can live in the dimension so that excitations (i.e. particles) travelling into either one of the boundaries are reflected. So a boundary isn't a completely crazy idea if you add some extra postulates (like those boundary conditions) that effectively tell you what happens to things travelling into a boundary. I definitely haven't heard any cosmologist talking about this stuff though. I'm the poster you replied to, despite the different IP! 86.137.169.18 (talk) 15:49, 1 August 2010 (UTC)[reply]
Inflation makes a large homogeneous region, but not (necessarily?) an infinite one. I remember one of my undergraduate professors sketching on the blackboard a flat region of space surrounded by a crazy wavy region on which he wrote "here be dragons". That's an accurate if sardonic representation of the present state of knowledge in cosmology. The problem is that you can't experimentally distinguish models that only differ outside the observable universe. If the flat region were smaller than the observable universe then we'd see huge inhomogeneities in the CMBR, so that much is ruled out. But whether the flat region is finite and larger than that is anybody's guess. Compact spatial topologies have the same problem. When people propose that space wraps around, they always have it wrap around on a scale smaller than the observable universe, simply because that's the only way the model can make any new predictions. So far, all testable models of this type have been ruled out, but whether space wraps around on a larger, untestable scale is an open question, and may always be. -- BenRG (talk) 22:06, 1 August 2010 (UTC)[reply]
Sure, the inflated region may well be finite, but what would happen at the edge? There would still be spacetime outside it, would anything actually happen when you crossed the edge other than the average density increasing (a lot)? You are right, though, what we are discussing is largely unscientific: by definition, what happens in the unobservable universe is not empirically verifiable. --Tango (talk) 23:24, 1 August 2010 (UTC)[reply]
For the universe to have a shape, that "shape" would take place on a "plane" that would be the "boundary" of said universe. The existing space within the boundary would be the universe, and the non-existing emptiness is what's outside the boundary. Since the universe has an age, it must have had such a boundary while it had a finite size early in its life, and the same should be true when multiverses are created. If such a boundary does exist, it would likely be moving faster than light since matter cannot move faster than but the expansion of space can. The centre of the universe could be considered the origin point of the Big Bang, but that point might no longer meaningfully exist as the universe itself could have moved relative to that point yet there is no outside frame of reference to compare the location of the universe to. ~AH1(TCU) 23:31, 1 August 2010 (UTC)[reply]
I'm afraid that's all nonsense. It just doesn't work like that. When we talk about the universe being a torus (for example) we don't mean a torus embedded in some higher-dimensional space, we're just talking about its inherent shape. It's a little difficult to get your head around, but thinking of the universe as being embedded leads to completely nonsensical conclusions. The big bang does not mean the universe is finite - the universe didn't necessarily start as a point, it could well have been infinite at the time of the big bang and then expanded at all points. --Tango (talk) 23:38, 1 August 2010 (UTC)[reply]
To expand on Tango's idea for a bit; the idea of defining the universe as expanding within something has the problem as this merely redefines what the Universe is; as does the idea of "Multiverses". The Universe is, by definition, everything; if you define the Universe as part of everything, then you aren't talking about the Universe, but something smaller. If the Universe is expanding into something, that something is the real Universe and the thing doing the expanding is a smaller subunit of it. --Jayron32 03:39, 2 August 2010 (UTC)[reply]
Indeed, well put. I like to define "universe" as what you get by taking your current point in spacetime and then adding every point that is causally connected (two points are causally connected if something that happens at one point can have an effect at the other, which basically means light can travel between them) to that point (which results in the observable universe), and then every point that is causally connected to any of those points and repeating ad infinitum. By that definition, any "other universe" in the "multiverse" can either have no effect on us whatsoever, so we might as well assume it doesn't exist, or is part of our universe. You can then forget about multiverses as anything more than a mathematical convenience. --Tango (talk) 12:05, 2 August 2010 (UTC)[reply]

Placebo Questions

I was reading the placebo article on wikipedia and I was wondering a few things...

1. Why does the placebo effect wear off overtime? 2. It also says that a placebo may not always give immediate relief or improvement and the real drug will, why is that? 3. Why does the placebo effect only work in 30% of people? Is it a certain group of people? People that really trust their doctor or people who really think it's going to work? I know the article talks about personality and the placebo effect and says there is no difference in placebo effect based on personality, but does anyone know of any studies that show a difference? —Preceding unsigned comment added by 76.91.30.156 (talk) 08:02, 1 August 2010 (UTC)[reply]

Placebos work on the premise that psyhology can affect physiology. There is a wide physiologic range among people, and there is likely a wide psychological range as well. Even physiologically speaking, a common dose of medication is usually referred to as the ED50, referring to the dose at which 50% of drug takers will manifest the effect. (In order to judge the lethal dose, a similar LD50 is used.) And there's a lot of psychology involved in almost everything one does -- numerous studies have shown that people who engage in social interaction, exercise regularly, etc. can be shown to complain of less pain. So there are so many numerous factors involved in things like post-operative pain/sensitivity/complaints. Perhaps that's a start for you. DRosenbach (Talk | Contribs) 13:05, 1 August 2010 (UTC)[reply]
I corrected your ED50 link. Dose thresholds are meaningful where there is a correlation between dosage and effect. There is no consistent correlation for placebos. Cuddlyable3 (talk) 13:23, 1 August 2010 (UTC)[reply]
There can be an effect between dosage and effect in placebos, if only insofar as the patient believes that two different placebo pills contain different dosages of the illusory drug... --Jayron32 03:34, 2 August 2010 (UTC)[reply]

Computer-controlled cars

I think it might have been in Time Cop; I recall a driverless car that took its occupant to the desired location. Assuming there is no track in the road guiding such a car, could such a system work on the GPS technology we current have, assuming we got vehicular technology up to such a level? Cars would be speeding around and would never crash because the GPS map would know where each car is at every moment. Assuming streets and highways would be off limits for pedestrians (or some kind of sensors would allow for them to be integrated into the GPS map), is such a system possible, or can, for instance, GPS not detect objects to such precise measurements? DRosenbach (Talk | Contribs) 13:10, 1 August 2010 (UTC)[reply]

Yes, it has been done. See DARPA Grand Challenge for an example. The third challenge in 2007 took place on the streets of a disused airbase, although the only other cars were other challengers. 62.56.61.163 (talk) 13:38, 1 August 2010 (UTC)[reply]
(ec) GPS satellite navigation can give adequate location precision though safety would require keeping generous spacing between vehicles and provision for signal shadow areas. There would be concerns about the vulnerability of the radio navigation to interference, whether it is accidental or deliberate jamming. The radio controlled traffic lanes would have to be isolated from all other traffic, comparable to creating a new railway system. With central control, traffic could move very efficiently but it would be constricted by the limited number of entrance/exit points. These points would have to include adequate spaces for acceleration, deceleration and queuing. All possible failure modes need study. Cuddlyable3 (talk) 13:42, 1 August 2010 (UTC)[reply]
You wouldn't just use GPS — the car itself would have localized sensors to detect other vehicles, walls, pedestrians, dogs, etc. It takes more than GPS to navigate through a real-life environment for fairly obvious reasons (GPS can't tell you when an old lady is in the street). Anyway, this is potentially something out there for the future, though aside from the rather copious technical hurdles involved, and the problematic legal ones — how would such a thing be insured? who is at fault when they crash? make no mistake, there will be crashes, no matter how clever the technology is, because that's how things work in the "real world" — there is also a high psychological barrier to being driven around in such a fashion by a computer.
Getting out my crystal ball, I would suggest that unmanned cars will probably only be used in rather limited situations, like guarding a border fence or a military base, where the conditions can be relatively controlled for and the car itself can defer to a remotely-controlling human in the case of anything anomalous. I doubt they will be used for general transportation, especially since if you want someone to drive you somewhere, it is not that expensive to just hire an actual human being. I suspect automatic transportation for general use would only be used on tracked systems, like subways or rail, where the conditions can be easily automatically limited. --Mr.98 (talk) 14:22, 1 August 2010 (UTC)[reply]
(ec) The autonomous vehicle article goes into more detail. Basically GPS is used only for deciding which junctions to take. Each car has short-range LIDAR (IR laser based RADAR) and video-cameras so it can detect what other vehicles are doing, and avoid collisions. There is a car platoon protocol; this allows cars to form convoys. Each car indicates to the car behind that it is about to brake, so that the second car can brake earlier. This allows the separation distance to be less. The lead car decides what speed to travel at and to avoid collisions. If each car communicates its acceleration/braking profile to the others, then the platoon can follow the least capable car, and the separation distance can be very close. Of course, there need to be a way for cars to announce that they are about to leave the platoon; if car in the middle needs to leave, then the car behind it will become a temporary lead and drop back a bit to make some room. The leaving car will then drop back a bit itself, and then leave at the next junction. The car behind will then rejoin the platoon. CS Miller (talk) 14:31, 1 August 2010 (UTC)[reply]
An Italian university is scheduled to have a driverless car driving from Italy to China in October. http://www.dw-world.de/dw/article/0,,5829135,00.html Impressive if true - but I thought it was impractical for a human driven car to do the same route due to wars? 92.29.127.162 (talk) 17:11, 1 August 2010 (UTC)[reply]
Wars? Between who? The link and my searches don't seem to discuss the route, but mention Siberia and Mongolia. In any case, the obvious route from what's been described would be reaching Russia somehow from Italy (many practical ways) then on to Mongolia and China. There are of course other routes, e.g. [16]. All these options have a variety of safety risks, likely a lot of paperwork & money & time & other facets of bureaucracy and probably a bunch of bribes too (for example you'll probably need a 'guide' to drive in China http://www.lonelyplanet.com/thorntree/thread.jspa?threadID=1780257) so while not for the faint hearted is doable and there must be a resonable number of people who do this sort of driving from Western Europe to China every year. Nil Einne (talk) 19:18, 1 August 2010 (UTC)[reply]
Technically, the Italian car isn't exactly driverless. There will be someone sitting behind the wheel the entire time - they'd never get permission to have an entirely unmanned machine to drive in those places. How many times will he have to take control? Lots, I suspect. SteveBaker (talk) 01:46, 2 August 2010 (UTC)[reply]

few question about glycolysis

1)What is the logic of glycolysis?? I mean its main purpose is to provide energy and also it is a multipathway i.e,its intermediates are use in many other pathways... bt do it have any other function other than the above....???

2)glucokinase is synthesized after 2 weeeks of birth. After feeding(insulin) stimulates the glucokinase system. What kind of regulatory mechanism is operational here....??

plz help me by replying fast.....m waiting!!!! —Preceding unsigned comment added by Priyankajoshi7 (talkcontribs) 14:59, 1 August 2010 (UTC)[reply]

As a point of netiquette it is generally considered impolite to ask for a quick response; we are all volunteers here. For future reference, please sign your questions and replies by typing in ~~~~. It's to the left of the '1' (one) key on US-keyboards and to the 3 keys right of the 'L' on UK-keyboards, or press the pen-like button on the standard editing toolbar.
Back to your question, bio-chemistry is not my discipline, but our glycolysis article gives some of the other uses of the intermediates, especially the biochemical logic and intermediates for other pathways sections. CS Miller (talk) 15:46, 1 August 2010 (UTC)[reply]

What is convergent evolution?

If an octopus and I have a common ancestor who possessed what I might call 'proto-eyeness', by which I mean not eyes themselves, as we know them, but everything necessary to the production of descendants with eyes, then can my own eyes and the octopus's eyes be said truly to have evolved independently, and is there really any such thing as convergence? By analogy (no pun intended), a woman with no formal education but wealth and social connections has three daughters, two of whom go to university. The university degrees did not evolve independently of one another: they are each the result of the mother's wealth and social connections (or 'proto-universitiness'). I mention the third daughter because I know some of our cousins have no eyes. 91.107.28.138 (talk) 20:13, 1 August 2010 (UTC)[reply]

Did you read Convergent evolution? A good example: sharks and dolphins share many similarities and look a lot alike. However they are not closely related at all. The converged on many of the same traits because they lived in similar environments and benefited from many of the same adaptations. Friday (talk) 21:17, 1 August 2010 (UTC)[reply]
You are describing parallel evolution rather than convergent evolution. --Tango (talk) 23:27, 1 August 2010 (UTC)[reply]
The term convergent evolution doesn't carry any implication about causes, it only says that two species are more similar now, with respect to a certain feature, than their ancestors were. It is entirely possible to have convergent evolution at the organism level in spite of divergent evolution at the gene level. In the case that you describe, convergence would result from unveiling of a previously hidden similarity. That isn't how convergence normally arises, in the opinion of most biologists, but if it did, it would still be called convergent evolution. Looie496 (talk) 00:32, 2 August 2010 (UTC)[reply]
An interesting recent paper argues that the distinction between "parallel evolution" and "convergent evolution" is artificial and should be discarded. See Convergence and parallelism reconsidered: what have we learned about the genetics of adaptation? Adrian J. Hunter(talkcontribs) 09:28, 2 August 2010 (UTC)[reply]

can loud music (esp. prominent electric guitars, drum, bass) kill cells on a microscope slide?

When imaging cells for sometimes up to six hours at a time, I of course take advantage of my laboratory's nifty sound system ;-)

but I want to ask if thundering bass in the same room can affect my experiments? John Riemann Soong (talk) 21:28, 1 August 2010 (UTC)[reply]

Sonication usually uses ultrasonic frequencies. It seems very unlikely that audio frequency music would be able to disrupt cell membranes. Nimur (talk) 21:42, 1 August 2010 (UTC)[reply]
Well I wasn't thinking disrupting cell membranes, but maybe activating sensitive mechanoreceptors that would set off apoptosis? John Riemann Soong (talk) 22:45, 1 August 2010 (UTC)[reply]
I read one paper where apparently sustained exposure to "low-frequency sonication" could kill cells. Would that be in the audible range? John Riemann Soong (talk) 23:09, 1 August 2010 (UTC)[reply]
Maybe something like this, this or even this? ~AH1(TCU) 23:19, 1 August 2010 (UTC)[reply]
The first link is from 1932 (!) and the others describe killing cells with ultrasound. Bass and sub-bass are infrasound so I don't think they can kill cells (I'm glad off this, although I do get a strange headache after listening to drum and bass) This paper states that "low-frequency sonication" is of 25 kHz, just outside of our hearing range and the opposite end of the spectrum compared to bass. I think that sonication must also be very loud sound, considering that bats produce ultrasound at 130 decibels but don't drop out of the sky as their cells die. Considering all of this I agree with Nimur that it is very unlikely that loud music could kill cells on a slide. Smartse (talk) 00:45, 2 August 2010 (UTC)[reply]
Sound waves have two measures of strength: Their intensity (or amplitude) and their frequency (or tone or note). Both can do damage. High frequency soundwaves are of a higher energy sound than lower frequency sounds of the same intensity, AND louder sounds have more energy. Your hearing can be damaged by high volume infrasound, even if you cannot hear it. Presumably, very high volume, but low frequency, soundwaves could cause damage even on the cellular level. --Jayron32 03:31, 2 August 2010 (UTC)[reply]
Yes, but noise-induced hearing loss occurs because the cells that are damaged have evolved specifically to absorb sound energy - normal cells won't be affected. My physics is pretty rusty but AFAIK the wavelength of low frequency music would be too large to interact with something as small as a cell. What frequencies and decibels do you mean by "very high volume, but low frequency" and can you find a rouce to back up your presumption? Smartse (talk) 08:30, 2 August 2010 (UTC)[reply]
Personally I would be more concerned about the stability of the optics during imaging than killing cells - excessive vibration could shift or blur your images (how long is each exposure?). However if you are not finding this a problem already it's probably ok - but you should check if anyone else in the lab is doing vibration sensitive experiments, or you might find yourself quite unpopular when they check their images/results. Equisetum (talk | email | contributions) 08:32, 2 August 2010 (UTC)[reply]

Units of Time

Are there any natural units for measuring time not based on the movements of the Earth?

Like, say you were far out in space trying to communicate to an alien how long a day on Earth is, what unit of measurement could you use that's universal? Are there any longer than the amount of time it takes hydrogen to perform a hyperfine transition? Just curious. 108.3.173.100 (talk) 00:26, 2 August 2010 (UTC)gejl[reply]

The most natural unit of time is the Planck time. There's a bit or arbitrariness to do with factors of pi, but otherwise it's something that aliens with similar understanding of physics to ours should share. The Planck time is very short, though, so doesn't answer your second question. The Hubble time is very long, but also pretty fundamental. --Tango (talk) 00:30, 2 August 2010 (UTC)[reply]
It's "natural" in a physics-y sense of "being based of fundamental physical quantities". However, it's not particularly "natural" in the colloquial sense of being familiar and easy to recognize. I would also point out that the Planck time has a standard uncertainty of 5×10−5 which is pretty crappy. This is mostly due to our difficulty in measuring G. By comparison, the best atomic clocks have a precision ~5×10−16 at measuring a second as defined to be "the duration of 9,192,631,770 periods of the radiation corresponding to the transition between the two hyperfine levels of the ground state of the caesium 133 atom". If we wanted a really rigorous comparison, I think we would definitely tell them something based on an atomic clock. For a quick and dirty comparison, it would probably make more sense to use a simpler physical process that is easily accessible such as the half-life of a common radioactive decay. Dragons flight (talk) 03:17, 2 August 2010 (UTC)[reply]
Assuming the alien can understand multiplication/exponents, is there anything wrong with applying those the tiny units? Lenoxus " * " 00:49, 2 August 2010 (UTC)[reply]
The primary unit of time, the second, is now defined using an atomic clock. Atomic clocks don't rely on movements of the Earth, Sun, moon or any other celestial body. Dolphin (t) 02:58, 2 August 2010 (UTC)[reply]

Precambrian rabbits and natural fossil exposure

A thought just occurred to me on the subject of Precambrian rabbits. Occasionally, natural events like erosion will expose fossils which date from tens of millions of years prior. Given this, couldn't an organism die immediately on top of or next to such a fossil site, after which the whole thing gets covered in sediment, resulting in "distant neighbor" fossils?

To put it another way: Given this small possibility (unless, as is far more likely, it's not a possibility), why is the fossil record so consistent? Why are there (at least) no Triassic rabbits? Lenoxus " * " 00:46, 2 August 2010 (UTC)[reply]

Because the experts are aware of this issue and take it into account. The rock the recent organism is fossilised in would be different from the rock the old fossils are in, so you would be able to tell that something had happened. Working out the ages of different layers of rock can be a bit of a challenge, but it can be done (in various ways). --Tango (talk) 00:55, 2 August 2010 (UTC)[reply]
As stated by tango, geologists understand the processes by which rock formation and erosion can occur; in fact rocks are often dated by index fossils, which determine their age. Thus, finding a rabbit lying in a rock layer which was immediately on top of a trilobite wouldn't lead people to conclude that the rabbit was really old, rather that the rock layer the rabbit was in was much younger than the one the trilobyte was in. --Jayron32 03:27, 2 August 2010 (UTC)[reply]
I had a strong feeling it would be something like that (since that relates to how they're able to tease apart folded strata); thank you both! So, now I'm wondering… has the situation I described ever happened? That is, are there any known "distant neighbors" (where the rocks are obviously quite different, etc)? If not, any particular reason why not? Lenoxus " * " 04:48, 2 August 2010 (UTC)[reply]
You'll want to follow the threads that lead from the article titled Stratigraphy, which is the science of reading and interpreting rock layers. Undoubtedly, examples exist of local areas where two non-sequential rock layers lie side by side. I cannot think of one specific example, but that's more a function of there being so many rather than being so few. --Jayron32 04:59, 2 August 2010 (UTC)[reply]
The specific term, BTW, for such a situation is an Unconformity. Examples given in that article show pictures of adjascent rock layers which are actually represent gaps of up to 1 billion years. --Jayron32 05:02, 2 August 2010 (UTC)[reply]
It does happen that sometimes fossils get into layers they don't belong in. Most commonly it is a case of older fossils getting incorporated into younger rock -- this can happen if the soft rock they are embedded in erodes away, leaving the fossil behind to be covered by younger sediment. It's much harder for younger fossils to get incorporated into older rock. Looie496 (talk) 05:44, 2 August 2010 (UTC)[reply]

Depression Medication

A while back I saw an article that showed which type antidepressant is more likely to work in what type of people. I can't remember the different categories they had for the type of people. Has anyone seen it? —Preceding unsigned comment added by 76.169.33.234 (talk) 05:04, 2 August 2010 (UTC)[reply]

That's rather vague, and could describe every single antidepressant; each probably works better in some types of depression than others. You could explore Category:Antidepressants yourself to see if you can dig it up. --Jayron32 05:07, 2 August 2010 (UTC)[reply]
The most common types of depression are major depressive disorder and bipolar disorder, and they are usually treated with different types of drugs -- the articles will give you more information. Looie496 (talk) 05:39, 2 August 2010 (UTC)[reply]
There's also a list of antidepressants which might be of use. Smartse (talk) 08:48, 2 August 2010 (UTC)[reply]

Contact between shaft and bearing

hey i want to know on what basis the contact(clearence,angle)between shaft and bearing(journel,liner) because am installing new roller of shaft dia 320 and my liner is of dia 320, its is a semi spherical(only i have liner at bottom half)??? and why???? —Preceding unsigned comment added by 82.129.222.71 (talk) 09:25, 2 August 2010 (UTC)[reply]

There's some basic info here on the hows and whys of journal bearings [17] does yours match any of these types?77.86.94.177 (talk) 12:53, 2 August 2010 (UTC)[reply]

Electrolysis

What are the typical values of current and voltage required for the electrolysis of water? In the current industry what power input do they use, is it mostly of national grid systems? Thanks very much--91.103.185.230 (talk) 11:15, 2 August 2010 (UTC)[reply]

You need to use DC if you want to separate the hydrogen and oxygen. A minimum voltage would be around 1.5 volts DC. In my experience, 24 volts DC works fine; add some baking soda, sodium hydroxide, or sulfuric acid for the electrolyte. Current might be 100 milliamps. --Chemicalinterest (talk) 12:11, 2 August 2010 (UTC)[reply]
The minimum voltage required for water electrolysis is 1.23V (see Electrolysis of water). However the overpotential needed at the electrodes is ~1V (depending on electrodes). Additionally producing the gas at above 1atm pressure also increases the voltage.
The current directly controls how much water is electrolysed per second.
The voltage used therefor will be roughly 2+IR where I is the current, and R is the internal resistance of the electrolysis cell.
Also see http://www.hydrogenassociation.org/general/faqs.asp and this google books Hydrogen fuel: production, transport, and storage By Ram B. Gupta p.162-163
Commercial cells produce work at high pressure. This link http://www.nrel.gov/docs/fy04osti/36705.pdf gives typical figures - there are high and low pressure cells, and electrolyte of KOH is used to reduced cell resistance. As with all processes that use electricity as a feedstock hydroelectric power is common.
Using the wattage figures in the nrel.gov link above you can work out the typical current values using V=~2V after taking into account the efficiency. However note that typical cells are connected in series, not parallel -so the figure may be a sum of the currents going through the cell rather than the supply voltage/current.77.86.94.177 (talk) 12:15, 2 August 2010 (UTC)[reply]

Fennec Hare

Is there any such animal as a Fennec Hare? This page says they are critically endangered, and one has just been born in captivity. However the photo looks like a kitten 'shopped to have bunny ears, and it's supposed to have been born in North Korea. Is this nonsense, propaganda, or maybe possibly true? 213.122.216.120 (talk) 12:53, 2 August 2010 (UTC)[reply]

documentary

Anyone know of a good documentary on bees / wasps / ants? 82.43.88.151 (talk) 13:10, 2 August 2010 (UTC)[reply]